Download as pdf or txt
Download as pdf or txt
You are on page 1of 100

JEE Companion

For JEE MAINS and ADVANCED

MATHEMATICS
MODULE-6
CONTENTS

CHAPTER – 1 : Function
THEORY 01-14
EXERCISE 15-34
ANSWER KEY 35

CHAPTER – 2 : Inverse Trigonometric Function


THEORY 36-44
EXERCISE 44-58
ANSWER KEY 59

CHAPTER – 3 : Limit
THEORY 60-66
EXERCISE 67-79
ANSWER KEY 80

CHAPTER – 4: Continuity
THEORY 81-85
EXERCISE 86-97
ANSWER KEY 98
FU NCTION MATHS-X II-IIT-JEE

CHAPTER

FUNCTION
1. Number System (v) Real Numbers :
The set which contains both rational and irrational
(i) Natural Numbers
numbers is called real number set and is denoted by
The set of numbers {1, 2, 3, 4, ...} are called natural
numbers, and is denoted by N. R. i.e., R = Q  Qc
i.e., N = {1, 2, 3, 4, ...} R = {... –2, –1, 0, 1, 2, 3, ...,
(ii) Integers 5 3 7 1 1 1
The set of numbers {..., –3, –2, –1, 0, 1, 2, 3, ...} are , , , , , , ..., 2, 3,  , ... }
6 4 9 3 7 5
called integers and the set is denoted by I or Z.
 Where we represent; Note:
(A) Positive integers by I  = {1, 2, 3, 4, ...} As from above definitions;
N  I  Q  R, it could be shown that real numbers
= Natural numbers.
 can be expressed on number line with respect to
(B) Negative integers by I = {..., –4, –3, –2, –1} origin as;
(C) Non–negative integers I 0 (or N0)
–3 –2  2 –1 0 1 2 3
= {0, 1, 2, 3, 4, ...} = Whole numbers 2
(D) Non–positive integers I 0 = {..., –3, –2, –1, 0} Intervals :
(iii) Rational Numbers The set of numbers between any two real numbers
A number which can be written as a/b, where a and is called interval. The following are the types of
b are integers, b  0 is called a rational number and interval.
their set is denoted by Q. (i) Closed Interval: [a, b] = {x : a  x  b}
a (ii) Open Interval: (a, b) or ]a, b[ = {x : a < x < b}
i.e., Q  { a, b  I and b  0}. (iii) Semi open or semi closed interval:
b
[a, b[ or [a, b) = {x: a  x < b}
Note:
Every integer is a rational number as it could be 
]a, b] or (a, b]= {x: a < x  b}
a  Inequalities
written as Q = (where b = 1) The following are some very useful points to
b remember:
All recurring decimals are rational numbers.
1.a  b  either a < b or a = b
1 2.a < b and b < c  a < c
e.g., Q  0.3333 ... 
3 3. a < b  –a > –b i.e., inequality sign reverses if
Q = 0.9999... = 1 both sides are multiplied by a negative number
(iv) Irrational Numbers : 4. a < b and c < d  a + c < b + d and a – d < b –
Those values which neither terminate nor could be
expressed as recurring decimals are irrational c.c  R
a 5. a < b  ma < mb if m > 0 and ma > mb if m<0
numbers. (i.e., it cannot be expressed as form),
b 6. 0 < a < b  ar< br if r > 0 and ar> br if r < 0
their set is denoted by Qc ( i.e., complement of Q).  1
1 3 7.  a    2 for a > 0 and equality holds for a=1
e.g., 2,   2, ,  a
2 2
1  1
3, 1  3,  ,  ... etc. 8.  a    – 2 for a < 0 and equality holds for a = –1
3  a

1
MATHS-X II-IIT-JEE FU NCTION
 The Absolute value of A real f s f4
2 number 3 t 2 s
The absolute value (or modulus) 4 of a real number x u 4 t
6 v u
(written |x|) is a non negative real number that w 6 v
8 8 w
satisfies the conditions.
| x | = x if x  0 Now see that f1 is not function from set A to set B,
| x | = – x if x < 0 since there is an element 6  A which is not
associated to any element of B, but f2 and f3 are the
Example:
function from A to B, because under f2 and f3 each
| 2 | = 2, | –5 | = 5, | 0 | = 0 elements in A is associated to a unique element in
From the definition it follows that the relationship B. But f4 is not function from A to B because an
x  |x| holds for any x. elements 8  A is associated to two elements u and
The properties of absolute values are w in B.
(i) the inequality | x |  means that –  x  ; if  Domain:
> 0  Set A is called domain of f i.e. Set of those
(ii) the inequality |x|  means that x  or elements from which functions is to be defined.

 x – ; if > 0
 Co-Domain :
(iii) | x ± y|  |x| + |y| ;
 Here set B is called co–domain of function.
(iv) | x ± y|  | | x | – | y || ; 

(v) |xy| = | x | | y | ;  Range :


x |x|  Set of images of each element in A, is called range
(vi) = (y  0). of f.
y | y|
Note:
2. Definition of Function Range  Co–domain
Let A and B be two non–empty sets. Then a function Real Valued Function :
‘f ’ from set A to set B is a rule which associates All those functions of which domain and Co–
elements of set A to elements of set B such thatAn Domain are subsets of R are called real valued
element of set A is associated to a unique element in functions. In this case for a given function we have
set B. All elements of set A are associated to element to find domain and Range.
in set B. Terms such as “map” (or mapping), Bounded Function :
“correspondence” are used as synonyms for function.
If f is a function from a set A to set B, then we write f : A function ‘f’ is said to be bounded if | f (x) |  m ,
A  B or A 
f
 B. which is read as f is a function forsomefinite‘m’for every x in Domainof f.
from A to B or f maps A to B. Equality of two functions :
Two function f and g are said to be equal functions,
Example :
if and only if
Let A = {2, 4, 6, 8} and B = {s, t, u, v, w} be two
(i)domain of f = domain of g
sets and let f1, f2, f3 and f4 be rules associating
elements of A to elements of B as shown in the (ii) co–domain of f = co–domain of g
following figures. (iii) f(x) = g(x)  x  their domain
s Example :
2 f1 t
4 u If A = {1,2}, B = {10,13}, f : AB, f(x) = x2 + 9
6 v and g = A  B, g(x) = 3x + 7,
8 w then f = g because domains and co–domains of both
f and g are same also
s f2 s
t 2 t f(A)= 10 = g(A) ; f(B)= 13 = g(2)
u 4 u
v 6 v Examples :- 1
w 8 w (i) Determine the values of x satisfying the equality:
f3 |(x2 + 4x + 9) + (2x –3)|
2 s
t = |x2 + 4x + 9| + |2x –3|;
4 u
6 v 1+x f(x).f(x 2 ) 1
w (ii) If f(x) = , show that = .
8 1– x 1 + [f(x)]2 2

2
FU NCTION MATHS-X II-IIT-JEE
Solution: 1 x2
(i) The equality |a + b| = |a| + |b| is valid if and only if
both summands have the same sign. Since 1  x 2 1 x2 1
  
x2 + 4x + 9 = (x + 2)2 + 5 > 0 at any values of x, 1  x  2  1  x 2 2  2x 2
2
the equality is satisfied at those values of x at which
2x – 3  0, i.e., 1  x 2
at x  3/2.
3. Domain and Range
1  x2 1  x2 Domain =All possible values of x for which f(x)
(ii) f(x2) = 
1  x 2 (1  x)(1  x) exists.
 1 x  1 x2 Range=For all values of x, all possible values of
  f(x).
f(x).f(x 2 )  1  x  1  x 1  x  A
  B
1  [f(x)]2 1  x 
2
a p
1 2
1  x  b q

c r
d s

4. Domain and Range of some important functions


FUNCTION DOMAIN RANGE
(y = f(x)) (i.e. values taken by x) (i.e. values taken by f(x))
Algebraic Functions
R, if n is odd
(i)xn, (n  N) R = (set of real numbers)
R+ {0}, if n is even
1 R – {0}, if n is odd
(ii) , (n  N) R – {0}
xn R+, if n is even
R, if n is odd R, if n is odd
(iii)x1/n, (n  N)
R+  {0}, if n is even R+ {0}, if n is even
1 R– {0}, if n is odd R – {0}, if n is odd
(iv) , (n  N)
x1/n R+, if n is even R+, if n is even
Trigonometric Functions
(i) sin x R [–1, 1]
(ii)cos x R [–1, 1]

(iii) tan x R– (2k +1) , k  I R
2

(iv)sec x R – (2k + 1) , k  I (–, –1]  [1, )
2
(v) cosec x R – k, k  I (–, –1]  [1, )
(vi) cot x R – k, k  I R
Inverse Circular Functions
  
(i) sin–1 x [–1, 1]  2 , 2 
 
(ii)cos–1 x [–1, 1] [0, ]
  
(iii) tan–1 x R  , 
 2 2
  
(iv)cosec–1 x (–, –1] [1, )  2 , – {0}
 2 

3
MATHS-X II-IIT-JEE FU NCTION

 
(v) sec–1 x (–, –1] [1, ) [0, ] –  
 2
(vi) cot–1 x R (0, )
Exponential Functions
(i) ex R R+
(ii)e1/x R – {0} +
R – {1}
(iii) ax, a > 0 R R+
(iv) a1/x, a > 0 R– {0} +
R – {1}
Logarithmic Functions
(i) logax, (a > 0) (a 1) R+ R
1
(ii)logxa =
log a x R+ – {1} R – {0}
(a > 0) (a  1)
Integral part Functions
(i) [x] R I
1 1 
(ii) R – [0, 1)  ,n  I – {0}
[x] n 
Fractional part Functions
(i) {x} [0, 1)
R
1
(ii) R–I (1, )
{x}
Modulus Functions
(i) |x| R R+ {0}
1
(ii) R – {0} R+
|x|
Signum Function
|x|
sgn(x) = ,x0
x R {–1, 0, 1}
= 0, x = 0
Constant Function
say f(x) = c R {c}
–1 2
(ii) f(x) = sin 4-x
5. Rules of domain (iii) f(x) = ln (– 2 + 3x – x2)
Dom (f + g + h.......) Solution :
= Dom f  Dom g  Dom h .......... (i) For f(x) to be defined – 1  x  1,
Dom (f – g) = Dom f  Dom g
Dom (f × g × h ......)
x  2  0 i.e., x  2 ,
= Dom f  Dom g  Dom h .......... x + 1 > 0 i.e., x > – 1 and
Dom (f / g) x + 1  1 i.e., x  0
= Domf  Dom g – {x : g (x) = 0} so, domain of f : (– 1, 0) (0, 1]

Examples :- 2 (ii) f(x) = sin–1 4  x 2


Find the domain of following functions: for f(x) to be defined 0  4 – x2 1
sin –1 x 1  x2 – 4  0 and x2 – 3  0
(i) f(x) = 2 + x+2 +
log10  x + 1  x  [ – 2, 2] and

4
FU NCTION MATHS-X II-IIT-JEE
1 2
x (– , –
16
]   3,    1 
=  1
1
1
 x 1 (x  1)2
 x   2  3    3, 2 
Note:
so domain (f) : x   2  3    3, 2  Domain of fog(x) is x  R – {1}
(iii) f(x) = ln (–2 + 3x – x2) (g) of (x) = g(f(x)) = g(x2 + 1) = g(u),
for f(x) to be defined – 2 + 3x – x2> 0 1 1 1
where u = x2 + 1 =  2  2
 x2 – 3x + 2 < 0 u  1 x  1 1 x
 (x – 1) (x – 2) < 0  x  (1, 2)
so domain (f) : x  (1, 2) Note:
Domain of gof(x) is x  R  {0}
Examples :- 3
Find the range of the following functions: Examples :- 5
1  2 + x, if x > 0
(i) f(x) = If f(x) =  , then find (f) of (x).
8 - 3sinx  2 - x, if x < 0
(ii) f(x) = x2 – 7x + 5 Solution :
(iii) f(x) = log2 (log1/2 (x2 + 4x + 4)) f(x) = 2 + |x|
Solution: ff(x) = 2 + |f(x)| = 2 + 2 + |x| = 4 + |x|
1
(i) f(x) = . We know that
8  3sin x Examples :- 6
–1  sin x  1
 – 3  3 sin x  3 5  8 – 3 sin x  11 Let f(x) = sin-1x , g(x) =  3  x  x  1 . Find f + g,
 1 1 f– g, f . g and f /g
 Range (f) =  ,  Solution :
11 5 
2
(ii) f(x) = x – 7x + 5 (f+g)x = sin 1 x   3  x 1  x  ,defined on -1 x1
2
 7  29 (f–g)x = sin 1 x  3  x 1  x  , defined on -1 x1
 f(x) =  x   
 2 4
(f–g)x = sin 1 x   3  x 1  x  , defined on -1 x1
 29 
Range (f) =   ,   sin 1 x
 4  (f/g)x = , defined on –1 < x 1
2
(iii) f(x) = log2 (log1/2 (x + 4x + 4))  3  x  x  1
since 0 < log1/2 (x2 + 4x + 4) < x  Domain (f)
 – < log2 (log1/2 (x2 + 4x + 4)) < Examples :- 7
Range (f) = (– , ) 1
Solve for x, log x ≥ 1
4
Examples :- 4
Key concept:
1
If f(x) = x2 +1, g(x) = , then find(fog) (x) and (g) First make the base of log same both the sides. Then
x -1 solve.
of (x).
 Common Mistake:
Solution :
Given, f(x) = x2 + 1 .....(1) 1 1
Given. log x ≥ 1  log 1 x  log 1
1 4 4 4
4
g(x) = .....(2)
x 1  After this step students used to solve in this
 1  manner
Now (fog) (x) = f(g(x)) = f   = f(z),
 x 1 1 1
log 1 x  log 1  x  They forget to check the
1 4 4
4 4
where z =
x 1 base of log. Hence the answer is wrong.
2
= z + 1 [ f (x)  x 2  1] Solution :

5
MATHS-X II-IIT-JEE FU NCTION
1 e.g.If f : R+R is defined by y = log2x,then
Given. log 1 x  1  log 1 x  log 1 f(x) is Onto function.
4 4 4
4
(iv) Into Function :
We know If there exist one or more than one element in
 a  a2 if b  1 the Co–domain Y which is not an image of any
log b a1  log b a 2   1
 a1  a 2 if 0  b  1 element in the domain X. Then f is into.
The base of log lying between 0 and 1 hence inequality In other words f : A  B is an into function if
1 it is not an onto function.
sign will change, hence x  . Also log x is defined e.g. Let f : R  R is defined by y = x2 + 1, then
4
f(x) is an into function. But when f : R  R+ is
only when x>0.
defined by y = x2 + 1, then f(x) is not into
 1 function.
Hence the answer is x   0, 
 4 (v) One-one onto Function or bijection :
6. Classification of Function A function f is said to be one-one onto if f is
The following are the kinds of function : one-one and onto both.
(i) One–One Function (Injective) : (vi) One-one into Function:
If each element in the domain of a function has A function is said to be one-one into if f is one-
a distinct image in the co–domain the function one but not onto.
is said to be one–one function and is also (vii)Many one-onto Function:
known as Injective Function. A function f : AB is many one-onto if f is
e.g. f : R  R+ given by y = ex onto but not one-one.
g : R  R, g(x) = 3x – 7 (i) f : R R+ {0}, f(x) = x2 .
are one – one functions. (ii) f : R  [0,  ), f(x) = |x|
or, f : A B is one – one (viii) Many one-into Function:
 a  b  f(A) f(B) for all a, b  A A function f is said to be many one-into if it is
    f(A) = f(B) a = b for all a, b  A neither one-one nor onto.
(i) f : R R, f (x) = sin x
(ii) Many–One Function : (ii) f : R R, f(x) = |x|
If there are two or more than two elements of
domain having the same image then f(x) is (ix) Identity Function:
called Many – One function. Let A be any set and the function f : A A be
e.g. f : R  R+ f(x) = x2 + 4 defined as f(x) = x, x  A i.e. if each element
g:RR +
g(x) = x + x4 +x2 + 4
8 of A is mapped by itself then f is called the
Both functions are many one identity function. It is represented by IA.
If the graph of y = f(x) is given and a line If A = {x, y, z} thenIA = {(x, x),(y, y), (z, z}
parallel to x–axis cuts the curve at more than
one point then function is many one.
or, f : A  B is a many – one function if there Note :
exist x, y  A such that x y  but f(x) = f(y). dy
1. If domain of f(x) is continuous and > 0,  x
e.g y = sin x, y = cos x, y = tan x, y = x2, y = x4, dx
etc are many one functions. in domain then f is One – One, where equality
exist at discreate point.
(iii) Onto Function (Subjective) :
dy
Let f : X Y be a function. If each element in 2. If domain of f(x) is continuous and < 0,  x
the co–domain Y has at least one pre–image in dx
the domain X i.e. Range f = Co domain, then f in domain then f is One – One, where equality
is called onto. exist at discreate point.
Onto function is also called subjective and if 3. If a continuous function f(x) which has either
function be both one–one and onto then local minima or local maxima or both then f(x)
function is called Injective. will be Many – One
or, f : A B is a surjection if for each b  B 4. Every even function is Many – One
[x   1,1 & f (x)  0] a  A such that f(A) = b . 5. Every periodic function is Many – One

6
FU NCTION MATHS-X II-IIT-JEE

Examples :- 8 Since f(x) is an onto function b–1 = 2  b = 3


Let f : (–, )  [2, ) be a function defined by 7. Even and Odd function
Even function
f(x) = x 2  2a  a 2 , a  R. Find a for which f is
If we put (–x) in place of x in the given function
onto. and if f(–x) = f(x),  x domain then function f(x)
Solution : is called even function.
For f to be onto range of the function should be [2, ).
So, a2 – 2a = 4 Odd function
If we put (–x) in place of x in the given function
a=1± 5. and if f(–x) = – f(x),  x  domain then f(x) is
called odd function.
Examples :- 9 Properties of even and odd Function
x +1 (i) The product of two even functions is even
If f: R  R where f ( x ) = 2
, find whether
x + x +1 function.
f(x) is one – one or many one. (ii) The sum and difference of two even functions
Key concept : is even function.
If f(x) is a rational function then f(x1) = f(x2) will (iii) The sum and difference of two odd functions is
always be satisfied when x1 = x2 in the domain. odd function.
Hence we can write (iv) The product of two odd functions is even
f ( x1 ) - f ( x2 ) = ( x1 - x2 ) g( x1 , x2 ) = 0 where g ( x1 , x2 ) function.
(v) The product of an even and an odd function is
is some function in x1 and x2. Now if g ( x1 , x2 ) = 0 odd function.
gives some solution which is different from x1 = x2 (vi) The sum of even and odd function is neither
and which lies in the domain, then f is many – one even nor odd function.
else one – one.
Solution : Examples: - 11
x 1 x 1 Determine the nature of the following function for
f (x1 )  f (x 2 )  2 1  2 2 even and odd:
x1  x1  1 x 2  x 2  1
One solution of this is obviously x1 = x2. Also we have 
(i) f(x) = log x + x 2 + 1 
got a relation in x1 and x2 and for each value of x1 in
the domain we get a corresponding value of x2 which  ax - 1 
(ii) f(x) = x  x 
may or may not be same as x1.  a +1
1 Solution :
If x1 = 1 we get x 2   x1 , and both lies in the
2 (i) f(x) = log (x  x 2  1)  f(–x) = log
domain of f.
Hence we have two different values x1 and x2 for which ( x  x 2  1)
f(x) has the same value.  1  2 –1
Hence ‘f’ is many one. = log   = log (x + x + 1 )
2
 x  x 1
Examples :- 10 = –log (x + x 2 + 1 ) = – f(x)
Let f: R  [2, ) defined by
2
So, f(x) is an odd function
f(x) = x + 2 x + b is an onto function, then find the
value of b.  ax 1
Key concept: (ii) We have f(x) = x  x


Given function is an onto function. Hence range  a 1
should be to codomain.
Solution : f(– x) = – x = f(x)
2
Given f(x)= x2 + 2x + b =  x + 1 + b - 1 . So, f(x) is an even function.
Hence range of f(x) is [b–1, ).

7
MATHS-X II-IIT-JEE FU NCTION
8. Explicit and Implicit Function  Properties of G.I.F :
Explicit Function (i) [x] = x if x is integer
A function is said to be explicit if it can be expressed (ii) [x + I] =[x] + I, if I is an integer
directly in terms of the independent variable. y = f(x) (iii) [x + y]  [x] + [y]
or x = (y) (iv) If [ (x)]  I then (x)  I
Implicit Function (v) If [ (x)]  I then (x) < I + 1
A function is said to be implicit if it cannot be (vi) If [x] > n  x  n + 1
expressed directly in terms of the independent (vii)If [x] < n  x < n, n  I
variable. (viii)[–x] = – [x] if  x  I
ax2 + 2hxy + by2 + 2gx + 2fy + c = 0 (ix) [– x] = – [x] – 1 if x  Integer
(x) [x + y] = [x] + [y + x – [x]]  x, y  R
9. Increasing and Decreasing Function 1  2 n  1
 
 Increasing Function (xi) [x]+  x     x    ....   x 
A function f(x) is called increasing function in the  n  n  n 
domain D if the value of the function does not = [nx] ; n  N
decrease by increasing the value of x. 
If x1 > x2 f(x1) > f(x2)  Fractional Part:
or x1< x2 f(x1) < f(x2) or f'(x) > 0 for increasing Fractional Part of any real number is defined as the
and f'(x)  0 for not decreasing. difference between the number ‘x’ and it’s integral
 Decreasing Function value ‘[x]’ and is symbolically denoted as {x}.
A function f (x) is said to be decreasing function in Thus, {x} = x – [x], e.g. if x = 5.68, then [x] = 5 and
the domain D if the value of the function does not {x} = 0.68.
increase by increasing the value of x (variable). If If x is an integer  x = [x]  {x} = 0
x1 > x 2  {[x]} = 0
 f(x1) < f(x2) or x1< x2 If x   0, 1 , then [x]=0  {x}=x
 f(x1) > f(x2) or f'(x) > 0 for decreasing and x  1, 2  , then [x]=1  {x}=x-1
f'(x)  0 for not Increasing. Y
10. Greatest Integer and Fractional Part (0, 1)
y={x}
 Greatest Integer:
Any real number x can always think of lying
between two consecutive integers say P and P+1. X
i.e. P  x < (P + 1). That means, there always exist -2 -1 0 1 2 3
an integer, say ‘P’ which is just less than or equal to Domain  R;
x. This unique ‘P’ is called the greatest integral Range  [0,1);
value of x and is symbolically denoted as [x] i.e. [x] Period  1;
stands for the greatest integer that is less than or Nature  neither even nor odd
equal to x. IMPORTANT POINTS:
e.g. x = 3.54  3 < x < 4  [x] = 3, x = –2.95
 0  {x} < 1
–3 < x <–2  [x] = – 3
 [{x}] = 0, {[x]} = 0
It is obvious that if x is integer, then [x] = x.
Y  x – 1 < [x]  x, 0  {x} < 1
2 0, if x  integer
y=[x]  {x} + {–x} = 
1 1, if x  integer
Examples :- 14
-2 -1 O 1 2 3 X
1 1
-1 Solve for x, [ x] + = 2+
x - 2.7 x - 2.7
-2
 Common Mistake:
Domain  R;  Student used to solve in this manner
Range I;  First they cancel the common term both the
Period  non periodic;Nature  neither even nor sides.  [x]=2  x   2, 3 .
odd

8
FU NCTION MATHS-X II-IIT-JEE
Solution :  Properties of Periodic Function :
First step is correct. Cancel common term. But If f(x) is periodic with period T, then
1 f(x) is periodic with period T.
The important point is that is not defined when f(x + c) is periodic with period T.
x  2.7
x=2.7, hence exclude this from the solution set. f(x) ± c is periodic with period T.
11. Signum Function T
f(ax + b) has period , i.e., period is affected
|a |
The signum function f is defined as
only by coefficient of x where; a, b, c, are constants
1, if x  0 | x |
  , x0 with a, b  0.
Sgn x = 0, if x  0 =  x If f(x) and g(x) be two periodic function with period
 –1, if x  0  0 , x  0 p & q respectively, then their any combination will

be periodic function with one period equal to
12. Modulus Function L.C.M of p & q provided L.C.M of p & q exists.
 x, x  0 Note :
It is given n  N by y = |x| = 
 –x, x  0 All periodic functions can be analyzed over an
interval of one period within the domain as the
 Properties of Modulus function: same pattern shall be repetitive over the entire
(i) |x|  a  – a  x  a domain.
(ii) |x|  a  x  – a or x  a
(iii) |x + y| = |x| + |y|  x, y 0 or x  0, y Examples :- 15
Find the period of f(x) = |sin x| + | cos x|
(iv) |x– y| = |x| – |y|  x  0 & |x|  |y| or x  0 and Solution :
y  0 and |x|  |y| |sin x| has period , |cos x| has period 
(v) |x ± y|  |x| + |y| Hence, according to the rule of LCM, period of f(x)
(vi) |x ± y|  |x| – |y| must be .
 
13. Periodic Function But | sin   x  | = |cosx| and
 Definition : 2 
A function f(x) is said to be periodic function if, 
| cos (  x) | = |sin x|
there exists a fixed positive real number T 2
independent of x, such that, f(x + T) = f(x)  x   
Domain & x + T  domain. Since,   , period of f(x) is
2 2
T is called one of the period of the function
In other words, a function is said to be periodic Examples :- 16
function if its each value is repeated after a definite If f(x) = sin x + cos ax is a periodic function, show
interval. that a is a rational number.
Here the least positive value of T (independent of x) Solution :
is called the fundamental period of the function. 2 2
Clearly f(x) = f(x + T) = f(x + 2T) = f(x + 3T) = . . . Period of sin x = 2 = and period of cos ax =
1 a
 For example :
2 2
(A) sin x, cos x, sec x and cosec x are periodic functions  Period of sin x + cosax = L.C.M of and =
with period 2. 1 a
LCM of 2 and 2 2
(B) tan x and cot x are periodic functions with period . = where  is the H.C.F. of 1
H.C.F of 1 and a 
(C) |sinx|, |cosx|, |tanx|, |cotx|, |secx|, |cosecx| are and a.
periodic functions with period . 1 a
(D) sinnx, cosnx, secnx, cosecnx are periodic functions with Since  is the H.C.F of 1 and a, and should be
 
period 2 or  according as n is odd or even. both integers.
(E) tannx and cotnx are periodic function with period 
whether n is odd or even.

9
MATHS-X II-IIT-JEE FU NCTION
1 Solution :
1 a n Given f(x) = n(x 2  3x  1)
Suppose = m and = n, then  = , where n, m
  a m 2x  3
  f (x)  2  0  x [1, 3]
(x  3x  1)
n
 I i.e., a = . Which is a strictly increasing function. Thus f(x) is
m
injective, given that f(x) is onto. Hence the given
Hence, a is rational numberwith period = 2
function f(x) is invertible.
Now f (f–1(x)) = x
14. Inverse Function
 n((f  1 (x)) 2  3(f 1 (x))  1)  x
If f : X  Y be a function defined by y = f(x) such
that f is both one–one and onto, then there exists a  (f 1 (x)) 2  3(f 1 (x))  1  e x  0
unique function g : Y  X such that for each y 
3  9  4.1(1  e x ) 3  (5  4e x )
Y, g(y) = x. The function g so \ defined is called the  f 1(x)  
inverse of f and denoted by f –1.Also f is the inverse 2 2
of g and the two functions f and g are said to be 3  (5  4e x )
inverse of each other.  f–1(x)  (as f 1 (x)  [1, 3] )
2
f (f 1 (x))  x,  x  Y and
3  (5  4e x )
1
f (f (x))  x,  x  X Hence f–1(x) 
2
Note:-
That f and f –1 are symmetric about the line y = x.
Examples :- 18

Find the inverse of the function
 Method of finding inverse of a function :
1. If you are asked to check whether the given  x, x <1
 2
function y = f(x) is invertible, you need to check f(x) =  x , 1  x  4 .
that y = f(x) is one–one and onto. 
8 x , x>4
2. If you are asked to find the inverse of a objective
function f(x), you do the following : if f –1 be the Solution :
inverse of ‘f ’ , then f(f –1(x)) = x . Apply the formula  x, x 1
of f on f –1(x) and use of the above identity to solve for  2
Given f(x) =  x , 1  x  4
f –1(x). 
Some standard functions given below along with 8 x , x4
their inverse functions Let f(x) = y  x = f–1(y) .......(i)

FUNCTION INVERSE  y, y 1
FUNCTION 
(i) f : [0, )  [0, ) f – 1 : [0, ) [0, )  x   y, 1  y  4
defined by f(x) = x2 defined by f – 1 (x)  2
y , y2
4
= x  64 64
(ii)    f – 1 : [–1, 1]  
f :   ,   [–1, 1]  y, y 1
 2 2   
  2 , 2  defined by 

defined by f(x) =    f–1(y) =  y, 1  y  16 [From (i)]
sinx f – 1 (x) = sin–1x  2
(iii) f : [0, ]  [–1, 1] f – 1 : [–1, 1]  [0, ] y , y  16
 64
defined by f(x) = defined by f – 1 (x) =
cosx cos–1x 
 x, x 1
–1

Examples :- 17 Hence f (x) =  x , 1  x  16
Find the inverse of the function  2
f(x) = ln( x 2 + 3 x + 1); x  [1, 3] assuming it to be an x , x  16
 64
onto function.

10
FU NCTION MATHS-X II-IIT-JEE

Examples :- 19 Solution :
Find the period of functionsin4x + tan2x. Given, (x) = 2x(x – 1)
Solution : log(x) = x(x – 1) loge2
  1
Period of sin4x is , also period of tan 2x is .  f '(x)  (2x  1)log e 2
2 2 f (x)
 ’(x) = 2x(x – 1)loge2 (2x – 1)
Hence period of f(x) is
2
Thus (x) is an increasing function in [1, ), therefore,
 Properties of Inverse Function
(x) is a one – on function.
(i) The inverse of a bisection is unique.
Also range of f(x) is [1, ) which is equal to co –domain.
(ii) If f: A  B is a bisection& g: B  A is the inverse
Hence the function is also onto.
of f, then fog = IB and gof = IA, where IA and IB are 

identity functionson the setsA and B respectively.  TO FIND  – 1(x):


Note that the graphs off and g are the mirror images Let f – 1 be the inverse function of f, then by rule of
of each other in the line y = x. As shown in the

identity f f 1  x   x 
figure given below a point (x, y) corresponding to

f -1 ( x ) f -1 ( x )-1  =x
y = x2(x  0) changes to (y, x) corresponding to 
f f 1 (x)  2
2
y = + x , the changed form of x = y .

  log2 x  f 1 (x)   f 1 (x)
y x y
x= y y= x
y=x 1  1  4log2 x
f 1 (x) 
y = x2 2
2
y=x
1  1  4log2 x
x y 0 x but f 1 (x) 
0 0 2
fig. 1 fig. 2 fig. 3
1  f 1 (x)  1  4log 2 x  1 and x  1 
(iii) The inverse of a bisection is also a bisection.  
 Therefore only positive sign is acceptable.
(iv) If f & g are two bisections f: A  B, g :
15. Composite Function
B  C then the inverse of gof exists &
(gof)–1 = f–1og–1 If f : A  B and g : B  C are two function then
thecomposite function of f and g, gof A  C will

Examples :- 20 be defined as gof (x) = g [f(x)],  x A.

The function : [1, )  [1, ) is defined by (x)=2x(x


 Properties of Composite Function
– 1)
, find  – 1(x). 

Key concept: (i) If f and g are two functions then for composite of

First check the function for one – one and onto. And two functions fog  gof .

if function is one – one and onto then find inverse (ii) Composite functions obeys the property of


using the identity f f -1  x  = x  associatively i.e. fo (goh) = (fog) oh.

11
MATHS-X II-IIT-JEE FU NCTION
(iii) Composite function of two one-one onto functions (iv) If f(x + y) = f(x) + f(y)  f(x) = x
if exist, will also be a one-one onto function. (v) if f(x + y) = f(x) = f(y)  f(x) = k, here k is constant
Algebra of function (vi) By considering a general nth degreepolynomial
(i) (fog) (x) = f [g(x)] and writing the expression

(ii) (fof) (x) = f [f(x)] 1 1


f(x) . f   = f(x) + f    f(x) =  xn + 1

x x
(iii) (gog) (x) = g[g(x)] 


(B) Algebraic Functions
(iv) (fg) (x) = f(x) . g(x)

(v) (f ± g) (x) = f(x) ± g(x) (i) Polynomial function:



A function having the form
f (x)
(vi) (f/g) (x) = , g(x)  0 y = a0 + a1x + a2x2 + a3x3 + .......... anxn,
g(x)
wherea0, a1, a2 ....... an are realconstant, an 0
 Composite functions in not commutative
and n  N called rational integral function
 Let f and g are two functions then if f & g are
orpolynomial of degree n.
injective or subjective or injective then "gof"
(ii)Rational Function:
also injective or subjective or injective.
The ratio of two polynomial is called Fraction
Rational function or simply rational function.
16. Even and Odd function
x12  x 2  1
e.g.y =
 Even Extension: x6  x 4  1
If a function f(x) is defined on the interval
[0, a], 0  x  a  – a  – x  0 we define f(x) in (iii) Irrational Function:
the [– a, 0] such that f(x) = f(–x). Functions with operations of addition,

f (x) : x  [0, a] subtraction, multiplication, division and raising to


Let I(x) = 
f (–x) : x  [–a,0] power with non-integral rational exponent are

called irrational functions.
 Odd Extension:
x 3  1  x11
If a function f(x) is defined on the interval (I) y = x (II) y =
x2  x  1
[0, a], 0  x  a  – a  – x  0
Such type of function are called Irrational
 x  [–a, 0], we define f(x) = – f(–x) Let I be the 

function.
f (x), x  [0,a]
odd extension then I(x) = 
 –f (–x), x  [–a,0] (iv) Transcendental function :
17. Some very important point All those function who has infinite terms while

(A) If x, y are independent variables then: expanded are called transdental function. for

(i) If f(x, y) = f(x) + f(y) then  f(x) = k logx example alltrigonometrically function.

(ii) If f(x y) = f(x) . f(y) then  f(x) = xn, n  R Inverse trigonometrically function, exponential

(iii) If f(x + y) = f(x).f(y)  f(x) = akx function, logarithmic function etc.

12
FU NCTION MATHS-X II-IIT-JEE
e.g. f(x) = sin x, y = cos–1 x Y
y = f(2x)
y = f (x)
y = logex, y = loge x  sin 1 x
y = f(x/2)
(C) Mapping :
X X
O Stretched the graph
One-one or injective mapping or homomorphism. If
2 times horizontally
f : A  B is one-one mapping A has m element and compressed the graph
2 times horizontally
B has n element hence the no. of mappings = Y

See more examples about the same.


 n pm , n  m
 Y
 0 , n  m
x1 x1 2x1 4x1
y = f(2x) y = f(x/2)
(D) If graph of y = f(x) be known then to find the compressed (stretched
(made half) double)
graph of
(i) y = f(x – a) or y = f(x + a) X

To find y = f(x – a) (Let a = 2) (iv) y = f (–x) :


Y Y
y = f(–x)
y = f(x+2) y = (x) y = f(x–2) y = f(x)

shifted
x
y = f(x) 2 2 O
2 units O X
Reflection of y = f(x) w.r.t. axis of y is y = f(–x)
towards left shifted y = f(x)
horizontally 2 units (v) To find y = k f (x) :
distance rightwise 

(ii) y = f (x) + a : or y = f (x) – a   Rule – Stretch the previous graph k

(Let a = 2) timesvertically

Shifted vertically up e.g. see below y = 2 sin x, y = 3 sin x


the pervious graph Y
Y
5 y = f(x) by 2 units 1 y = 3 sinx
4 2 1 y = 2 sinx
3 y=f(x) +2 1 y = sinx
2 2 2
X
O 
1 y=f(x)
0 2 X
2
–1 y=f(x) –2
–2 2
–3 Shifted vertically down (vi) y = – f(x) :
the pervious graph Y
y = f(x) by 2 units
y = f(x)
(iii) y = f(x/a) or y = f(ax) : (Let a = 2, 1/2)
X

y = –f(x)

13
MATHS-X II-IIT-JEE FU NCTION
Reflection of y = f(x) w.r.t. axis of x isy = –f(x)  Rule :
(vii)To find y = f|x| : Take the image of the portion line below axis of x
Y about axis of x. Remain as it is the portion above
the axis of x.

y = f(x)
X Examples :- 21

Y Draw the graphs of the following functions:


y = f |x|
(i) y = |sin x|, x  [0, 2]
(ii) y = sin |x|, x  [– 2, 2]
X (iii) |y| = sinx, x  [– 2, 2]

(iv) |y| = –sinx, x  [– 2, 2]
 RULE:
Solution:
Neglect the graph lying in IInd and IIIrd quadrant
and, Take the image of graph lying in I and IVth
1
quadrant w. r. t. axis of y. (i)
O 2
The original graph including its image is called
y = f | x |.
Here we took the image of the portion lying in first 1
(ii)
quadrant about axis of y and left the portion which -2 O 2
-1

was lying in second quadrant.

(viii)To find y = |f(x)| : (iii) 1


 2 O 2
Y -1
y = f (x)

X 1
O (iv)
O 2
 2 -1
Y
y = f |x|

X
O

14
FU NCTION MATHS-X II-IIT-JEE

EXERCISE # 1
Based On Domain 8. Find the range of the following function,

 5x  x 2  y  log 7 ( 2(sin x  cos x)  5)


1. Domain of y  log10  :
 4  (A) R (B) Z
(A)(0, 5) (B) [1, 4] (C) [log7 4, log7 5] (D) [2 log7 3, 2]
(C)(–, 0)  (5, ) (D) (–, 1)  (4, )
9. Which of the following function (s) has the range
2. The domain of definition of
[–1, 1]
 log0.3 (x  1)
f(x) = is: (A) f(x) = cos (2 sin x)
x 2  2x  8
(A) (1, 4) (B) (–2, 4)  1 
(B) g(x) = cos 1  2
(C) (2, 4) (D) [2, )  1 x 

3. The function (C) h(x) = sin (log2 x)


2
f(x) = cot–1
(x  3)x + cos–1 x  3x  1 is (D) k(x) = tan (ex)
defined on the set S, where S is equal to:
Based On Kinds of functions
(A) {0, 3} (B) (0, 3)
(C) {0, –3} (D) [–3, 0] 10. Let f : R  R be a function defined byf(x) =
x 2  2x  5
1  2 |
x | is :
4. The domain of sec   is x2  x 1
 4 
(A)One-one and into
(A) R (B) R – (–1, 1)
(C) R – (–3, 3) (D) R – (– 6, 6) (B)One-one and onto
(C)Many-one and onto
5. The domain of the function
f(x) = 24 – xC3x – 1 +40 – 6xC8x – 10 is - (D)Many-one and into
(A) {2, 3} (B) {1, 2, 3} 11. The function f: [2, )  Y defined by
(C) {1, 2, 3, 4} (D) None of these f(x) = x2 – 4x + 5 is both one–one & onto if:
Based On Range (A) Y = R (B) Y = [1, )

1 (C) Y = [4, ) (D) Y = [5, )


6. The range of the function y = is :
2  sin 3x
12. Let f : R  R be a function defined by
1  1 
(A)  , 1 (B)  , 1 f(x) = x3 + x2 + 3x + sin x. Then f is:
3  3 
(A)One– one & onto
1 
(C)  , 1 (D) None of these (B)One –one & into
3 
(C)Many one & onto
7. The value of the function
(D)Many one & into
x 2  3x  2
f(x) = 2 lies in the interval - 13. Which of the following function from
x x 6
A = {x: –1  x  1} to itself are bisections-
1 
(A) (– , ) –  , 1 (B) (– , ) (A) f(x) = x/2 (B) g(x) = sin (x/2)
5 
(C) (– , ) – {1} (D) None of these (C) h(x) = |x| (D) k(x) = x2

15
MATHS-X II-IIT-JEE FU NCTION

Based On Inverse function Based On Even and odd function


14. If f (x) = x3 – 1 and domain of f = {0,1, 2, 3}, 20. Which of the following is an even function?
then domain of f –1 is - ax  1
(A) {0, 1, 2, 3} (B) {1, 0, –7, –26} (A) x (B) tan x
ax 1
(C) {–1, 0, 7, 26} (D) {0, –1,– 2, –3}
a x  a x ax 1
x
e –e –x (C) (D)
15. The inverse of the function y = is 2 ax  1
ex  e–x
21. Which of the following function is an odd
1 1 x 1 2x
(A) log (B) log function
2 1 x 2 2x
1 1 x (A) f(x) = 1  x  x 2 – 1 x  x2
(C) log (D) 2 log (1+ x)
2 1 x  ax  1
(B) f(x) = x  x 
 a 1
Based On Composite function
 1 x 
16. The function f(x) is defined in [0, 1] then the (C) f(x) = log  2
 1 x 
domain of definition of the function
(D) f(x) = k (constant)
f[n (1–x2)] is given by :
(A) x  {0} Based On Miscellaneous
(B) x [– 1  e –1]  [1 + 1  e ] 22. The set of points for which
(C) x  (– , ) f(x) = cos (sinx) > 0 contains -
(D) None of these (A) (– , 0] (B) [–1, 1]

Based On Periodic function (C) (–, ) (D) All are correct

17. If f : R  R is a function satisfying the property 23. If [x] stands for the greatest integer
f(x +1) + f(x + 3) = 2  x R then the period function, then the value of
(may not be fundamental period) of f (x) is 1 1  1 2   1 999 
 2  1000  +  2  1000  +.........+  2  1000 
(A) 3 (B) 4      
(C) 7 (D) 6 (A) 498 (B) 499 (C) 500 (D) 501
18. The fundamental period of the function:
24. Let the function f(x) = 3x2 – 4x + 8 log (1 + | x |) be
f(x) = x + a – [x + b] + sin x + cos 2x
defined on the interval [0, 1]. The even extension
+ sin 3x + cos 4x + …. + sin (2n – 1) x+ cos
of f(x) to the interval [–1, 0] is -
2 nx for every a, b R is:
(A) 3x2 + 4x + 8 log(1 + | x |)
(where [.] denotes the greatest integer function)
(B) 3x2 – 4x + 8 log(1 + | x |)
(A) 2 (B) 4
(C) 3x2 + 4x – 8 log(1 + | x |)
(C) 1 (D) 0
(D) 3x2 – 4x – 8 log (1 + | x |)

19. Let f(x) = sin [a] x (where [ ] denotes the 25. Let f : N  N where f (x) = x + (– 1) x –1 then f
is-
greatest integers function). If f is periodic with (A) Inverse of itself
fundamental period , then a belongs to - (B)Even function
(A) [2, 3) (B) {4, 5} (C)Periodic
(C) [4, 5] (D) [4, 5) (D)Identity

16
FU NCTION MATHS-X II-IIT-JEE

26. The function f(x) = log10 cos(2x) exists -  


32. Let f : R  A =  y | 0  y   be a function
(A)For any rational x  2
(B)Only when x is a positive integer such that f (x) = tan–1 (x2 + x + k), where k is a
(C)Only when x is fractional constant. The value of k for which f is an onto
(D)For any integer value of x including zero
function, is -
27. The domain of the function sec–1[x2
– x + 1], is (A) 1 (B) 0
given by– (C) 1/4 (D) None of these
where [·] is greatest integer function -
(A) [0, 1] (B) (–, 0]  [1, ) 33. Which of the following functions are not
1 – 5 1  5  injective mapping-
(C)  ,  (D) None of these
 2 2  (A) f(x) = |x + 1|, x  [–1, )
28. The domain of definition of the function 1
–1
(B) g(x) = x + ; x  (0, )
cot x x
f(x) = , where [x] denotes the
{x 2 – [x 2 ]} (C) h(x) = x2 + 4x – 5 ; x  (0, )
greatest integer less than or equal to x is - (D) k(x) = e–x ; x  [0, )
(A) R 34. Let f be an injective map. with domain {x, y, z}
(B) R – {± n : n  I+ {0}} and range {1, 2, 3}, such that exactly one of the
(C) R – {0}
following statements is correct and the remaining
(D) R – {n : n }
are false : f(x) = 1 , f(y)  1, f(z)  2
29. The domain of the definition of
The value of f –1(A)is -
f(x) = log{(log x)2 – 5 log x + 6} is equal to-
(A) (0, 102) (B) (103, ) (A) x (B) y (C) z (D) None of these
(C) (10 , 10 )
2 3
(D) (0, 102)  (103, ) 35. Let f : R  R and g : R  R be two one-one onto
   functions such that they are mirror image of each
30. If A = x :  x   and other about the line y = 0, then h(x) = f(x) + g(x) is-
 6 3
(A)One-one and onto
f(x) = cos x – x (1 + x) then f(A) is equal to- (B)One-one but not onto
  (C)Not one-one but onto
(A)  ,  (D) Neither one-one nor onto
6 3
   36. Which of the following functions is inverse of
(B)  ,   itself -
 3 6
1 x
1  (A) f (x) = (B) g (x) = 5log x
   3     1 x
(C)   1   ,  1   
2 3  3  2 6  6  (C) h (x) = 2x (x–1) (D) None of these

 1     3     37. Period of f(x) = ecos {x} + sin  [x] is (where, [.] and
(D)   1   ,  1    { } denote the greatest integer function and
 2 3  3  2 6  6  fractional part of function respectively).
31. If A be the set of all triangles and B that of (A) 1 (B) 2 (C) (D) 2
positive real numbers, then the mapping f : A  38. If f(x) = cos (ax) + sin (bx) is periodic, then
B given by f() = area of , ( A) is which of the followings is false -
(A)One-one into mapping (A)A and b both are rational
(B)One-one onto mapping (B) Non-periodic if a is rational but b isirrational
(C)Many-one into mapping (C)Non-periodic if a is irrational but b isrational
(D)Many-one onto mapping (D)None of these

17
MATHS-X II-IIT-JEE FU NCTION
39. The function Based On Domain
f(x) = 2(x – [x]) + sin2 (x – [x]) is -
45. The domain of definition of
(Where [.] denotes greatest integer function)
log2  x  3
(A) Non periodic f(x) = is
x2  3x  2
(B)Periodic with period 1
(A) R–{–1, –2} (B) (–2, )
(C)Periodic with period 2 (C) R–{–1, –2, –3} (D) (3, )
(D) None of these
Based On Even and Odd
 x2 
40. If f: [–20, 20]  R is defined by f(x) =   sin 46. Among the following which is odd function
a  a x  a x
(A)
x + cos x, is an even function, then the set of 2
values of a is- x x
(B) x 
(A) (–, 100) (B) (400, ) e 1 2
(C) (– 400, 400) (D) None of these (C) 1  x  x 2  1  x  x 2
41. Let f be a function satisfying f (x + y) = f (x).f (y) (D) x3  sin x
n
for all x, y  R. If f (A)= 3 then  f (r) is equal 47. Let f be a function satisfying 2f(x) – 3f(1/x) = x2 for
r 1 any x  0, then the value of f(B)is
to - (A) –2 (B) –7/4
3 n 3 7
(A) (3 – 1) (B) n (n + 1) (C)  (D) 4
2 2 8
(C) 3n+1 – 3 (D) None of these 48. If f(x + 2y, x – 2y) = xy, then f(x, y) equals
x 2  y2 x 2  y2
42. If (A) (B)
8 4
f() = ( 2 cos  – 1)( 2 cos 2 – 1)( 2 cos 4 – 1) ..... (2 cos 2 n –1  – 1) x 2  y2 x 2  y2
2 cos 2 n   1 (C) (D)
4 2
2
for n  N and  2m ± , m  I,
3 Based On Periodic Function
then f(/4) = 49. The period of the function
(A) 1 – 2 (B) 2 – 1 1  | sin x | sin x 
f(x) =    is
2  cos x | cos x | 
(C) 2 + 1 (D) None of these (A) (B) 2
 
(C) (D)
43. If f(x) = [x2] – [x]2 where [·] denotes the greatest 2 3
integer function and x  [0, 2], the set of values 50. The function f(x) = cos x is
of f(x) is - (A)Periodic with period 2 
(A) {–1, 0} (B) {–1, 0, 1}
(B)Periodic with period 2
(C) {0} (D) {0, 1, 2}
(C)Periodic with period 42
(D)Not a periodic function
 2x  1 
44. Domain of y   log x  4  log 2 
2 
3 x  51. If f(x) + f(x + a) + f(x + 2a) + … + f(x + na) =
(A) (–4, – 3)  (4, ) constant;  xR and a > 0 and f(x) is periodic,
(B) (–, –3)  (4, ) then period of f(x), is
(C) (–, –4)  (3, ) (A) (n + 1)a (B) e(n+1)a
(D) (–4, – 3)  (3, 4) (C) na (D) ena

18
FU NCTION MATHS-X II-IIT-JEE
52. If f(x) + f(x + 4) = f(x + 2) + f(x + 6) x  R , 58. The entire graphs of the equation
100 y = x2 + kx – x +9 is strictly above the x-axis if
and f(5) = 10, then  f (5  8r) equal to and only if
r 1

(A) 1000 (B) 100 (C) 10000 (D) 10 (A) k < 7 (B) –5 < k < 7
(C) k > –5 (D) –7 < k < 5
Based On Composite
Based On Domain
1 x  2x
53. f  x   log   , g(x) = ,
 1  x  1  x2 59. The domain of the function

3x  x3  x2 
h(x) = , then f(g(x)) + f(h(x)) f(x) = sin 1  log 2  is given by
1  3x2  2 

(A) 0 (B) x (C)5f(x) (D)3f(x) (A) [–2, –1]  [1, 2] (B) [–3, –1]  [1, 3]
(C) (–2, –1)  (1, 2) (D) [–2, –1)  (1, 2]
x
54. If f(x) = 3x, g(x) = , h(x) = f(g(x)) then h(h(..... Based On Range
3
n times)) equal to 1
n n
60. Range of the function f(x) = x2 + 2
, is
(A) x (B) x (C) 3 x (D) 2x x 1
(A) [1, ) (B) [2, )
Based On Inverse (C) [3/2, ) (D) (–)

55. f:[2, )  (– , 4], where f(x) = x(4 – x) then f–1 x2


61. If the function f : R  A given by f (x) =
x2  1
(x) is
is a surjection, then A is
(A)2 – 4  x (B)2 + 4 x
(A)R (B)[0, 1]
(C)– 2 + 4 x (D)– 2 – 4 x (C)(0, 1] (D)[0, 1)
  x2  e 
56. The value of the parameter , for which the 62. If  = sin –1  ln 
2   , then range of  is
  x 1 
function f(x) = 1 + x,  0 is the inverse of itself,
is (A) (0, /2) (B) [0, /2)
(A)– 2 (B)– 1 (C)1 (D) 2 (C) (0, /2] (D) (–/2, /2)

Based On Periodic
Based On General
63. Which of the following functions is periodic ?
57. Complete solution set of the inequality (A) f(x) = x – [x] where [x] denotes the greatest
x 2
x(e – 1)(x + 2)(x – 3)  0 is integer less than or equal to the real number x
(A)[–2, 3] 1
(B) f(x) = sin for x  0, f(0) = 0
(B)(– 2, 0] x
(C)(– , – 2]{0, 3} (C) f(x) = x cos x
(D)(– , 2)[0, 3] (D) f(x) = sin x

19
MATHS-X II-IIT-JEE FU NCTION

Based On Inverse 69. If f(x) = cos( n x) , then


64. Let f N  Y be a function defined as 1 x 
f (x)f (y)  f    f (xy)  has the value
2  y 
f(x) = 4x + 3 where Y = {yN : y = 4x + 3 for
1
some x  N}. its inverse is (A) –1 (B)
2
y3 y3 (C) – 2 (D) 0
(A) g(y) = (B) g(y) =
4 4  x 
70. Solution set of log (   x   0 is
x2 )
3y  4 1 | x | 
(C) g(y) = (D) g(y) =
4 4y  3 (A) (,0)  (1,2)
(B) ( ,1)  (2, )
65. If the function f :[2,  )  [ 1,  ) is defined by (C) ( , – 1)  (0,1)
f(x) = x2 – 4x + 3 is (D) ( , – 2]  (0,1)

(A) 2  x  1 (B) 2  x  1 71. If f(x) = x3 + 3x2 + 12x – 2 sinx, where


f : R R, then
2  x 1 (A) f(x) is many-one and onto
(C) (D) not defined (B) f(x) is one-one and onto
2
(C) f(x) is one-one and into
(D) f(x) is many-one and into
66. Let f:[–/3, 2/3]  [0,4] be a function defined
1 1
as f(x) 3 = sin x – cosx + 2. Then 72. If f(x) f   = f(x) + f    x  R– {0}, where
x x
f –1(x) is given by f(x) be a polynomial function and f(5) = 126 then
f(C)=
1  x  2    x2 
(A) sin   (B) sin–1   (A) 28 (B) 26 (C) 27 (D) 25
 2  6  2  6
x
2 1  x  2  73. If f :[0, )  [0, ) and f(x) = then f is
(C) cos  (D) None of these. 1 x

3  2  (A)One - one and into
(B)Onto but not one - one
2
67. Let f(x) =  x  1  1,  x  1 . Then the set S = (C)One - one and onto
(D)Neither one - one nor onto
{x : f(x) = f –1(x)} is, if f is onto 2
74. If fog = |sin x| and gof = sin x then f(x) and
 3  i 3 3  i 3  g(x) are:
(A) 0,  1, , 
 2 2  (A) f(x) = sin x , g(x) = x2
(B) {0, 1, –1} (C) {0, –1} (B) f(x) = |x|, g(x) = sin x
(D)Empty (C) f(x) = x , g(x) = sin2x

Based On Composite and General (D) f(x) = sin x , g(x) = x2

68. If the function f: [2, )  [1, ) is defined by f


75. If f(x) = sinx + cosx, g(x) = x2 – 1, then g(f(x)) in
(x) = 3x(x –2), then f–1 (x) is
invertible in the Domain
(A) 1 + 1  log3 x     
(A) 0,  (B)   , 
(B) 1 – 1  log3 x  2  4 4
(C) 1 + 1  log3 x   
(C)   ,  (D) [0, ]
(D) does not exist  2 2

20
FU NCTION MATHS-X II-IIT-JEE
76. Let R be the set of real numbers. If f : R  R is a 1 x 3x  x3
function defined by f(x) = x2 then f is 80. Given f(x) = ln and g(x) = . Then
(A) Injective but not subjective
1 x 1  3x2
(B)Subjective but not injective f(g(x)) is equal to
(C)Injective (A)– f(x) (B) 3f(x)
(D)Neither injective nor subjective 2
(C) [f(x)] (D) none of these
81. If f(x) = cos–1x and g(x) = x2, then range of
77 If f(x) =  f(x) and g(x) = f(x) and
f(g(x)) equal to
2
  x 2   x   
F(x) =  f     g     and given that F(5)  
  2    2  (A)  0,  (B) [0, ]
   2
= 5, then F(10) is equal to  
(A) 5 (B) 10 (C) 0 (D) 15 (C)  ,   (D) none of these
2 
2
78. 
If g(f(x)) = sin x and f (g(x))  sin x  , then
 x2  e 
(A) f(x) = sin 2 x, g(x)  x 82. If f(x) = ln  2  , then range of f(x) is
 x 1 
(B) f(x) = sin x, g(x) = |x|
(A) (0, 1) (B) (0, 1]
(C) f(x) = x 2 , g(x)  sin x
(C) [0, 1) (D) {0, 1}
(D) f and g cannot be determined

79. If log3 (x2 – 6x + 11) < 1, then the exhaustive 83. If f(x) is a function that is odd and even
range of values of x is simultaneously, then f(C)– f(B)is equal to
(A)(– , 2)(4, ) (A) 1 (B) – 1 (C) 0 (D) 2
(B)(2, 4)
(C)(– , 1)(1, 3) (4, )
(D)  , 1  1, 3

21
MATHS-X II-IIT-JEE FU NCTION

EXERCISE # 2
One or More Than One Correct Answer 7. Function f(x) = sin x + tan x + sign (x2 – 6x + 10) is
Question
Type Questions (A)Periodic with period 2
(B)Periodic with period 
2 1
1. If f(x) = x  | x | , g(x) = then (C)Non-periodic
9  x2 (D)Periodic with period 4
Df+gcontains –
(A)(–3, –1) (B) [1, 3) 8. In the following functions defined from
(C)[–3, 3] (D) {0}  [1, 3) [–1, 1] to [–1, 1] the functions which are not
injective are:
3x  1
2. If f(x) = and S = {x| f(x) >0} then 2
3x  2x 2  x
3
(A) sin (sin–1x) (B) sin 1 (sin x)
S contains – 
1  (C) (sign x) n ex (D) x3 sign x
(A) (–, –2) (B)  ,5 
3 
9. Let f(x) = tan (loge x), then
1 
(C) (–, –1) (D) (0, ) –    x  f (x)  f (y)
3 (A) f   
 y  1  f (x) f (y)
3. If D is the domain of the function
f (x)  f (y)
 3x  1  (B) f  xy  
f(x) = 1  2x + 3 sin–1   then D contains- 1  f (x) f (y)
 2 
 1 1  1  f  x   f  y  f z   f  x f  yf  z
(A)   ,  (B)  , 0 (C) f  xyz  
1  f  x  f  y   f  yf  z  f z f  x 
 3 2  3 
 1 
(C)  , 1
1 
(D)  , 1
 
(D) f e /4  1
 3  2  10. Let f(x) = cos x + sin x
(A)Domain of f(x) is R
4. Let A = R – {2} and B = R – {1}.
x3 (B)Range of f(x) is [–2, 2]
Let ƒ : A  B be defined by f(x) = then–
x2 (C)Range of f(x) is   2, 2 
(A) f is one-one (B) f is onto
(C) f is injective (D) None of these (D)Period of f(x) is 2
11. Among the following which is / are periodic
sin [x] functions
5. If F(x) = , then F(x) is:
{x} |x|
(A)
(A) Periodic with fundamental period 1 x
(B) Even (B) x  x where {} to decimal part of x
sin
(C) Range is singleton
(C) cos x + tan x
 {x} 
(D) Identical to sign  sgn   1 , where {x}  1
 {x}  (D) max sin x, 
  2
denotes fractional part function and [.]
12. Among the following which is / are true
denotes greatest integer function and sign (x)
(A) If f, g are only two functions then
is asignum function.
fog = gof always
6. Let f: [–1, 1]  [0, 2] be a linear function which (B) f(f(x)) is always defined, where ‘f’ is any
is onto then f(x) is/are. function
(A) 1 – x (B) 1 + x (C) f(g(x)) = x  g = f–1 (x)
(C) x –1 (D) x +2 (D) None

22
FU NCTION MATHS-X II-IIT-JEE
13. If 1 + (x2 – 1) + (x2 – 1)2 + (x2 – 1)3 + .... up to  x  3 x0
1 
terms = then x belongs to 19. Let f(x) =  4 x  0 , then
2  x2  2
x  7 x 0

(A) 0, 2  (A) f(–5) = 5 (B) f(–5) = –5
(C) f(5) = 32 (D) f(–1) + f(A)= 6
(B)   2, 2 
 
20. If f (x)  cos 2 x  cos 2   x   cos x.cos   x 
(C)   2, 
2 – {0}  3  3  
(D) None of these then
(A) f(x) is an even function
14. The value of f(x) = 3 sinx – 4 cosx + 5 lies in the
 
interval (B) f    f  
8 4
(A) [0, 10] (B) (0, 10)
(C) f(x) is a constant function
(C) [–5, 10] (D) none of these
(D) f(x) is not a periodic function
 2 | x | 
15. If f(x) = cos–1  –1
 + [log (3 – x)] , then its 21. Let f(x) = sec–1[1 + cos2x] where [.] denotes the
 4  greatest integer function. Then
domain is (A)The domain of f is R
(A)[– 2, 6] (B)[– 6, 2)  (2, 3) (B)The domain of f is [1, 2]
(C)[– 6, 2] (D)[– 2, 2) (2, 3] (C)The range of f is [1, 2]
(D)The range of f is {sec–11, sec–12}
16. Let f :[0, 2]  [0, ) defined as
22. Let f(x) = 9  x 2  x 2  9 , then
f(x) =  x 2  4 , then the values of ‘x’ for which (A)domain of f(x) is {±3}
f(x) = f 1  x  is (B)f(x) is many one function
(C)Range of f(x) is {0}
(A)  2 (B) 2
(C) [0, 2] (D)none (D) 'f' is an even function

17. Let f(x) = tan x, g(x) = cot x, then 23. f(x) = sinx, g(x) = x(x2 – 1), h(x) = (x2 – 1)x then
(A)f(x) + 2f(2x) + 4f(4x) + 8f(8x) + 16 f(16x) = which of the following is/are periodic functions
g(x) (A) g(f(x)) (B) f(g(x))
(B)f(x) – g(x) = – 2g(2x) (C) h(f(x)) (D) g(h(x))
1  f 2  x
(C)f(x) + g(x) = f (x)  0 24. Let f(x) = [x]2 + [x + 1] – 3 where [x] = the
f (x)
greatest integer  x. Where f : R  R. Then
(D)The least value of 4f (x)  9g(x) is 12 (A) f(x) is a many-one and into function
(B) f(x) = 0 for infinite number of values of x
18. Let g(x) be a function on [–1, 1]. If the area of the (C) f(x) = for only two real values
equilateral triangle with two of itsvertices at (0, 0) (D)None of these
3
and [x, g(x)] is , then the function g(x) is    
4  x   cos2   x 
25. Let f(x) = cos2 x + cos2  3  3 ,
(A) g(x) =  1  x  2

then
(B) g(x)  1  x  2
3   3
(A) f  0  (B) f   
2 6 2
(C) g(x) =  1  x  2

 3  3 3
(C) f    (D) f  1 
(D) g(x)  1  x  2
 17  2 2

23
MATHS-X II-IIT-JEE FU NCTION
1 31. Find the set of values of 'b' such that
26. If f(x) = x2 + 1 and g(x) = , then equation h(x) – b = 0 has exactly 8 real solution
x 1
(A) b  [0, 1/4] (B) b  [0, 1/4)
5
(A)f (g(2)) = 2 (B)f(g(3)) = (C) b  (0, 1/4) (D) None of these
4
32. Which statement is true for I(x) = 0 -
1   1 
(C)g(f(4)) = (D)g  f     9 (A) Two values of x is satisfied for I(x) = 0
16   3  (B) One value of x is satisfied for I(x) = 0 and
27. Which of the following statements are true for that x lie between 1 and 2
(C)One value of x is satisfied for I(x) = 0 and
|x – 1| + |x – 2| + |x – 3| +…….+ |x – 100|
that x lie between 3 and 4
(A) Range of the function is [2500, ) (D) None of these
(B) The function is symmetrical aboutx = 50.5
Passage # 2 (Q.33 to Q.35)
(C) The number of solution of max(|x|, |y|) = 2500 If f(x)= 0 ; if x  Q= 1 ; if x  Q.
with the curve is 1 then answer the following questions-
(D) None
33. f(x) is -
28. Which of the following statements are true for the (A)An even function
function (B)An odd function
 x2  e   x2  e  (C) Neither even nor odd function
f (x)  sin log  2   cos log  2  (D)One-one function
 x 1   x 1 
34. f (f(x)) is-

(A)Range of the function is 1, 641/8 
(A)A constant function
(B)Domain of the function  R (B)An even function
(C)Function is even (C)An odd function
(D)Function is invertible (D)Many one function
29. Which of the following statements is/are correct 35. Domain of g(x) = ln (sgn f(x)) is-
 (A) R
(A) f1  x   sin x  cos x has a period of if
2 (B) set of all rational numbers
(C) set of all irrational number
4  x  4
(D) R+
(B) f 2  x   sin x  cos x, x  0 is a periodic Passage # 3 (Q.36 to Q.38)
function Consider the function
 1
1  sin x 1  sec x   x  [x]  ; if x  
(C)The period of f3  x   is . f(x) =  2
1  cos x 1  cos ecx   0 ; if x  
(D) If derivative of a function is periodic, then where [.] denotes greatest integer function.
the function may be periodic. If g(x) = max. {x2, f(x), | x |} ; –2  x  2, then.
Question Passage Based Type Questions 36. Range of f(x) is-
Passage# 1 (Q.30 to Q.32)  1 1
(A) [0, 1) (B)  , 
Let f(x) = x2 – 3x + 2, g(x) = f(|x|)  2 2
h(x) = |g(x)| and I(x) = |g(x)| – [x]  1 1  1 1
(C)   ,  (D)  , 
Are four function, where [x] is the integral part of  2 2  2 2
real x.
30. Find the value of 'a' such that equation 37. f(x) is-
g(x) – a = 0 has exactly 3 real roots- (A)Non periodic
(A) 2 (B) 1 (B)Periodic with period 1
(C) 0 (D) None of these (C)Periodic with period 2
(D)Periodic with period 1/2

24
FU NCTION MATHS-X II-IIT-JEE
38. The set of values of a, if g(x) = a has three real 45. The value of f(2), if f(A)= k is
and distinct solutions, is - (A) k (B) 2k2 + 1
(A) (0, 1/2) (B) (0, 1/4) (C) 2k2 – 1 (D) –k
(C) (1/4, 1/2) (D) (0, 1)
46. The value of f(C)– 2kf(B)if f(A)= k is
Passage # 4 (Q.39 to Q.41) (A) k (B) –k
Consider the function (C) 2k (D) –2k
2
x  1, 1  x  1 Passage # 7 (Q.47 to Q.49)
f(x) = 
 nx, 1  x  e A function f : A  B is said to be injective if
Let f1(x) = f(|x|) distinct elements in A have distinct images in B.
f2(x) = |f(|x|)| And subjective if f A = B. Then answer
f3(x) = f(–x) thefollowing
Now answer the following questions.
47. If the function f: A  B defined by
39. Number of positive solutions of the equation 1  cos 2x
2f2 (x) –1 = 0 is- f  x  is injective then the set A can be
2
(A) 4 (B) 3 (C)1/2 (D) 1 (A) [0, ] (B) [–, ]
40. Number of integral solution of the equation (C) [–/2, 0] (D) [–, 0]
f1(x) = f2(x) is
48. If the function f: R  B defined by f  x   x 2
(A) 1 (B) 2 (C) 3 (D) 4
is subjective then the set ‘B’ can be
41. If f4(x) = log27 (f3(x) + 2), then range of f4(x) is (A) (–, 0] (B) [0, )
1  (C) (0, ) (D) R
(A) [1, 9] (B)  ,  
3 
49. The function f : RB defined by f(x) = [x] + [–x]
 1 (where [.] is g.i.f) is subjective then B =
(C) 0,  (D) [1, 27]
 3 (A) R (B) [0, 1]
Passage # 5 (Q.42 to Q.43) (C) [–1, 0] (D) {–1, 0}
If notation [x] denotes integer less than or equal Passage # 8 (Q.50 to Q.52)
to x and (.) denotes integer greater than or equal If a function y  f  x  ;f : A  B then the set A
to x, then is called as domain of the function & B is called
42. The solution set of the equation co-domain of the function. For all x  A, the
values of y thus obtained comprise the set ‘C’
(x)2 + [x]2 = [x – 1]2 + (x + 1)2 is -
where C is called as range of function
(A) {x; x  R} (B) {x; x  R – Z}
(C) {x; x  Z} (D) {x; x } 1
50. The domain of the function f  x  
43. Let f(x) = x + (x) ; x < 0,3x – 2(x) ; x  0 ln  cos 1 x 
Range of sgn f(x) is - is where [.] indicates greatest integer function
(A) {–1, 0, 1} (B) {–1, 1} (A) [0, 1]
(C) {1, 0} (D) {–1, 0} (B) [–1, cos2]
(C) [–1, cos3)  (cos3, cos4)
Passage # 6 (Q.44 to Q.46) (D) [–1, cos3)  (cos3, cos2)
Let f(x + y) + f(x – y) = 2f(x) f(y)  x, y  R and 51. The domain of the function
f(0)  0. Then
f  x   cos  sin x   log x x where {.}
44. The function f(x) is indicates fractional part function
(A)Even (A) [1, ) (B)  0, 2  1,  
(B)Odd
(C)Both even & odd  
(C)  0,   1 (D) (0, 1)
(D)Neither even nor odd  2

25
MATHS-X II-IIT-JEE FU NCTION
52. The range of the function of 54. The function f(x) is
(A)Even
f  x   sin 1 x 2  x  1 is
(B)Odd
    (C)Neither even nor odd
(A) 0,  (B) 0, 
 2  3 (D)Periodic as well as odd
   
(C)  ,  (D)  ,  55. The function f : R  R , then function will be
3 2 6 2
(A)One one onto
Passage # 9 (Q.53 to Q.55) (B)One one into
Mr. X is a teacher of mathematics. His students (C)Many one onto
want to know the ages of his son's S1 and S2. He (D)Many one into
told that their ages are 'a' and 'b' respectively such
that f(x + y) – axy = f(x) + by2  x, y  R after
some time students said that information is
insufficient, please give more information.
Teacher says that f(A)= 8 and f(B)= 32.
53. The age of S1& S2 will be respectively
(A) 4, 16 (B) 8, 16
(C) 16, 8 (D) 32, 8

26
FU NCTION MATHS-X II-IIT-JEE

EXERCISE # 3
Question Column Match Type Questions 4. Column I Column II
(A) The period of the function (P)1/2
Match the entry in Column 1 with the entry
y = sin (2t + /3)
inColumn 2. + 2 sin (3t + /4)
1. Column I Column II + 3 sin 5t
(A) Range of (P){1, 2, 3} (B) y = {sin (x)} is a many one (Q) 8
[sin 2x]  [cos 2x] Function for x (0, a) where
(B) Domain of (Q){1} {} denotes fractional part of
2
x and a may be
x  4x
C2x 2 3 (C) The Fundamental period of (R) 2
The function
(C) Range of (R){0, 1}
1  | sin( / 4)x | sin(  / 4)x 
log(cos(sin x)) y   
2  cos( / 4)x | cos(  / 4)x | 
(D) Range of (S){0} (D) If f : [0, 2] [0, 2] is injective (S) 0
[|sinx| + |cos x|] Function defined by f(x)
[where [ ] denotes G.I.F] = ax2 + bx + c, where a, b, c
Are non - zero realnumbers,
2. Column I Column II Then f(B)is equal to
(A) Period of (P) 2 5. The domain of the function
1 | sin x | | cos x |  Column-I
  
2  cos x sin x 
1  3  2x 
(B) Range of (Q) 2 (A) 3  x  sin  
 5 
|x|
cos1 log[x] (B) log10 (1  log10 (x 2  5x  16))
x
(C) Total number (R) 1
of solution 1  2 
(C) cos  
x2 – 4– [x] = 0  2  sin x 

(D) Period of (S) (D) sin x  16  x 2
2
4 2 Column-II
ecos x  x [x] cos x
(P)  (2k,(2k  1))
[where [ ] denotes G.I.F] kI
(Q) [ 4,  ]  [0, ]
3. Column I Column II (R) (2, 3)
(A) Domain of f(x) = 2 x  3x (P) [0, 1] (S) [–1, 3]
+ log3log1/2x is
6. In this question each entry in column 1 is
(B) Solution set of equation (Q) [0, ) related to exactly one entry in column 2. Write
2 2 2 2 the correct letter from column 2 against the
2cos x/2.sin x = x + 1/x is
(C) If A = {(x, y) ; y = 1/x, x R0} & (R) [1, ) entry number in column 1. Let the function
defined in column 1 have domain
B = {(x, y); y = –x, x R}
  / 2,  / 2  and co-domain  ,  .
then A B is
Column I Column II
(D) The functions f(x) = x x  1 (S)  (i) 1 + 2x (A)Onto but not one-one
(B)One-onebut not onto
&(x) = x 2  x are identical
(ii) tan x (C)One-one and onto
in
(D)Neither one-one noronto

27
MATHS-X II-IIT-JEE FU NCTION
7. Match the following with their least periods 13. Let f: R  R such that f(x – f(y)) = f(f(y)) + x
Column I Column II f(y) + f(x) – 1x, yR. Then find the value of
2 |(f(16))| – 125.
(A) sin 3x (P)
3 14. Let f be a function from the set of positive
(B) 2x – [2x] (where is [.] gif) (Q)  integers to the set of real number such that
x (i) f(A)= 1
(C) tanx + tan (R) 3
3 n
(D) cos (sin x) (S) 1/2 (ii)  rf (r) = n (n + 1) f(n),n  2 then find
r 1
8. Column-I Column-II the value of 2126 f(1063).
(A) The number of values of (P) 0
x
‘x’ satisfying e lnx = 1 is/are 15. Let f(x) be a function such that
(B) The number of real (Q) 1 f(x–1) + f(x + 1) = 2 f(x) R. If f(B)= 7
solution of the equation 17

x2log x (x 3)
 16 is then find the value of  [f (2  8r)  7] .
r 0
(C) The number of roots of the (R) 2
  1 1  2 1
equation x  2 tan x  in 16. If f  x   = x3 + 3 – 4  x  2  + 13,
2  x  x  x 
the interval [0, 2] is
then find the value of f (2  3) .
(D) The period of the function (S) 3
sin3t + sin4t is 17. If f3 (x) – 3f2 (x) + 3f(x) – 1 = x6, then find the
9. Column-I Column-II value of f(0).
(A) The period of the function (P) 1 18. If f(x), g(x) are any two real valued functions
x  such that |f(x) + g(x)|  |f(x)| + |g(x)| and g(x) 
sin log   (where {x} denotes
3 100

the fractional part of ‘x’) is


0, f(x) g(x)  0 then find the value of  f (r) .
r 1
(B) The total number of solutions o (Q) 4
the equation
x 4  x2  1
loge x  sin x in   / 2,  / 2 is 19. If f(x) = , then find the value of
x2  x  1
(C) The least value of the function (R) 5 f(n) where ‘’ is the non real root of the
f  x   sec2 x  cosec2 x is equation
(D) Let f(x) = | sin x | + | cos x | then (S) 3 z3 = 1 and ‘n’ is a multiple of ‘3’.
Number of elements in range set
1 x 
Of [f(x)] where ([] is G.I.F is) 20. If f2(x) f  3
 . = x [x  –1, 1 & f(x)  0]
(T) Infinite  1  x 
Question Numerical Type Questions then find |[f(–2)]| (where [.] is the g.i.f).
10. If f(x) = (1005 – x10)1/10, then find the value of 8
f(–2) =  [f(–2)] = 2  |[f(–2)]| = 2.
1 3
f(f(1024)).
210 21. If ‘f’ is polynomial such that
11. If f(x + f(y)) = f(x) + y  x, yR and f(0) = 1, 1 x  1 x   1 x  1  x 
f  .f   = f f 
f (10) 1 x   1 x  1 x   1 x 
then find the value of . f(x) is not a
5 where (x  0, ± 1) and f(C)= 28, then find the
constant function.
1  10 
12. An odd function is symmetric about the vertical value of   (f (n)  1)  .
 605  n 1 
line x = a (a > 0) and if [f (1  4r)]r = 8, then
r 0
find the value of 8f(1).

28
FU NCTION MATHS-X II-IIT-JEE

EXERCISE # 4
Question Previous Year (JEE Main) 8. Let f : (–1, 1)  B, be a function defined by

1. Which one is not periodic ? [AIEEE-2002]  2x 


f(x) = tan-1  2 
,then f is both one-one and
 1 x 
(A) sin 3x  sin 2 x (B) cos x  cos2 x
onto when B is the interval [AIEEE-2005]
(C) cos4x  tan2 x (D) cos2x  sin x
   
2 (A) 0,  (B)  0, 
2. The period of the sin  is [AIEEE-2002]  2  2
(A) (B)
     
(C)+3 (D)/2 (C)   ,  (D)   , 
 2 2  2 2
3. 
The function f(x) = log x  x 2  1 is  9. Let ƒ: N  Y be a function defined as ƒ(x) = 4x
(A) An odd function [AIEEE-2003] + 3 whereY = {y  N : y = 4x + 3 for some x 
(B) A periodic function N}. show that f is invertible and its inverse is
(C) Neither an even nor an odd function
[AIEEE-2008]
(D) an even function
y3 3y  4
(A) g(y) = (B) g(y) =
4. A function f from the set of natural numbers to 4 3
integers defined by [AIEEE-2003] y3 y3
n 1 (C) g(y) = 4 + (D) g(y) =
 2 , when n is odd 4 4
f(n) =  is
  n , when n is even 10. Let f (x) = (x + 1)2 – 1, x  –1.[AIEEE-2009]
 2 Statement-1 The set {x : f(x) = f-1(x)} = {0, –1}.
(A) Onto but not one-one Statement-2 : f is bisection.
(B) one-one and onto both
(A) Statement-1 is true, Statement-2 is false.
(C) Neither one-one nor onto
(D) one-one but not onto (B) Statement-1 is false, Statement-2 is true.
(C) Statement-1 is true, Statement-2 is true;
5. If f : R  R satisfies f (x + y) = f(x) + f(y), for Statement-2 is a correct explanation for
n
Statement-1
all x, y  R and f (A)= 7, then  f (r) is (D) Statement-1 is true, Statement-2 is true;
r 1
[AIEEE-2003] Statement-2 is not a correct explanation for
7  n  1 Statement-1
(A) (B) 7n  n  1
2 11. For real x, let f(x) = x3 + 5x + 1, then
7n  n  1 7n
(C) (D) [AIEEE-2009]
2 2 (A)f is onto R but not one-one
(B)f is one-one and onto R
6. The range of the function F(x) = 7  x Px  3 is
(C)f is neither one-one nor onto R
[AIEEE-2004]
(D)f is one-one but not onto R
(A) {1, 2, 3, 4} (B) {1, 2, 3, 4, 5, 6}
(C) {1, 2, 3} (D) {1, 2, 3, 4, 5} 1
12. The domain of the function f(x) = is
1x x
7. If f : R  S, defined by f(x) = sinx –
3cos x  1 , is onto, then the interval of S is [AIEEE-2011]
[AIEEE-2004] (A)  ,0  (B)  ,    {0}
(A) [0, 1] (B) [–1, 1]
(C)  ,  (D)  0,  
(C) [0, 3] (D) [–1, 3]

29
MATHS-X II-IIT-JEE FU NCTION

 1 1 18. Let f (x) = ax (a > 0) be written as


13. The function f : R    ,  defined as f (x) = f1(x) + f2(x), where f1(x) is an even
 2 2
function of f2(x) is an odd function. Then f1(x +
x y) + f1(x – y) equals [April 2019]
f(x) = , is [JEE Mains-2017]
1  x2 (A) 2f1(x) f1(y)
(A) Injective but not subjective (B) 2f1(x) f2(y)
(B)Subjective but not injective (C) 2f1(x + y) f2(x – y)
(C) Neither injective nor subjective (D)2f1(x + y) f1 (x – y)
(D) Invertible
19 If the function f : R – {1, –1}  A defined by
14. For x  R – {0, 1}, let f1(x) = 1/x, f2 (x) = 1 – x x2
f (x) = , is subjective, then A is equal to
1 1 x2
and f3 (x) = be three given functions. If a [April 2019]
1 x
function, J(x) satisfies (f2oJof1) (x) = f3(x) then (A) R – [–1, 0) (B) R – (–1, 0)
J(x) is equal to :- [January 2019] (C) R – {–1} (D) [0, )
(A) f3 (x) (B) f1 (x)
20. The domain of the definition of the function
(C) f2 (x) (D) (1/x) f3 (x)
1
f (x) = 2
+ log10 (x3 – x) is –[April 2019]
4x
15. Let A = {x R : x is not a positive integer}
(A) (1, 2)  (2, )
2x
Define a function f : A  R as f (x) = (B) (–1, 0) (1, 2)  (3, )
x 1
(C) (–1, 0)  (1, 2)  (2, )
thenf is [January 2019]
(D) (–2, –1) (–1, 0) (2, )
(A) injective but not subjective.
(B) not injective. 21. Let f (x) = x2, x  R. For any A R, define
(C)subjective but not injective.
g (A) = {x  R, f (x)  A}. If S = [0, 4], then
(D) neither injective nor subjective
which one of the following statements is not true ?
[April-2019]
16. Let N be the set of natural numbers and two
(A) f (g (S))  f (S) (B) f (g (S)) = S
functions f and g be defined as f,g : N  N
(C) g (f (S)) = g (S) (D) g (f (S))  S
n 1
 2 , if n is odd 22. Let f (x) = loge (sin x), (0 < x <) and g (x) =
 sin–1 (e–x), (x  0). If  is a positive real number
such that : f (n)  
n such that a = (fog)' () and b = (fog) (), then :
 , if n is even [April-2019]
2
2
and g (n) = n – (–1)n. The fog is : (A) a – b – a = 0
(A) Both one-one and onto. (B) a2 + b – a = –22
(B) One-one but not onto. (C) a2 + b + a = 0
(C) Neither one-one nor onto. (D) a2 – b – a = 1
(D) onto but not one-one.
23. For x (3/2), let f (x) = x , g (x) = tan x and
2
1 x   2x  1 x
17. If f (x) = loge  , | x | < 1, then f  2
h (x) = . If  (x) = ((hof) og)(x), then
1 x  1 x  1  x2
is equal to : [April 2019]  = /3 is equal to : [April-2019]
(A) 2f (x) (B) 2f (x2) (A) tan (/12) (B) tan (7/12)
(C) (f (x))2 (D) –2f (x) (C) tan (11/12) (D) tan (5/12)

30
FU NCTION MATHS-X II-IIT-JEE
24. For x  R, let [x] denote the greatest integer  k (x  )2  1, x  
x, then the sum of the series 30. If the function f (x) =  1
k 2 cos x, x
 1  1 1   1 2 
  3     3  100     3  100  is twice differentiable, then the ordered pair
      (k1, k2) is equal to : [4th September 2020]
 1 99  (A) (1/2, 1) (B) (1, 1)
.....     is – [April-2019] (C) (1/2, –1) (D) (1, 0)
 3 100 
(A) –153 (B) –133 (C) –131 (D) –135
31. Let f : R  R be a function which satisfies
2x 2x f (x + y) = f (x) + f (y)  x,y R. If f (A) = 2 and g
8 8
25. The inverse function of f (x)  , (n 1)
82x  82x
x (–1, 1), is [Jan. 2020]
(n) =  f (k), n  N then the value of n, for
k 1
1 1 x  1 1 x  which g(n) = 20, is : [September 2020]
(A) log8   (B) log8   (A) 5 (B) 9 (C) 20 (D) 4
4 1 x  2 1 x 
1 1 x  1 1 x 
(C) log8   (D) log8   32. If f (x + y) = f (x) f (y) and
4 1 x  2 1 x  

   f (x)  2, x, y  N ,where N is the set of all


26 If f '(x) = tan–1(sec x + tan x),   x  , and x 1
2 2 f (4)
f (0) = 0, then f (A) is equal to : natural numbers, then the value of is –
f (2)
 1 2 [September 2020]
(A) (B)
4 4 (A) 1/9 (B) 4/9 (C) 1/3 (D) 2/3
 1 1
(C) (D)
4 4 33. For a suitably chosen real constant a, let a
function, f : R – {–a}  R be defined by f (x) =
27 If g (x) = x2 + x – 1 and gof (x) = 4x2– 10x + 5, ax
then find f (5/4). . Further suppose that for any real number
(A) 1/2 (B) –1/2 ax
(C) –1/3 (D) 1/3 x  –a and f (x)  –a, (fof) (x) = x. Then f (–1/2)
is equal to :
x [x] [September 2020]
28 Let f (x) = : (1,3)  R then range of f (x)
x2 1 (A) 1/3 (B) 3 (C) –3 (D) –1/3
is (where [.] denotes greatest integer function) 34. The domain of the function
 1   3 7  2 1   3 4  | x | 5 
(A)  0,    ,  (B)  ,    ,  f (x) = sin–1  2 
 2   5 5  5 2   5 5  x 1 
is (–, –a] [a, ). Then a is equal to :
 2   4  1   2 4 [September 2020]
(C)  ,1  1,  (D)  0,    , 
 5   5  3  5 5 1  17 17  1
(A) (B)
2 2
29. Let [t] denote the greatest integer  t. Then the 17 17
(C) 1 (D)
equation in x, [x]2 + 2 [x + 2] – 7 = 0 has : 2 2

[September 2020] 35. Suppose that a function f : R  R satisfies


(A) no integral solution f (x + y) = f (x) f (y) for all x, y  R and
n
(B) exactly four integral solutions f (A) = 3. If  f (i)  363 ,then n is equal to
(C) exactly two solutions i 1

(D) infinitely many solutions ______. [September 2020]

31
MATHS-X II-IIT-JEE FU NCTION
36. Let A = {a, b, c} and B = {1, 2, 3, 4}. Then the x
number of elements in the set C = {f : A  B | 43. Let f (x) = , x  – 1, then for what value
x1
2  f (A) and f is not one-one} is _______. of  f {f (x)} = x. [IIT Scr. 2001]
[September 2020]
1 x  (A) 2 (B) – 2
37. The domain of the function cosec–1   (C) 1 (D) –1
 x 
[Aug. 2021] log 2 ( x  3)
44. The domain of definition off (x) =
 1   1  x 2  3x  2
(A)   , 0   [1,  ) (B)   ,    {0}
 2   2  is– [IIT Scr. 2001]
 1  1  (A) R – {–2, + 2} (B) (– 2, )
(C)  1,    (0, ) (D)   ,    {0}
 2   2  (C) R– {–1, –2, –3} (D) (–3, ) / {–1, –2}
38. The domain of the function
2 1
1  3x x 1  1  x  1  45. If f : [1, )  [2, ) is given byf (x) = x +
f (x)  sin    cos   x
 (x  1)2   x 1 
 
then f–1 (x) equals - [IIT Scr. 2001]
is – [JEE MAIN 2021]
(A) [0, 1/4] (B) [0, 1/2] x  x2  4 x
(A) (B)
(C) [1/4, 1/2] {0} 2 1  x2
(D) [–2, 0] [1/4, 1/2] x  x2  4
(C) (D) 1 + x2  4
39. Let f : N  N be a function such that f (m + n) = f 2
(m) + f (n) for every m, n  N. If f (6) = 18, then f
(2). f (3) is equal to [JEE MAIN 2021]
R|1, x0
Let g (x) = 1 + x – [x] and f (x) = S0, x0
40.
(A) 54 (B) 18 (C) 6
The range of the function
(D) 36 46.
|T1, x0
  3    Then for all x, f (g(x) is equal to
f (x)  log 5  3  cos   x   cos   x 
  4   4  [IIT Scr. 2001]
(A) x (B) f (x)
   3 
 cos   x   cos   x  is – (C) 1 (D) g (x)
4   4 
[JEE MAIN 2021]
(A) [0, 2] (B) [–2, 2]
47. Suppose f(x) = (x + 1)2 for x  – 1. If g(x) is the
function whose graph is the reflection of the
(C) (0, 5) (D) [1/ 5, 5]
graph of f(x) with respect to the line y = x, then
g(x) equals– [IIT Scr. 2002]
Question Previous Year (JEE Advanced) 1
41. Let f (x) = (1 + b2)x2 + 2bx + 1 and let m(B) be (A) – x – 1, x  0 (B) ,x>–1
(x  1)2
the minimum value of f(x). As b varies, the
range of m(B) is - [IIT 2001] (C) x  1 , x  – 1 (D) x – 1, x  0
(A) [0, 1] (B) [0, 1/2]
(C) [1/2, 1] (D) (0, 1] 48. Let function f : R  R be defined by f(x) = 2x
+sin x for x  R. Then f is–[IIT Scr. 2002]
42. Let E = {1, 2, 3, 4} and F = {1, 2}. Then
(A) one to one and onto
thenumber of onto functions from E to F is-
(B) one to one but not onto
[IIT 2001]
(C)Onto but not one to one
(A) 14 (B) 16
(C) 12 (D) 8 (D)Neither one to one nor onto

32
FU NCTION MATHS-X II-IIT-JEE
x (C)one-one but not onto
49. Let f(x) = defined as [0, )  [0, ),
1 x (D)onto but not one-one
f(x) is– [IIT Scr.2003]
(A) one- one & onto 55. If X and Y are two non-empty sets wheref :
(B) one–one but not onto X  Y is function is defined such that
(C)Not one–one but onto f(C) = {f(x): x  C} for C  X
(D)Neither one–one nor onto and f–1(D) = {x : f(x)  D} for D  Y
for any A  Y and B  Y then-
tan 2 
50. If f(x) = x2  x + ,  (0, /2), x > 0 [IIT 2005]
x2  x
(A)f (f(A)) = A
–1

then value of f(x) is greater than or equal to–


(B)f–1(f(A)) = A only if f(X) = Y
[IIT Scr.2003]
(C)f(f–1(B)) = B only if B  f(x)
(A) 2 (B) 2 tan 
(D)f(f–1(B)) = B
(C) 5/2 (D) sec 

x2  x  2 56. Find the range of values of t for which 2 sin


51. Find the range of f(x) = is –
x2  x 1
1  2x  5x2   
[IIT Scr.2003] t= 2
; t   ,  [IIT 2005]
3x  2x  1  2 2
 11 
(A) (1, ) (B) 1,  3  3 
 7 (A) t (B) t
10 2 10 2
 7  7
(C) 1,  (D) 1,   
 3  5 (C) t (D) None of these
2 10

52. Domain of f(x) = sin 1 (2x)   / 6 is–


57. For any real number x, let |x| denote the largest
[IIT Scr.2003] integer less than or equal to x, Let f be a
 1 1  1 1 realvalued function defined on the interval [–
(A)  , (B)  ,
 4 2   2 2 
 x  [x], if [x] is odd
10, 10] by f (x)  
 1 1  1 1 1  [x]  x, if [x] is even
(C)  ,  (D)  , 
 4 4  2 4 10
2
f (x) cos x dx is___
10 10
Then the value of
53. Let f(x) = sin x + cos x and g(x) = x2 – 1, then
g (f(x)) will be invertible for the domain- [IIT 2010]
[IIT Scr.2004]
  
(A) x [0, ] (B) x   – ,  58. Let f(x) = x2 and g(x) = sin x for all x R. Then
 4 4
the set of all x satisfying (fogogof)(x), where
    
(C) x  0,  (D) x   – ,0  (fog)(x) = f(g(x)), is [IIT 2011]
 2  2 
(A) , n {0, 1,2,......}
 x, x  Q 0 x Q
54. f (x)   ; g (x)   (B) , n {1, 2,.....}
 0, x  Q x x Q
(C) , n {..., –2,–1,0,1,2...}
then (f – g) is [IIT Scr.2005]
(A)one-one, onto (D) 2np, n{..., –2,–1,0, 1, 2,...}
(B)neither one-one, nor onto

33
MATHS-X II-IIT-JEE FU NCTION
bx 63. Let X be a set with exactly 5 elements and Y be
59. Let ƒ : (0, 1)  R be defined by f(x) = ,
1  bx aset with exactly 7 elements. If  is the number
where b is a constant such that 0 < b < 1. Then of one-one functions from X to Y and is the
[IIT 2011]
number of onto function form Y to X, then the
(A) f is not invertible on (0,1)
1 1
(B) f  f –1 on (0, 1) and f’(B) = value of (  ) is ______ .
f '(0) 5!
1 [IIT-Advance-2018]
(C) f = f –1 on (0, 1) and f’ (B) =
f '(0)  x 
(D) f –1 is differentiable on (0,1) 64. Let E1 = x   : x  1and  0 and
 x  1 
60. The function f : [0, 3] [1, 29] defined by    x  
E 2  x  E1 : sin 1  log e    is a real number 
f(x) = 2x3 – 15x2 + 36x + 1, is   x 1
 
[IIT-Advance-2012] (Here, the inverse trigonometric function sin-1x
(A) one- one and onto   
(B) Onto but not one-one assumes values in   ,  .)
 2 2
(C) one-one but not onto Let f: E1  R be the function defined by
(D) Neither one-one nor onto  x 
f(x) = loge   and g : E2  R be the
61. Let ƒ: (–1, 1)  R be such that f (cos 4)  x 1
       x 
=
2
for    0,    ,  . Then function defined by g(x) = sin-1  log e  
2
2  sec   4  4 2   x 1
COLUMN - I COLUMN - II
1
the values of ƒ   is (are)[IIT-Advance-2012] P. The range of ƒ 1.  1   e
3 
  ,    e  1 ,  
is  1  e
3 3
(A) 1  (B) 1  Q. The range of g 2. (0, 1)
2 2
contains
2 2
(C) 1  (D) 1 
3 3 R. The domain of 3.  1 1
fcontains  2 , 2 
 
  
62. Let f(x) = sin  sin  sin x   for all x  R
6 2  S. The domain of 4.  ,0    l, 
 g is
and g(x) = sin x for all x  R, Let (fog)(x)
2 5.  e 
 ,
denote f(g(x)) and (gof)(x) denote g(f(x)). Then  e  1
which of the following is (are) true ?
6. 1 e 
[IIT-Advance-2015]  ,0    , 
 2 e  1
 1 1
(A) Range of f is   , 
 2 2 The correct options is : [IIT-Advance-2018]

1 1 (A)P  4;  Q  2; R  1;S  1
(B) Range of fog is  , 
 2 2 (B)P  3; Q  3; R  6; S  5
f (x)  (C)P  4; Q  2; R  1; S  6
(C) lim 
x  0 g(x) 6 (D)P  4; Q  3; R  6; S  5
(D) There is an x  R such that (gof)(x) = 1

34
FU NCTION MATHS-X II-IIT-JEE

ANSWER KEY
EXERCISE-1

Qus. 1 2 3 4 5 6 7 8 9 10 11 12 13 14 15 16 17 18 19 20
Ans. A D C D A C A D C,D D B A B C A A B A C A
Qus. 21 22 23 24 25 26 27 28 29 30 31 32 33 34 35 36 37 38 39 40
Ans. A D C A A D B B D C D C B B D A A D B B
Qus. 41 42 43 44 45 46 47 48 49 50 51 52 53 54 55 56 57 58 59 60
Ans. A B D A D D B A B D A A D B B B C B A A
Qus. 61 62 63 64 65 66 67 68 69 70 71 72 73 74 75 76 77 78 79 80
Ans. D C A B B C C A D D B A A C B D A A B B
Qus. 81 82 83
Ans. A B C

EXERCISE-2
Qus. 1 2 3 4 5 6 7 8 9 10 11 12 13 14 15
Ans. A,B,D A,B,C,D A,B A,B,C A,B,C,D A,B A B,C,D A,B,C,D A,C,D B,D D C A,C B
Qus. 16 17 18 19 20 21 22 23 24 25 26 27 28 29 30
Ans. B,C A,B,C,D A C,D A,B,C A,D A,B,C,D A,C A,B A,B,C,D A,B,C,D A,B A,B,C C,D A
Qus. 31 32 33 34 35 36 37 38 39 40 41 42 43 44 45
Ans. B C A A,B,D C C B C C D C B A A C
Qus. 46 47 48 49 50 51 52 53 54 55
Ans. B C B D B D C C A D
EXERCISE-3

[MATCH THE COLUMN]


1. [A] [R] [B] [P] [C] [S], [D] [Q] 2. [A] [Q] [B] [S] [C] [P], [D] [R]
3. [A] [S] [B] [S] [C] [S], [D] [R] 4. [A] [R] [B] [R] [C] [Q], [D] [S]
5. [A] [S] [B] [R] [C] [P], [D] [Q] 6. [A] [Q] [B] [R]
7. [A] [P] [B] [S] [C] [R], [D] [Q] 8. [A] [Q] [B] [P] [C] [S], [D] [R]
[NUMERICAL]
10.[1] 11.[2] 12.[7] 13.[2] 14.[2] 15.[0] 16.[9] 17.[1] 18.[0] 19.[3] 20.[2] 21.[5]

EXERCISE-4
Qus. 1 2 3 4 5 6 7 8 9 10 11 12 13 14 15 16 17 18 19 20
Ans. B A A B C C D C A A B A B A A D A A A C
Qus. 21 22 23 24 25 26 27 28 29 30 31 32 33 34 35 36 37 38 39 40
Ans. C D C B 1 3 2 2 4 1 1 2 2 1 5 19 B C A A
Qus. 41 42 43 44 45 46 47 48 49 50 51 52 53 54 55 56 57 58 59 60
Ans. D A D D A C D A A B C A B A C A 4 A A B
Qus. 61 62 63 64
Ans. AB ABC 119 A

35
MATHS-X II-IIT-JEE INVERSE TRIGONOMETRIC FUNCTION

CHAPTER

INVERSE TRIGONOMETRIC FUNCTION


1. DEFINITION line y  x from x   / 2 to x   / 2 graph of

 If f : A B is one to one and onto function and g is y  sin 1x


a rule under which for every element Y  B, There y y = sin x
–1

/2
exists an unique element X  A then g : B  A is
called Inverse function of f : A  B . –1
x
0 1
1
i.e g  f
–/2
x  g(y)  x  f 1 (y)
So y  f (x) & x  g(y) (ii) If cos y = x then y = cos–1 x
such that f{g(y)}  y & x  g(f (x)) then f & g are where 1  x  1&0  y   graph of
said to be inverse function of each other. y  cos 1 x
1 1
(i) f (x)  [f (x)] y –1
 y =cos x
1
(ii) arc sin x  sin x
(iii) Arc sin x  2n   arcsin x,n  I /2

(iv) Only one-one onto function has an inverse


x
function. –1 0 1
S. FUNCTION DOMAIN RANGE
N.
1 sin x xR y  [–1, 1] (iii) If tan y = x then y = tan–1 x
2 cos x xR y  [–1, 1] where x  R &   / 2  y   / 2)
3 tan x x  R–(2n+1)/2; yR
nI Graph of y  tan 1 x
4 cot x x  R–n; yR
y y = tan–1 x
nI
/2
5 cosec x x  R–n; y  (–, –1] 
nI [1, )
x
6 sec x x  R–(2n+1)/2; y  (–, –1]  0
nI [1, )
– /2

2. DOMAIN, RANGE AND GRAPH OF INVERSE (iv) If cot y = x then y = cot–1 x


TRIGONOMETRIC FUNCTIONS where x  R &0  y   graph of y = cot–1 x
1
(i) If sin y  x then y  sin x y y =cot–1x

where 1  x  1&   / 2  y   / 2
Note: /2
for the graph of y = sin–1 x take the plane mirror
image of the portion of the graph of y  sin x on the x
0

(v) If sec y  x then y  sec1 x

36
INVERSE TRIGONOMETRIC FUNCTION MATHS-X II-IIT-JEE
where x  1&0  y  , y   / 2 Similarly range of sin 1 x is ( /2,  /2)
graph of y  sec1 x 1
y  sin 1    5/6
 2
/2
3. PROPERTIES OF INVERSE TRIGONOMETRIC
FUNCTIONS
–1 0 1 x Property I:
(i) sin 1 (  x)   sin 1 x, x   1,1
–/2
(ii) tan 1 ( x)   tan 1 x  x  R
–1
(vi) cosec y = x then y = cosec x
(iii) cosec 1 ( x)   cosec 1 x  x  ( , 1]  [1, )
where x  1 &   / 2  y   / 2, y  0
graph of y = cosec–1x (iv) cos 1 ( x)    cos 1 x  x   1,1
y
 (v) cot 1 (  x)    cot 1 x  x  R
(vi) sec1 (  x)    sec1 x  x  (, 1]  [1, )
/2
 
Example-02

x  3
–1 0 1 The principal value of sin 1    is
 2 
S. FUNCTION DOMAIN RANGE(PRINCIPA 2  4 5
(A)  (B)  (C) (D)
N. L VALUE) 3 3 3 3
1 1  x  1  / 2  y   / 2 Solution: (B)
y  sin 1 x
 3 3
2 1  x  1 0 y Let sin 1       sin   
y  cos 1 x 2 2
 
3 y  tan 1 x x R  / 2  y   / 2
    3 
and   sin      ,sin 1     
4 y  cot 1 x x R 0 y 2 2 3  2  3

5 y  cosec1 x x 1  / 2  y   / 2, y  0 Example-03
  1 
6 The value of sin cos1     is
1
y  sec x x 1 0  y  , y   / 2   2 
1 3 1
(A) (B) 1 (C) (D)
Example-01 2 2 2
Explain the Following: Solution: (C)
(i) cos π / 4 = 1/ 2 &cos(-π / 4) = 1/ 2   1   1
sin cos1     sin    cos1 
  2   2
But cos 1 (1 / 2) = π / 4 & cos 1 (1 / 2)   π / 4
(ii) sin π / 6 = 1 / 2&sin 5 π / 6 = 1 / 2 ( cos1 (  x)    cos 1 x)
   3
But sin 11 / 2 = π / 6 & sin 11 / 2  5π / 6  sin      sin 
 3 3 2
Solution: (ii)
Range of cos 1 x is [0, ] Example-04

1   The principal value of cot -1 (-1) is


1 
 cos  4 (A)  / 4 (B)  / 4
 2
(C) 3 / 4 (D) 5 / 4

37
MATHS-X II-IIT-JEE INVERSE TRIGONOMETRIC FUNCTION
Solution: (C) (ii) cos 1 (cos x) 
1
Let cot ( 1)    cot   1 and 0    
3
   ,  cot 1 (1) 
3
.
2n2nx,x,(2n2n1)xx(2n 2n 1),n, n I I
4 4
Hence (C) is the correct answer.
Example-05
  3 

The value of sin  tan 1  3 + cos1   
  is
  2  
(A) 1 (B) -1 (C) 0 (D) 1/2 Period of y  cos1 (cos x) is 2
Solution: (A)
(iii) tan 1 (tan x)  n  x
  3 
 
sin  tan 1  3  cos1    
where n 
 
 x  n  n  I
  2   2 2
 3
 sin   tan 1 3    cos 1  graph of y  tan 1 (tan x)
 2 
 tan 1 (  x)   tan 1 x andcos 1 (  x)    cos1 x 
 
    
 sin       sin  1
 3 6 2 
Hence (A) is the correct answer.
Property II:
1
(i) sin 1    cosec 1 x  x  ( , 1]  [1, ) Period of tan 1 (tan x) is 
x
1 (iv) cot 1 (cot x)   n  x,x  (n,(n  1) ),n  I graph
(ii) cos 1    sec 1 x  x  ( , 1]  [1, )
x
of y  cot 1 (cot x)
–1
(iii) tan (1/x) =  cot 1 x, x  0
  cot  1x, x  0

Property III:
(i) sin 1 (sin x) =
  
 2n  x, 2n  2  x  2n  2 ,n  I
  
(2n  1)  x, (2n  1)    x  (2n  1)   ,n  I
 2 2
Graph of y  sin 1 (sin x) Period of cot 1 cot x is 

(v) The curve for y  sec 1 (sec x)

Period of sin 1 (sin x) is 2

Period of sec1 (sec x) is 2 

38
INVERSE TRIGONOMETRIC FUNCTION MATHS-X II-IIT-JEE

(vi) cosec 1 (cosec x)  Example-08


  3π  
    The value of tan 1  tan    is
 2n  2 , 2n  2  x  2n  2 , n  I   4 
  
(2n  1)   x, (2n  1)   x  (2n  1)  , x  n, n  I    
 2 2 (A) (B)  (C) (D) 
3 3 4 4
graph of y  cosec1 (cosec x) Solution: (D)
  3      
tan 1  tan     tan 1  tan    
   4   4 
 
 tan 1   tan   tan 1 (1)   tan 1 1
 4

 
tan 1 (  x)   tan 1 x   .
4
Property IV:
Period of sin 1 (sin x) is 2  (i) sin(sin 1 x)  x, x   1,1
(ii) cos(cos1x)  x, x   1,1
Example-06
(iii) sec(sec 1 x)  x  x (, 1]  [1, )
1  5π 
The principal value of sin sin  is
 6 (iv) cosec(cosec1 x)  x  x  ( , 1]  [1, )

(A)

(B)
5
(C)
7
(D)
5 (v) tan(tan 1 x)  x, x  R
6 6 6 6
(vi) cot(cot 1x)  x, x  R
Solution: (A)
5  
sin is to be written as sin  , where     Property (V):
6 2 2 
 5         (i) sin 1x  cos1x  , x   1,1
 sin 1  sin  =sin 1 sin       sin1  sin   2
 6    6   6 6 
(ii) sec1x  cosec1x  , x  ( , 1]  [1,  )
2
Example-07

 7π  (iii) tan 1 x  cot 1x  , x  R
The value of cos 1  cos  is 2
 6 
Example-9
  7 5 π
(A)
6
(B) 
6
(C)
6
(D)
6
If sin -1x = for some x   1,1 , then the value of
5
Solution: (D) cos1x is
7  3 5 7 9
 (A) (B) (C) (D)
Let cos 1  cos  10 10 10 10
 6 
Solution: (A)
7      3
 cos   cos  cos     cos 1 x   sin 1 x   
6  6 2 2 5 10
 3
  cos  Example-10
6 2
If cos 1x - sin 1x = 0 , then x is equal to
3
cos    and0    
2 1 1 1
(A)  (B) 1 (C)  (D)
 5 2 3 2
    
6 6
Solution: (D)

39
MATHS-X II-IIT-JEE INVERSE TRIGONOMETRIC FUNCTION
  Solution: (D)
cos 1 x  sin 1 x  0   sin 1 x  sin 1 x  0   2sin 1 x  0
2 2  1 2 
 1 1 2    1
 sin 1 x   x  tan 1  tan 1  tan 1  4 9   tan1
4 2 4 9 1 1. 2  2
 4 9
Example-11 Hence (D) is the correct answer.
 1  3
The value of cos 1    + sin 1    is Example-13
 2  2  In a ΔABC,if A = tan 1 2 and B = tan 1 3 then C is
 2 
(A) (B) 0 (C) (D) equal to
3 3 2   
Solution: (A) (A) (B) (C) (D) None
3 4 6
2     Solution: (B)
    ,
3  3 3 Since A, B, C are angles of a triangle,
Property VI: A  B  C    C    (A  B)
23 
tan 1 x  tan 1 y Now, A  B  tan 1 2  tan 1 3    tan 1  
(i) tan-1x+tan-1y  1  2.3 
 1 x  y    tan 1 ( 1)    tan 11 ....(A)
 tan 1  xy , xy  1
  1 1 1  x  y  
 xy  tan x  tan y    tan  
=    tan 1 , x  0, y  0, xy  1   1  xy  
 1  xy
   tan 1 x  y , x  0, y  0, xy  1  3 3 
   From (1), C    
 1  xy 4 4 4 4
-1
(ii) tan x – tan y= -1 Hence (B) is the correct answer.
 1 x  y
 tan 1  xy , xy  1 Example-14

 1 x  y 41
   tan , x  0, y  0& xy  1 cot 1 9  cosec1 is
 1  xy 4
   tan 1 x  y , x  0, y  0& xy  1  
 1  xy (A) (B)
3 4
Remark: 
If x1 , x 2 , x 3 ,...., x n  R ,then (C) (D) None of these
6
 s  s  s ...  Solution: (B)
tan 1 x1  tan 1 x 2  .....  tan 1 x n  tan 1  1 3 5 
 1  s 2  s 4  s6 ...  41 41
cot 1 9  cosec1  cot 1 9  cot 1 1
Where s1  x1  x 2  .....  x n   x1 4 16
s 2  x1x 2  x 2 x 3  .....  x n 1x n   x1x 2  cosec1 x  cot 1 x 2  1
 
s3   x1x 2 x 3 ..... and so on. 5 1 4
 cot 1 9  cot 1  tan 1  tan 1
4 9 5
Example-12
 1 4 
1 2 
tan 1  tan 1 is equal to 1  9 5 
4 9  tan  
 1 1 . 4 
1 3 1 3
(A) cos 1 (B) sin 1  9 5
2 5 2 5
 41  
1 3 1  tan 1    tan 11  .
(C) tan 1 (D) tan 1  41  4
2 5 2
Hence (B) is the correct answer.

40
INVERSE TRIGONOMETRIC FUNCTION MATHS-X II-IIT-JEE
Property VII: 1 5 2 8 
 sin 1  .  . 1
  sin x
sin 1 x  sin 1 y
3 3 3 3 
(1) sin-1x+sin-1y =
 54 2 


 
sin 1 x 1  y 2  y 1  x 2 ,if  1  x, y  1& x 2  y 2  1  x  
 9


 or if xy  0 & x 2  y 2  1
Hence (C) is the correct answer.


 
  sin 1 x 1  y 2  y 1  x 2 if 0  x, y  1& x 2  y 2  1
Property VIII:
1
 2 2

  sin x 1  y  y 1  x if  1  x, y  0 & x  y  1

2 2
(A) cos 1 x  cos 1 y 
-1 -1
(2) sin x-sin y =  1  2 2
cos  xy  1  x 1  y  if  1  x, y  1 & x  y  0
 
sin 1 x 1  y2  y 1  x 2 , if  1  x, y  1 & x 2  y 2  1
 

 or if xy  0 & x 2  y2  1  2  cos 1  xy  1  x 2 1  y 2  if  1  x, y  1& x  y  0
  


 
   sin 1 x 1  y 2  y 1  x 2 if 0  x  1,  1  y  0 & x 2  y 2  1
(B) cos 1 x  cos1 y 
1
 2 2

   sin x 1  y  y 1  x if  1  x  0, 0  y  1 & x  y  1

2 2

 1  2 2
cos  xy  1  x 1  y  if  1  x, y  1, x  y
Example-15 
  cos1  xy  1  x 2 1  y 2  if 0  x  1,  1  y  0 & x  y
2π   
If sin 1x + sin 1y = , then cos 1x + cos 1y is equal to
3 Example-17
2  
(A) (B) (C) (D)  If cos-1x + cos-1y + cos-1z = π ,
3 3 6
Solution: (B) then x 2 + y2 + z 2 + 2xyz is equal to
We have, (A) 0 (B) 1 (C) –1 (D) 2
2 Solution: (B)
sin 1 x  sin 1 y 
3 Let cos 1 x  ,cos 1 y   and cos 1 z  
    2  cos   x,cos   y and cos   z .Given
   cos 1 x     cos 1 y  
 2   2  3   
2  cos(  )  cos(  )
   (cos1 x  cos 1 y)   cos  cos   sin  sin    cos 
3

 cos1 x  cos1 y   
2 
  xy  z  1  x 2 1  y2
3 3
 x 2 y2  z 2  2xyz  1  x 2  y 2  x 2 y 2
Hence (B) is the correct answer.
(Squaring both sides)
Example-16  x 2  y2  z 2  2xyz  1 .
1 2 Hence (B) is the correct answer.
If sin -1 + sin -1 = sin -1x , then the value of x is
3 3 Example-18
( 5  4 2) p q p2
(A) 0 (B) If cos 1   + cos1   = α , then 2 + k
9 a b a
( 5  4 2)  q2
(C) (D) cosα + = sin 2 α , where k is equal to
9 2 b2
Solution: (C) 2pq 2pq pq pq
(A) (B) (C) (D) 
1 2 ab ab ab ab
We have, sin 1  sin 1  sin 1 x
3 3 Solution: (B)
1 4 2 1   p2   q2  
p q
 sin 1  1   1    sin 1 x cos 1  .   1  2   1  2    
3 9 3 9 a b  a   b  

41
MATHS-X II-IIT-JEE INVERSE TRIGONOMETRIC FUNCTION
y
pq  p2   q2 
  1  2  1  2   cos 
ab  –2tan–1x
 a   b 
2
–1
n
x –1 1
 pq  p 2 q 2 p 2q 2 –1
– –2tan x
2 ta
   cos    1  2  2  2 2 x
 ab  a b a b
p2 2pq q2 1  x2 2 tan 1 x if x  0
  cos    1  cos 2   sin 2  (4) cos1  
a 2 ab b2 1
1  x 2 2 tan x if x  0
2pq
k   .Hence (B) is the correct answer. 1  x2
ab graph of y  cos1
Property IX: 1  x2
y
(A) 2sin 1 x (B) 2cos 1 x (C) 2 tan 1 x –2 –1
tan –1 2tan x
(1) sin 1  2x 1  x 2   x
 
 1 1 1 0 x
2sin x if
2
x
2
 1
1  2tan 1 x, if  1  x  1
  2sin x, if  x 1
1 2x

 2 (5) tan 2
    2 tan 1 x, if x  1
  2sin 1 x if  1  x  1 1 x    2 tan 1 x if x  1
 2 

graph of y  sin 1  2x 1  x 2 
–1
an x
 –2t –1
  x
–1 x 0 –1
an tan x
2t –  2

Property X:
(A) 3sin 1 x (B) 3cos1 x (C) 3tan 1 x
 1 1 1
3sin x ; 2  x  2
(B) cos 1 (2x 2  1)   
(1) sin 1 3x  4x 3     3sin 1 x; 1  x  1
2

 2cos 1x if 0  x  1    3sin1 x;  1 x 
1
 1  2
 2  2cos x if  1  x  0
graph of y  cos 1 (2x 2  1)

(2) cos 1 (4x 3  3x) 


 1 1
 2  3cos x;  1  x  2
 2tan 1 x if  1  x  1  1 1
2x 1
  2  3cos x; x
(3) sin 1     2 tan 1 x if x  1 2 2
1 x 2 
   2 tan 1 x if x  1 3cos1x ;
1
  x 1
 2
graph of y  cos 1 (4x 3  3x)

42
INVERSE TRIGONOMETRIC FUNCTION MATHS-X II-IIT-JEE
Example-19

 3
sin  cos-1 
 5
Solution:
3
Let cos1  
5
 1 1
  3tan x, x  3 Then, cos  
3
 sin  
4
 3x  x 3   1 1 1 5 5
(3) tan 1    3tan x, x
 1  3x 2   3 3 4
    sin(cos1 3 / 5)  sin  
  3tan 1 x, x  1 5
 3
Example-20
 3x  x 3 
Graph of y  tan 1    3
 1  3x 2 
  cos  tan -1 
 4
y y=/2 Solution:
–1x
x

tan
–1

–1 x
n
 +3
an

3ta –  3
3t

y= y=
Let, cos  tan 1    .Then,
+

x
y=

 1   1 
  ,0 
 3 
 ,0 
 3   4

y=–/2 3 4
tan    cos    cos(tan 1 3 / 4)  cos   4 / 5
4 5
Property XI:
tan   3 / 4  cos   4 / 5
Conversion.
 x  Example-21
(i) sin 1 x  cos 1 1  x 2  tan 1  
 2 
 1 x  π  3 
sin  - sin -1  -  
2
 1  x2  
  sec1  1
 1   2 
 cot 1    cosec1  
 x   2  x
   1 x 
Solution:
 2 

(ii) cos 1 x  sin 1 1  x 2  tan 1  1  x   3      
 x 
sin   sin 1      sin      
2
    2   2  3 
 5  1
 x   1   sin   
1  6  2
cot 1    sec1    cosec1  
 2  x  2  4. SUMMATION OF SERIES CONTAINING
 1 x   1 x 

 x   1  INVERSE TRIGONOMETRIC TERMS


(iii) tan 1 x  sin 1    cos1  
 2   2 
 1 x   1 x 
Example-22

1  1 + x2  Find the sum to the n term of the series


cot -1   = sec-1  1 + x 2  = cosec-1  
x    x 
cosec–1 10 cosec–1 (n 2  1)(n 2  2n  2) + cosec–1
 

50 + cosec–1 170 + . . . +

43
MATHS-X II-IIT-JEE INVERSE TRIGONOMETRIC FUNCTION
Solution: Solution:

Let q = cosec–1 (n 2  1)(n 2  2n  2) Let tr denote the rth term of the series 3, 7, 13, 21, . and
S = 3 + 7 + 13 + 21 + . . . + tn
cosec2  = (n2 + 1) (n2 + 2n + 2)
Also S = 3 + 7 + 13 + . . . + tn–1 + tn
= (n2 + 1)2 + 2n (n2 + 1) + n2 + 1 = (n2 + n + 1)2 + 1
Subtracting we get
cot2 = (n2 + n + 1)2
= 3 + 4 + 6 + . . . + 2n – tn
1
(n  1)  n
tan  = 2 = tn = 3 + 4 + 6 + . . . + 2n
n  n 1 1  (n  1)n
1
 (n  1)  n  =3+ (n – 1) (4 + 2n) = n2 + n + 1
 = tan–1 2
1  (n  1)n 
 
 1 
Let Tr = cot–1 (r2 + r + 1) = tan–1  2 
= tan–1 (n + 1) – tan–1 n  r  r 1
Thus, sum n terms of the given series  r 1 r 
–1 –1 –1
= (tan 2 – tan 1) + (tan 3 – tan 2) + (tan 4 – –1 –1 = tan–1  1  r(r  1)  = tan–1 (r + 1) – tan–1 r
 
tan–1 3) + . . . + (tan–1 (n+1) – tan–1 n)
Thus, the sum of the first n terms of the given series is
 tan 1 (n  1)   / 4  n 11 
tan–1 (n + 1) – tan–1 (A) = tan–1 1  1(n  1) 
 
,,,,,...

Example-23  n 
= tan–1  n  2 
Find the sum of the first n terms of the series
cot–1 (C) + cot–1(7) + cot–1(13) + cot–1(21) + . . .

44
INVERSE TRIGONOMETRIC FUNCTION MATHS-X II-IIT-JEE

EXERCISE # 1
Based On Domain & Range   3  
8. cos cos1      
1. If, f (x)  sec 1 x  tan 1 x then f(x)is real for   2  6 
(A) x   1,1 3 1
(A) –1 (B) 1 (C) (D)
(B) x   1,1 2 2
(C) x  ( , 1  1, )
(D) x  (, 1)  (1,  ) 9. If y  cos1 (cos 4) then y =
(A) 4 (B) 2  4
2. If
(C) 2  4 (D) –4
1  x 2 x3   x4 x6  
sin  x    ...   cos 1  x 2    ...  
 2 4   2 4  2
     2 
10. tan 1  tan  
for, 0  x  2 then x equals  3 

1  2
(A) 1/2 (B) 1 (C) (D) –1 (A) (B)
3 3
2
 2
(C)  (D) 
3. If x satisfies the equation, t 2  t  2  0 then 3 3
there exists a value for
 33 
(A) sin 1 x (B) cos 1 x 11. The value of sin 1  cos 
 5 
(C) sec1 x (D)sin–1 2x 3 7
(A) (B)
5 5
Based On Principle Values
 
4. cos–1 (–1) = (C) (D) 
10 10

(A) (B) 0
2 
12. If, sin 1 x  , x  (1,1) then cos 1 x 
(C)  (D) 2 5

3  3 5
 (A) (B)
–1  tan  10 10
5. tan  4  is equal to-
3 9
  (C) (D)
(A) – (B) 10 10
4 4
 2 1   1  1  1 
3 3 13. sin 1  1
  cos    tan (  3)  cot   
(C) (D) –  2   2   3
4 4
17  11
6. cos–1(cos 5/4) is given by - (A) (B)
12 12
(A) 5/4 (B) 3/4
5 
(C) – /4 (D) None of these (C) (D)
12 12
 4 
7. cos1  cos    1  1
 3  14. The value of tan 1 (1)  cos 1     sin 1   
 2  2
4 2
(A) (B) is equal to
3 3
  (A)  / 4 (B) 5 / 12
(C) (D)
3 6 (C) 3 / 4 (D) 13 / 12

45
MATHS-X II-IIT-JEE INVERSE TRIGONOMETRIC FUNCTION
2  33 
15.  is the principal value of 23. The value of sin–1  cos 5  is -
5
 7   7 
(A) cos 1  cos  (B) sin 1  sin  3 7  
 5   5  (A) (B) (C) (D) –
5 5 10 10
 7    2  
(C) sec1  sec  (D) sin 1  sec   
 5    5  24. If x  2, then cos–1(cos x) is equal to-

16. The value of cos 1 (cos12)  sin 1 (sin12) is (A) x (B) –x


(A) 0 (B)   (C) 2 + x (D) 2 – x
 (C) 8  24 (D) 8  24

25. If tan–1 2x + tan–1 3x = then x =
   2  4
17. The value of  tan  cos1      / 2  is
   7  1
(A) –1 (B)
2 2 6
(A) (B)
3 5 3 1
(C) –1, (D) None of these
1 4 6
(D) (D)
5 5
1 1 1
26. cot [tan –1  tan –1  tan –1 ] =
Based On Properties of Inverse Functions 2 5 8
  17   (A) 1 (B) –1 (C) 2 (D) – 2
18. cos–1 cos   15    is equal to -

17 17  x –1 x 1 
(A)  (B) 27. If tan–1 + tan–1 = , then x =
15 15 x2 x2 4
2 13 1 1 5 5
(C) (D) (A) (B) – (C) (D) ±
15 15 2 2 2 2

  1  3 4
19. The value of sin arc cos  –  is- 28. If cos–1 – sin–1 = cos –1x, then x =
  2  5 5
1 (A) 0 (B) 1 (C) ½ (D) ¼
(A) (B) 1
2 29. If sin x + sin (1 – x) = cos –1x, then x =
–1 –1

3
(C) (D) none of these (A) 1, –1 (B) 1, 0
2
(C) 0, 1/2 (D) None
–1 –1 2
20. If sin x + sin y = , then cos–1 x + cos–1 y =
3 30. 1 + cot2 (sin–1 x) =
2  1 1 1
(A) (B) (A) (B) x2 (C) (D)
3 3 2x 2 4

(C) (D)  31. sin [cot–1 cos tan–1 x] is equal to -
6
21. tan–1 [tan (–6)] is equal to  x2 1   x2 
(A) 2 – 6 (B) 2 + 6 (A)  2  (B)  2 
x 2  x 1
(C) – 2 + 6 (D) – 6
22. cos–1 (cos 10) is equal to  x2  1 
(C)  2  (D) None of these
(A) 4 + 10 (B) 4 – 10 x 2
(C) – 4 + 10 (D) 10

46
INVERSE TRIGONOMETRIC FUNCTION MATHS-X II-IIT-JEE
32. If the sum of the acute angles tan–1 x and tan– 40. If, a  sin 1 x  cos 1 x  tan 1 x  b then
1
1   3 
 3  is 45º, then the value of x is - (A) a   , b  (B) a  0, b 
4 4 2
1 1   3
(A) (B) (C) a  , b   (D) a  , b 
3 4 2 4 2
1 1
(C) (D) 1 x 
5 2 41. If a  tan 1    b where 0  x  1 then (a, b)=
1 x 
1 4
33. If x = sin (2 tan–1 2), y = sin ( tan–1 ). Then (A) (0, )
 
(B)  , 
2 3
4 4
(A) x = y2 (B) y2 = 1 – x
y     
(B) x2 = (D) y2 = 1 + x (C)  0,  (D)  , 
2  2  4 2
 1
34. The value of sin(2sin 1 0.8) is equal to 42. The value of sin  2 tan 1   cos(tan 1 2 2)
 3
(A) sin 1.2 (B) sin 1.6
(A) 14/15 (B) –14/15
(C) 0.48 (D) 0.96
(C) 13/12 (D) 12/13
35. The value of cos(2cos 1 x  sin 1 x) at x=1/5 is 1
43. Given 0  x  then the value of
2 6 2 6 2
(A)  (B)
5 5   
x 1  x 2 
3 6 3 6 tan sin 1   1
  sin x  is
(C) (D)  2 2  
5 5    
1
1 1  (A) –1 (B) 1 (C) (D) 3
36. If sin x  cos x  then x = 3
6
(A)
1
(B) 
1 44.  
sin cot 1 (tan cos 1 x) is equal to
2 2
3  3 (A) (1  x 2 ) (B) x
(C) (D)
2 2 (C) 1/x (D) (1  x 2 )

37. The value of tan 2 (sec1 2)  cot 2 (cosec1 3) is x 3  2x   


(A) 13 (B) 15 45. If A  tan 1 and B  tan 1   then
2  x   3 
(C) 11 (D) 12
the value of A  B
2
(A) 00 (B) 300 (C) 450 (D) 600
  1 3  
38. sin  tan 4    46. If cos 1 x  sin 1 x then
  
9 9 1
(A) (B) (A) 0  x  (B) 1  x  0
16 25 2
16 16 1
(C) (D) (C) x  0 (D) 1  x 
25 9 2
3 
39. If, sin 1 x  sin 1 y  sin 1 z  then the value 47. 3cos1 x  x   0 has
2 2
9 (A) One solution
of x100  y100  z100  100 100 101
x y z (B) Two solutions
(A) 0 (B) 1 (C) 2 (D) 3 (C) No solution
(D) More than one solution

47
MATHS-X II-IIT-JEE INVERSE TRIGONOMETRIC FUNCTION
48. The set of values of  for which 54. The value of ‘ a ’ for which
2 1
x  x  sin (sin 4)  0 for all, x  R is
ax 2  sin 1 (x 2  2x  2)  cos1 (x 2  2x  2)  0
(A)  (B) (–2, 2)
(C) R (D) (–3, 3) has a real solution, is

49. If cos1 x  tan 1 x then  


(A) (B) 
2 2
5 1
(A) x 2  2 2
2 (C) (D) 
 
5 1
(B) x 2 
2 55. If  is the only real root of the equation,
5 1 x 3  bx 2  cx  1  0(b  c) then the value
(C) sin(cos1 x) 
2
1
5 1 of tan 1   tan 1   is equal to
1 
(D) tan(cos x) 
2
 
(A) (B) 
50. If, cos 1 x  cos1 y  cos1 z   then 2 2
(C) 0 (D) does not exist
(A) x 2  y 2  z 2  2xyz  1

(B) 2(sin 1 x  sin 1 y  sin 1 z)


56. For the equation, 2x  tan(2 tan 1 a) + 2 tan
 cos1 x  cos 1 y  cos1 z
(tan 1 a  tan 1 a 3 ) which of the following is
(C) xy  yz  zx  x  y  z  1 valid ?
 1  1  1 (A) a 2 x  2a  x (B) a 2  2ax  1  0
(D)  x     y     z  
 x  y  z
(C) a  0 (D) a  1,1
51. The value of
57. Identify the pair(s) of functions which are
   2 3  
1  12 
sin 1 cot  sin 1    cos    sec
1
2  identical.
   4   4  
  
1  x2
(A) 0 (B)  / 4 (C)  / 6 (D)  / 2 (A) y  tan(cos1 x); y 
x
52. The number of real solutions of 1
(B) y  tan(cot 1 x); y 
x
y  sin x, y  cos1 (cos x), 2  x  2
x
(A) 2 (B) 1 (C) 3 (D) 4 (C) y  sin(arctan x); y 
1  x2
53. The number of solutions for the equation, (D) y  cos(tan 1 x); y  sin(cot 1 x)
n
cos 1 (1  x)  m cos1 x  where,
2
58. a a  b  c b  a  b  c c a  b  c
tan 1  tan 1  tan 1 is
m  0,n  0 is bc ca ab

(A) 0 (B) 1 (C) 2 (D) infinite (A)  / 4 (B)  / 2 (C)  (D) 2

48
INVERSE TRIGONOMETRIC FUNCTION MATHS-X II-IIT-JEE
59. If x1 , x 2 , x3 , x 4 are roots of equation, 61. The sum of the infinite series
 1  1  2  1  1  3  2 
x 4  x 3 sin 2  x 2 cos 2  x cos   sin   0 then sin 1    sin    sin    ....
 2  6   12 
4  n  n 1 
 tan 1 x i   sin 1 
 n(n  1) 
 ....
i 1  
(A)  (B)  / 2   (C)    (D)      
(A) (B) (C) (D) 
8 4 2
Based On Series
60. Sum of infinite terms of the series
 3  3  3
cot 1 12    cot 1  22    cot 1  32    .... is
 4  4  4
(A)  / 4 (B) tan 1 2
(C) tan 1 3 (D) tan 1 4

49
MATHS-X II-IIT-JEE INVERSE TRIGONOMETRIC FUNCTION

EXERCISE # 2
1 sin(x /3)
One or More Than One Correct Answer 7. Let f  x   e cos then
Question
Type Questions
Which one is true  8   8 
1. (A) f    e5 /18 (B) f    e13/18
–1 –1 2
(A) 2 tan x = sin 2x / (1 + x ), when |x| 1  9   9 

(B) 2 sin–1 x = sin–1 { [2x (1  x 2 )],  7   7 


(C) f     e /12 (D) f     e11 /12
 4   4 
1 1
when x
2 2 8. If 6 sin–1 (x2 – 6x + 8.5) =  , then
(C) 2 tan–1 x = sin–1 2x / (1 + x2), when x >1 (A) x = 1 (B) x = 2
(D) 2 tan–1 x = sin–1 2 x / (1 + x2), when x< –1. (C) x = 3 (D) x = 4

 3  ax  –1  x  b 
2. 2 cot–17 + cos–1   is equal to 9. cos–1    sin   is possible, if
5  ab   a b 
 44   125  (A) a > x > b (B) a < x < b
(A) cot –1   (B) cosec –1  
 117   117  (C) a = x = b
 4   44  (D) a > b and x, takes any value
(C) tan –1   (D) cos –1  
 117   125 
 x 1 –1  2x  1  –1  23 
1  1  1  2 
10. If tan –1    tan    tan   ,
3. tan    tan   equals to  x 1  2x  1   36 
4 9
then x equals to
1  3 1 1  3 
(A) cos 1   (B) sin   (A)
4
(B) –
3
2  5 2  5 3 8
1  3 1 4 3
(C) tan 1   (D) tan 1   (C) – (D)
2  5 2 8 8
x 1 
4.
1 
  tan 1 (2tan 2 )  tan 1  tan   if 11. If f (x)  cos –1 x  cos –1   3 – 3x 2  then
3  2 2 
 
 2  2 2 
(A) tan   2 (B) tan   0 (A) f    (B) f    cos –1 
(C) tan   1 (D) tan   2  3 3 3 3 3
 1 1 1 
(C) f  –  (D) f    2cos –1 
1  14   2  3 3 3
5. The value of cos  cos1 cos    is
2  5  12. Let f(x) = sin–1 x + cos–1x. Then /2 is equal to

 7    1
(A) cos    (B) sin   (A) f  –  (B) f (k 2 – 2k  3), k  R
 5   10   2

 2   3   1 
(C) cos   (D)  cos   (C) f  , k  R (D) f(–2)
 5   5   1 k2 
6. Indicate the relation which is true
 x 1  –1  x  1  
13. If tan –1    tan    , then x is
(A) tan tan 1 x  x (B) cot cot 1 x  x x2 x2 4

(C) tan 1 tan x  x (D) sin sin 1 x  x 1 1 3


(A) – (B) 0 (C) (D)
2 2 2

50
INVERSE TRIGONOMETRIC FUNCTION MATHS-X II-IIT-JEE
14. The value(s) of x satisfying the equation 22. One of the values of x satisfying tan (sec–1x) = sin
1
sin 1 sin x  sin 1 sin x is/are given by cos–1 is
5
(n is any integer)
(A) n  1 (B) n 5 3
(A) (B)
(C) n  1 (D) 2n  1 3 5
5 3
15. If sin–1 (3x/5) + sin–1 (4x/5) = sin–1x then x equals (C) – (D) –
3 5
1
(A) 0 (B) 1 (C) –1 (D)  5   2 
2 23. Let  = tan–1  tan  and  = tan–1  – tan 
 4   3 
16. ,  and  are the angles given by
then-
 1  1  1  (A) >  (B) 4 – 3 = 0
  2 tan 1 ( 2  1),   3sin 1    sin   2 
 2   7
(C)  +  = (D) None of these
1 12
and   cos 1   , then
3 1
24. If < |x| < 1 then which of the following are
(A)    (B)    2
(C)    (D)    real-
(A) sin–1 x (B) tan–1 x
–1
17. If, tan 1 y  4 tan 1 x the y is infinite, if (C) sec x (D) cos–1 x

(A) x 2  3  2 2 (B) x 2  3  2 2 
25. Let f (x) = sin–1x + cos–1x. Then is equal to-
2
(C) x 4  6x 2  1 (D) x 4  6x 2  1
 1
(A) f  – 
18. 1 2
If the equation sin (x  x  1)  cos 1  2
(B) f(k2 – 2k + 3), k  R

(x  1)  has exactly two solutions, then   1 
2 (C) f  ,kR
 1 k 2 
cannot have the integral value  
(A) –1 (B) 0 (C) 1 (D) 2 (D) f(–2)

19. Let y = (sin–1 x)3 + (cos–1 x)3 then 26. sin–1 x > cos–1 x holds for-
(A) all values of x (B) x  (0, 1/2)
3 3
(A) min y  (B) min y  (C) x  (1/2, 1) (D) x = 0.75.
4 32
7 3 7 3
27. ,  and  are three angles given by
(C) min y  (D) min y  1  1
8 32  = 2 tan–1 (2 – 1),  = 3 sin–1 + sin–1  – 
2  2
20. The solution of the equation 1
sin[2cos–1 {cot (2 tan–1x)}] = 0 are and  = cos–1 . Then-
3
(A) 1 (B) 1  2 (A)  >  (B)  > 
(C) –1  2 (D) none of these (C)  <  (D) none of these
Question Passage Based Type Questions
21. Exhaustive set of parameter ‘a’ so that Passage # 1 (Q. 28 to Q. 30)
sin 1 x  tan 1 x  a has a solution, is
This section contains 2 paragraphs. Based upon each
     
(A)   ,  (B)   ,  paragraph, with multiple choice questions have to be
 6 6   4 4
      answered. Each question has 4 choices (A), (B), (C)
(C)   ,  (D)   , 
 2 2  12 12  and (D) out of which only one is correct.

51
MATHS-X II-IIT-JEE INVERSE TRIGONOMETRIC FUNCTION
The functions sin–1x, cos–1x, tan–1x, cosec–1x and sec–1x 31. cos 1 (cos )   , for all  belonging to
are called inverse circular or inverse trigonometric
  
functions, which are defined as follows (A)  0, (B)   , 
 2 2
 3
sin 1 x 1  x  1  sin 1 x     
2 2 (C)   ,   0 (D)  0,    
 2 2 2
cos1 x 1  x  1   cos 1 x  0
  32. sec1 (sec )   , for all  belonging to
tan 1
x x R   tan 1 x 
2 2 (A)  0,    / 2 (B)  0,     / 2
 1 3  
cosec 1x x 1  cosec x   (C)  0,  (D)  0, 
2 2  2

sec1 x x 1   sec1 x  0 
2 33. tan 1 (tan )    2 , for all  belonging to
cot 1 x x R 0  cot 1 x   (A) r (B) 

28. For x   0,1 ,sin 1 x is equal to  3 5    3 


(C)  ,  (D)  , 
 2 2  2 2 

(A) cos 1 1  x 2 (B) cos 1 1  x 2 
2 34. The value of, sin 1 (sin12)  cos1 (cos12) is

(C)  cos1 1  x 2 (D) sin 1 1  x 2   


(A) 0 (B)  (C) (D)
2 2 3

29. Number of solutions of tan 1 x  cos 1 x  0 35. The, sin 1[cos{cos 1 (cos x)  sin 1 (sin x)}]
is/are  
where, x   ,   is
(A) 2 (B) 1 (C) 0 (D) 3 2 
 
4 2 (A) (B)  (C)  (D) 
30. The root of the equation x  6x  1  0 is 2 2
  Passage #3 (Q. 36 to Q. 39)
(A) tan (B) tan While defining inverse trigonometric functions, a new
12 4
system is followed where domains and ranges have
  been redefined as follows.
(C) tan (D) tan
8 16 Function Domain range
Passage # 2 (Q. 31 to Q. 35)   3 
sin 1 x  1,1 2 , 2 
We know that corresponding to every bisection (one-one  
function), f : A  B there exists a bisection g : B A
  3 
defined by g(y)  x if f (x)  y and only if. g : B  A tan1 x R  , 
2 2 
the function f : A  B is called the inverse of function
and is denoted by. f 1 thus, we have. cos 1 x  1,1 [ , 2]

f (x)  y  f 1 (y)  x we have also learnt that for all cot 1 x R [ , 2]
1
x  A and, (fof 1 )  f (f 1 (y))  f (x)  y for all. y  B 36. sin ( x) is equal to
we know that trigonometric functions are periodic (A)  sin 1 x
functions and hence, in general all trigonometric
functions are not injective. Consequently, their inverse (B)   sin 1 x
do not exists. However, if we restrict their domains and (C) 2  sin 1 x
co domains, they can be mod bisections and we can
obtain their inverse. (D) 3  cos 1 1  x 2 , x  0

52
INVERSE TRIGONOMETRIC FUNCTION MATHS-X II-IIT-JEE

37. If f (x)  3sin 1 x  2cos 1 x then f (x) is 3


40. sin–1x < then solution set of x is-
(A) even function 4
(B) odd function  1   1 
(A)  ,1 (B)   , –1
(C) neither even nor odd  2   2 
(D) even as well as odd function  1 1 
(C)   ,  (D) none of these
38. The minimum of (sin 1 x)3  (cos 1 x)3 is equal  2 2
to 41. sin–1 x + cosec–1 x at x = –1 is -
63 63 3 (A) (B) 2
(A)  3 (B) 
8 8 (C) 3 (D) – 
1253 1253 42. If x  [–1, 1], then range of tan–1 (–x) is
(C) (D)
32 32  3 7    3 5 
(A)  ,  (B)  , 
4 4 4 4
39. If in another such system range of sin 1 x is
 5 3  5   
1 1 (C) [–, 0] (D)   , 
  2 ,  2  and that of, sin x  cos x  2  4 4
 
then in the same system, range of cos 1 x will be Passage # 5 (Q. 43 to Q. 45)
(A) [4,5] Let us define here two functions in the domain of
(B) [3,4]
[–1, 1], f(x) = sin–1 x, g(x) = cos–1 x
(C) [6,7] (D) [5,6] Now another function
Passage #4 (Q. 40 to Q. 42) I (x) = max. {f(x), g(x)}, – 1  x  1
The domain and range of inverse circular function are J (x) = min {f(x), g(x)}, – 1  x  1
defined as follows :
On the basis of above passage, answer the following
Function Domain Range
questions:
  3  43. The range of J(x) is-
sin–1x [–1, 1] 2 , 2 
       
–1 (A)  – ,0 (B)  – , 
cos x [–1, 1] [0, ]  2   2 4
  3 
tan–1x R  ,    
(C)  – ,  (D) none of these
2 2   2 2
cot–1x R (0, )
44. The number of points where I(x) is not
  3 
cosec–1x (–, –1]  [1, )  2 , 2  –{} differential are -
  (A) 3 (B) 1
 (C) 0 (D) Infinite
sec–1x (–, –1]  [1, ) [0, ] –  
2
45. The set of x such that J(x) is increases, is -
(A) (–1, 0) (B) (–1, 1/2)
(C) (–1, 1) (D) none of these

53
MATHS-X II-IIT-JEE INVERSE TRIGONOMETRIC FUNCTION

EXERCISE # 3
Question Column Match Type Questions
3. Let f(x) = sin–1x , g(x) = cos–1x and h(x) = tan–1x. For
1. Match the items of column-I with the items what interval of variation of x the following are
column-II true.
Column - I Column I Column II
(A) f ( x )  g( x )   / 2 (p) [0, )
(A) cos 1   cos1   cos1   3
then      is (B) f (x)  g  1  x 2   0 (q) [0, 1]
 
10 10  1  x2 
(B)  cos 1 x i  0 then  x i is (C) g    2h(x)
 1  x2 
(r) (–, 1)
i 1 i 1  
2n 2n 1 x 
(D) h(x) + h(A) = h   (s) [–1, 0]
(C)  sin 1 xi  n then,  xi is 1 x 
i 1 i 1

 3 1  Question Numerical Type Questions


(D) f (x)  sin 1  x  1  x 2  ,
 2 2  4. If the sum of the series cot–1 2 + cot–1 8 + cot–1 18
1 k
  x  1 is + ... Upto  is then find k.
2 4

Column -II 
 2m  k
1 
5. If.  tan –1  m 4  m 2  2   4
Find k
(p) 2n (q) sin x m 1
2
(r) 10 (s) 3 6. If r = x + y + z, &
xr yr zr
tan –1  tan –1  tan –1  k
1 x 1 x yz zx xy
2. Let t1  (sin 1 x)sin , t 2  (sin 1 x)cos ,
then find k
sin 1 x cos1 x
t 3  (cos1 x) , t 4  (cos 1 x)
7. The number of positive integral solutions (x, y)
Match the items of column I with the items column II. of the equation tan–1 x + cot–1 y = tan–1 3 are
Column - I Column - II
5 2
(A) x   0,cos1 (p) t1  t 2  t 4  t 3 8. If (tan–1 x)2 + (cot–1 x)2 = , find |x|
8
 1 
(B) x   cos1,  (q) t 4  t 3  t1  t 2 9. If sin–1 x + sin–1 y + sin–1 z =  then find
 2
x 4  y4  z 4  4x 2 y2 z 2
 1 
(C) x   ,sin1 (r) t1  t 2  t 3  t 4 x 2 y 2  y2 z 2  z 2 x 2
 2 
(D) x   sin1,1 3
(s) t 3  t 4  t1  t 2 10. If sin–1 x + sin–1 y + sin–1 z = find
2
(x101  y101 )(x 202  y202 )

(x 303  y303 )(x 404  y 404 )

54
INVERSE TRIGONOMETRIC FUNCTION MATHS-X II-IIT-JEE

EXERCISE # 4
Question Previous Year (JEE Mains)  2x 
7. If f (x)  2 tan 1 x  sin 1   , x  1, then
y  1 x2 
1. If cos–1x–cos–1 2 =  then 4x2 – 4xy cos  +y2
f (5) is equal to [JEE Main 2015]
is equal to [AIEEE 2005]
(A) 4 (B) 2sin2 (A)  / 2 (B) 
(C) –4sin2 (D) 4sin2 65
(C) 4 tan 1  5  (D) tan 1
156
   
2. The largest interval lying in  ,  for which
 2 2
2   8. The value of
the function, f (x)  4 x  cos 1   1  log
2   1  x2  1  x 2  1
(cos x) is defined, is [AIEEE 2007] tan 1   x  , x  0, is equal
2
 1 x  1 x  2 2
    
(A)   ,  (B)  0,
 4 2 to [JEE Main 2017]
    
(C) 0,  (D)   ,   1  1
 2  2 2 (A)  cos 1 x 2 (B)  cos 1 x 2
4 2 4 2
 x  5  5   
3. If sin–1   Cosec–1  4   4   2 , then value of x (C)  cos 1 x 2 (D)  cos1 x 2
 5 4 4
is [AIEEE2007]
(A) 4 (B) 5 (C) 1 (D) 3 9. A value of x satisfying the equation

4.
 5 2
The value of cot  cosec 1  tan 1  is sin[cot 1 (1  x)]  cos[tan 1 x], is
 3 3
[JEE Main 2017]
[AIEEE 2008]
5 6 1 1
(A) (B) (A) (B) 0 (C) –1 (D) 
17 17 2 2
3 4
(C) (D)
17 17
 2   3   3
–1 –1 – 10. If cos 1    cos 1     x   then x
5. If x, y, z are in A.P. and tan x, tan y and tan  3x   4x  2  4
1
z are also in A.P., then [AIEEE 2013]
(A) 2x = 3y = 6z (B) 6x = 3y = 2z is equal to : [January 2019]
(C) 6x = 4y = 3z (D) x = y = z 145 145
(A) (B)
 2x  12 10
6. Let tan 1 y  tan 1 x  tan 1   , where
 1 x2  146 145
1 (C) (D)
x . Then a value of y is 12 11
3
[JEE Main 2015]
3 11. If x = sin–1(sin 10) and y = cos–1 (cos10), then y
3x  x 3x  x 3
(A) (B) – x is equal to: [January 2019]
1  3x 2 1  3x 2
3x  x3 3x  x 3 (A)  (B) 7
(C) (D)
1  3x 2 1  3x 2 (C) 0 (D) 10

55
MATHS-X II-IIT-JEE INVERSE TRIGONOMETRIC FUNCTION
 19  n  horizontal top of a table such that PC = 5
12. The value of cot   cot 1 1   2p   is –
 n 1


 p 1

 inches and PCB = tan–1 2. The acute angle
through which the pencil must be rotated about
[January 2019] C so that the perpendicular distance between
(A) 22 / 23 (B) 23 / 22 eraser and pencil becomes exactly 1 inch is
(C) 21 / 19 (D) 19 / 21 [Aug. 2021]
P
 3  1

in
13. If   cos1   ,   tan 1   , where

5
5  3
 A 5 in C 5in B
0  ,   , then  –  is equal to :
2 (A) tan–1 (1/2) (B) tan–1 (3/4)
[April 2019] (C) tan–1 1 (D) tan–1 (4/3)
 9   9
(A) sin 1   (B) tan 1   50
1
 5 10   14  19. If  tan 1 2r 2  p, then the value of tan p is
r 1
1 9  9 
1  [Aug. 2021]
(C) cos   (D) tan  
 5 10   5 10  (A) 100 (B) 50 / 51
(C) 101 / 102 (D) 51 / 50
y
14. If cos–1x – cos–1 = , where – 1  x  1,
2 20. If (sin–1x)2 – (cos–1 x)2 = a ; 0 < x < 1, a  0, then
–2  y  2, x  y/2, then for all x, y, 4x – 4xy 2 the value of 2x2 – 1 is [JEE MAIN 2021]
 2a   4a 
cos  + y2 is equal to [April-2019] (A) cos   (B) sin  
2 2 2     
(A) 4 sin  – 2x y (B) 4 cos2 + 2x2y2
4a
   2a 
(C) 4 sin2  (D) 2 sin2 (C) cos   (D) sin  
    

15. The value of sin–1 (12/13) – sin–1 (3/5) is equal 21. If y (x) = cot–1 [JEE MAIN 2021]
to: [April-2019]  1  sin x  1  sin x   
  , x   ,  
(A)  – sin–1 (63/65) (B)  – sin–1 (33/65)  1  sin x  1  sin x  2 
(C) /2 – sin–1 (56/65) (D) /2 – sin–1 (9/65) then
dy
at x 
5
is
dx 6
(A) –1/2 (B) 0 (C) –1 (D) 1/2
16. If S is the sum of the first 10 terms of the series
tan–1 (1/3) + tan–1 (1/7) + tan–1 (1/13) 22. cos–1 (cos (–5)) + sin–1(sin (6)) – tan–1(tan (12))
is equal to (The inverse trigonometric functions
+ tan–1 (1/21) + ......, take the principal values) [JEE MAIN 2021]
then tan (S) is equal to : [September 2020] (A) 3 + 1 (B) 3 – 11
(A) 5 / 11 (B) –6 / 5 (C) 4 – 11 (D) 4 – 9
(C) 10 / 11 (D) 5 / 6 Question Previous Year (JEE Advanced)
 x 2 x3 
23. If sin–1  x –  – .......... 
 4 5 16   2 4 
17. 2   sin 1  sin 1  sin 1  is equal to:  
 5 13 65 
 x4 x6  
[September 2020] + cos–1  x 2 –  – ..........  =
 2 4  2
 
(A) 7/4 (B) 5/4 (C) 3/2 (D) /2
for 0 < |x| < 2 , then x equals [IIT 2001]
1 1
18. A 10 inches long pencil AB with mid point C (A) (B) 1 (C) – (D) – 1
and a small eraser P are placed on the 2 2

56
INVERSE TRIGONOMETRIC FUNCTION MATHS-X II-IIT-JEE
24. cos tan–1 sin cot–1 x = [IIT-2002] (Q) If cos x + cos y + cos z = 0 = sin x +sin y +
2
x 1 2
x –1 xy
(A) (B) sin z then possible values of cos   is
2
x 2 x2  2  2 
 
x2 1 x 2 –1 (R) If cos   x  cos2x + sin x sin 2x sec x =
(C) (D) 4 
x2 – 2 x2 – 2
 
25. For which value of x, cos x sin 2x sec x + cos   x  cos2x then
4 
sin (cot–1 (x + 1)) = cos (tan–1 x) [IIT Scr. 04]
(A) 1/2 (B) 0 (C) 1 (D) – 1/2 possible value of sec x is

26. Let (x, y) be such that (S) If cos (sin 1 1  x 2 )  sin(tan 1 (x 6)),
 x  0 then possible value of x is
sin–1 (ax) + cos–1 (y) + cos–1 (bxy) = Lest-II
2
Match the statement in Column I with 1 5 1
(A) (B) 2 (C) (D) 1
statements in Column II. [IIT- 2007] 2 3 2
Column I Codes :
(A) If a = 1 and b = 0, then (x, y) P Q R S
(B) If a = 1 and b = 1, then (x, y) (A) 4 3 1 2
(C) If a = 1 and b = 2, then (x, y)
(B) 4 3 2 1
(D) If a = 2 and b = 2, then (x, y)
(C) 3 4 2 1
Column II
(P) lies on the circle x2 + y2 = 1 (D) 3 4 1 2
(Q) lies on (x2 – 1) (y2 – 1) = 0 31. Let f :[0, 4]  [0, ] be defined by f(x) = cos–1
(R) lies on y = x
(S) lies on (4x2 – 1) (y2 – 1) = 0 the number of points x  [0,4] satisfying the
10  x
27. If 0 < x < 1, then 1  x 2 [{x cos (cot–1 x) + sin equation f (x)  is [JEE Adv. 2013]
10
(cot–1 x)}2 – 1]1/2 is equal to [IIT- 2008]
x 32. For any positive integer n, define fn : (0,  R
(A) (B) x n
 1 
1  x2 as fn(x) =  tan 1  
j1  1  (x  j) (x  j  1) 
(C) x 1  x 2 (D) 1  x 2
for all x ).
  sin    (Here, the inverse trigonometric function tan–1x
28. Let f() = sin  tan –1    , where
  cos 2     
assumes values in   ,  ).Then, which of
  d(f ())  2 2
– <  < . Then the value of is
4 4 d(tan ) the following statement(s) is (are) TRUE ?
[IIT-2011] [JEE Adv. 2013]
5
 23  n  
29. The value of   cot 1 1   2k   is
(A)  tan 2 (f j (0))  55
  j1
 n 1  k 1  
[JEE Adv. 2013] 10
23 25 23 24
(B)  (1  f j(0))sec2 (f j (0))  10
(A) (B) (C) (D) j1
25 23 24 23
(C) For any fixed positive integer n,
30. Match list-I with list-II and select the correct 1
answer using the code given below the lists. lim tan (f n (x)) 
x  n
[JEE Adv. 2013]
(D) For any fixed positive integer n,
Lest-I
  1 1 2
1/2
 lim sec2 (f n (x))  1
(P)  1  cos(tan y)  y sin(tan y)   y 4  takes x 
 y 2  cot(sin 1 y)  tan(sin 1 y)  
    lim sec2 (f n (x))  1
x 
value

57
MATHS-X II-IIT-JEE INVERSE TRIGONOMETRIC FUNCTION
x 35. The number of real solutions of the equation
33. Let E1 = {x  R : x 1  0 and } and
x 1  i
x 

sin 1   x i1  x    
 
i 1  2  
   x  
E 2   x  E1 : sin 1  log e    is a real number   i 1
   x 1 
    x i  
(Here, the inverse trigonometric function sin–1x   cos1        (  x)i  lying in the
2   2 
    i1 i 1 
assumes values in   ,  )
 2 2  1 1
interval   ,  is ___. (Here the inverse
Let f : E1  R be the function defined by f (x) =  2 2
loge  x  and g : E2  R be the function trigonometric functions sin–1x and cos–1x
 x 1     
  x  assume values in  ,  and  0,
defined by g (x) = sin–1  log e  x  1    2 2
respectively.) [JEE Adv. 2018]
[JEE Adv. 2013]
LIST-I 36. The value of
(P) The range of f is
(Q) The range of g contains  1 10  7 k   7 (k  1)  
sec1   sec    sec   
4 2  
(R) The domain of f contains  k 0  12 2   12
(S) The domain of g is
LIST-II   3 
In the interval   ,  equals___ .
 1   e   4 4
(A)  ,   , 
[JEE Adv. 2019]
 1  e   e 1 
(B) (0, 1)
 1 1 37. For any positive integer n, let Sn : (0, ) R
(C)   ,  be defined by
 2 2
(D) (–, 0)  (0, ) n  1  k (k  1) x 2 
Sn (x)   cot 1   , where for
 e   x 
(E)  , k 1  
 e  1 any
1 e  x R , cot–1x  (0, ) and tan–1x  .
(F) ( , 0)   , 
 2 e  1
  
The correct option is  ,  Then which of the following
(A) P - 4; Q - 2; R - 1; S - 1  2 2
(B) P - 3; Q - 3; R - 6; S - 5 statements is (are) TRUE ? [JEE Adv.2021]
(C) P - 4; Q - 2; R - 1; S - 6   1  11x 2 
(D) P - 4; Q - 3; R - 6; S - 5 (A) S10 (x)   tan 1   , for all x > 0
2  10x 
6  4  
34. If  = 3 sin–1   and  = 3cos–1   where
 11  9 (B) lim cot (Sn (x))  x , for all x > 0
n 
the inverse trigonometric functions take only
the principal values, than the correct option(s) is 
(C) The equation S3 (x) = has a root in (0, )
(are) [JEE Adv. 2015] 4
(A) cos   0 (B) sin   0 (D) tan (Sn (x)) 1/2, for all n  1 and x > 0
(C) cos       0 (D) cos   0

58
INVERSE TRIGONOMETRIC FUNCTION MATHS-X II-IIT-JEE

ANSWER KEY

EXERCISE-1

Qus. 1 2 3 4 5 6 7 8 9 10 11 12 13 14 15 16 17 18 19 20
Ans. C B C C A B B A B C D A A C B D A D C B
Qus. 21 22 23 24 25 26 27 28 29 30 31 32 33 34 35 36 37 38 39 40
Ans. A B D D B A D B C C C D B D A C C B A A
Qus. 41 42 43 44 45 46 47 48 49 50 51 52 53 54 55 56 57 58 59 60
Ans. B A B A B D A A C A A C A B B A D C B B
Qus. 61
Ans. C

EXERCISE-2

Qus. 1 2 3 4 5 6 7 8 9 10 11 12 13 14 15
Ans. AB ABD AD ACD BCD ABD BC BD AB AB AD AC AC ABC ABC
Qus. 16 17 18 19 20 21 22 23 24 25 26 27 28 29 30
Ans. BC ABC ACD BC ABC BC BD BC ABD AC CD BC A B C
Qus. 31 32 33 34 35 36 37 38 39 40 41 42 43 44 45
Ans. A A C A D C B A A A C B B A B

EXERCISE-3
1. [A] [S], [B] [R], [C] [P], [D] [Q]
2. [A] [Q], [B] [S], [C] [R], [D] [P]
3. [A] [Q], [B] [S], [C] [P], [D] [P][Q][R][S]
4. [1] 5. [1] 6. [1] 7. [2] 8. [1] 9. [2] 10. [3]

EXERCISE-4
Qus. 1 2 3 4 5 6 7 8 9 10 11 12 13 14 15
Ans. D C D B D C B B D A A C A C C
Qus. 16 17 18 19 20 21 22 23 24 25 26 27 28 29 30
Ans. D C B B D A C B C D C 1 B B
Qus. 31 32 33 34 35 36 37
Ans. 3 D A B,C,D 2 0 A,B

26. [A]  [P] ; [B]  [Q] ; [C]  [Q] ; [D]  [S]

59
LIMIT MATHS-X II-IIT-JEE

CHAPTER

LIMIT
1. DEFINITION 3. EXISTENCE OF LIMIT
 Let the function y = f(x) be defined in a certain  Let f(x) has the indeterminate form at x = a then the
neighbor hood of a point x = a. The function y = f(x) limit of a function at a point a exists only when its
left hand limit and right hand limit at that point are
approaches the limit L (y  L) as x approaches a (x
equal and unique. Thus
 a), if for every positive number h arbitrarily small,
we are able to indicate k > 0 such that for all x, lim f(x) exists
x a
different from a and satisfying the inequality.
 lim– f(x) = lim f(x) = unique
|x–a|<h x a x a
we have the inequality i.e. RHL = LHL = unique value
| f(x) – L | < k
then lim f(x) = L Example-1
x a
[x]
or f(x)  L as x  a. Evaluate Lim , where [.] is G.I.F.
x→ ∞ x
y = f(x)
1 [x ]
Y Solution .:We have x – 1 < [x]  x,  1   1
L+k x x
M 2k  1
L Now L im  1  1.
L–k x   x 
[x]
Therefore by Sandwich theorem Lim 1
x x

h h
X Example-2
O a–h a a+h ln x
Evaluate Lim .
If f(x)  L as x  a, this is illustrated on the graph x→  x
of the function y = f(x) as above. Since from the Solution .: As 0 ln x  x (x > 1)
inequality |x – a| < h there follows the inequality
n x 1
|f(x) – L| < k, this means that for all points x that are 0   (x > 1)
not more distant from the point a than h, the points x x
M of the graph of the function y = f(x) lie within a n x 1
0  Lim  Lim =0
band of width 2k bonded by the lines y = L – k and x x x  x
y = L + k. n x
 Lim =0
x x
2. INDETERMINATE FORMS
 Some times we come across some functions which
4. THEOREMS ON LIMIT
do not have definite value corresponding to some
 The following theorems are very helpful for
particular value of the variable.
evaluation of limits-
For example for the function
(i) lim [k f(x)] = k lim f(x), where k is a constant
x2  4 44 0 x a x a
f(x) = , f(B) = =
x 2 22 0 (ii) lim [f(x) + g(x)]= lim f(x) + lim g(x)
x a x a x a
f (B) cannot be determined. Such a form is called an
Indeterminate form. Indeterminate forms are : (iii) lim [f(x) – g(x)] = lim f(x) – lim g(x)
x a x a x a
0/0, /, – , 0 ×  , 1, ° , 00

60
MATHS-X II-IIT-JEE LIMIT

(iv) lim [f(x). g(x)] = lim f(x). lim g(x)


x a x a x a
 x   x as x   , x is negative 
1 1
(v) lim [f(x)/g(x)]= lim f(x)/[ lim g(x)]  lim 
x a x a x a x  - 1 2
provided lim g(x)  0  1 1
x a x2
This is the correct answer
(vi) lim f[g(x)]= f [ lim g(x)]
x a x a Key concept : Using the concept of method of
(vii) lim [f(x)+k] = lim f(x) + k substitution put
x a x a 1
x as x     t  0
where k is a constant t
(viii) lim log {f(x)} = log { lim f(x)} 1 1
x a x a 1 
limg(x) t2 t
(ix) lim [f(x)]g(x) ={ lim f(x) }xa lim
x a x a t 0 t
(x) lim f(x) = lim f(1/x)
x x 0
 lim
 1  t  1 
2
1  t2  1
t 0 t21  t2  1
Example-3
1 1
x - b - (a - b)  lim 
Evaluate lim , a>b x - 2
 1 t 1 2
x→a x2 - a 2
x  b  (a  b) x  b  (a  b) Method when x   : If given limit is in this form
Sol.: lim .
x a 2
x a 2
x  b  (a  b) f (x)
lim ,where f(x) and g(x) are algebraic function in
x  g(x)
(x  a) 1
lim  x we divide numerator and denominator by highest
x a
 x  a  x  a   x b  a b  4a a  b power of x in f(x) and g(x)

Example-5
Example- 4
3 x + x1/ 3 - x1/ 4
Evaluate lim x x + 1 + x 2
x -
  Evaluate lim
x + 5x1/ 3 + 7
x →

Common mistakes: How student used to solve this Solution .:Here highest power in f(x) and g(x) is x1/2.
problem. Hence divide numerator and denominator by x1/2 and
then apply the limit.
1 + x2 - x
x→ - 
2
Step1. lim x x + 1 + x .   1 + x2 - x 3
1
1/6
1
 1/4
lim x x 3
x x  5 7
lim 1  1/6  1/2
x→ – 
1 + x2 - x x x
x
Step2. lim =
1x→ -  5. SQUEEZE PLAY THEOREM
+1 -1
x2 (SANDWICH THEOREM)
But this is wrong, can you find out where is the
mistake.
1  x2  x
2
Solution : lim x x  1  x .
x -
  1  x2  x
x
Step1. lim
x -
1 x2  x If f (x)  g(x)  h(x)  x  (, )  {a} and
x
Step2. lim Limf (x)    Limh(x) then Limg(x)   ,
x -  1 x a x a x a
x 1  x where a  ( , ) .
x2

61
LIMIT MATHS-X II-IIT-JEE
6. SOME STANDARD APPROACHES TO FIND  x  1  x 2  x  1   x  1 log x 
LIMIT OF A FUNCTION = lim
6.1 Substitution
x 1  x  1 x  1
6.2 Factorization 12  1  1  1  1 log1 3
6.3 Rationalization = 
1  1 2
6.4 Expansion of function
6.3 Rationalization method:
6.5 By application of standard limits
In this method we rationalize the factor containing the
6.6 By use of logarithm
square root and simplify and we put the value of x.
6.7 L' Hospital rule
6.1 Substitution: These are few important substitutions Example-8
which are often used.
Evaluate lim x
x→±∞
 
x2 + k – x , k > 0
Example-6

Evaluate lim =
1- x Solution.: lim x  x  k  x 
 2
x2  k  x 
  k  x
2 x 
x 1
cos x  -1 x2

Key concept: First of all substitute cos–1x = , use the 


x x2  k  x2 =
method of substitution. xk
= lim lim
Solution .: Let cos–1x = , then x = cos
Now as x  1   cos–1(A), i.e.   0
x 
 2
x k x 
  x 
k
 | x | 1  2
 x

   x
  
1 x 1  cosθ Here we have to consider two cases
lim 2
 lim
θ2
x 1

cos1 x 
θ 0 (i) When x ; |x| = x then the given limit
xk
1  cosθ 2sin 2 θ/2 = lim
 lim  lim x  k 

x 1 θ 2
1  cosθ  θ 0 θ2
1  cos θ  x 1  2   x
 x 
2
  xk k
= lim 
 sin θ/2  θ 2 x   k   2
2  x   1  2   1
 θ  4 
  x  

1
 lim  2   (ii) When x  – ; |x| = – x then we have

θ  0 θ 2 1  cos θ 4  lim
xk
f (x) x–
 k 
6.2 Factorization : Consider lim .  x 1  2   x
g(x) x a
 x 
Factories numerator and denominator and cancel out the xk k k
common factor (x – a). After cancelling out the lim  –  –  –
x –   k  1  1 0
common factor (x – a), put x = a in given expression x   1  2  1
and the process is repeated till a meaningful number is  x 
obtained.
6.4. Expansion of function
Example-7
(Some standard expansions)
x 3  x 2 log x  log x  1
Evaluate lim x2 x3 xn
x 1 x2 1 (i) ex = 1 + x + + +....... +.....
2! 3! n!
Solution .: The given limit
x2 x 3
lim
x 3
 
 1  x 2  1 log x  (ii) n (1+x) = x –
2
+
3
– ….; –1 < x  1
x 1 x2 1
 x  1  x 2  x  1   x  1 x  1 log x (iii) ax = 1+ (x n a) +
= lim
x 1  x  1 x  1

62
MATHS-X II-IIT-JEE LIMIT
(x  n a ) 2 (x  n a)3 (x n a) n = 1/6
+ +.. +…
2! 3! n! Example-10

x3 x5 2x – sin -1 x
(iv) sin x = x – + – …… Evaluate lim .
3! 5! x  0 2x + tan -1 x

x2 x4 Solution:
(v) cos x = 1 – + – …..
2! 4!  x3 
2x   x   ... 
 6 1
x3 2 5 lim
(vi) tan x = x + + x + ….. x 0  x 3  3
3 15 2x   x   ... 
 3 
x3 9x 5
(vii) sin–1x = x + + + ….. 6.5. Application of Standard Limits:
3! 5!
(Some standard limits)
–1 x3 x5 x7
(viii) tan x = x –   + ….. sin 1 x
3 5 7 sin x lim x
(i) lim = = lim =1
x 0 x x 0 sin x x 0 x
n n(n  1) 2
(ix) (1 + x) = 1 + nx + x + ….. x
2! lim = 1; lim sin x = 0
x 0 sin 1 x x 0
(for rational or integral n, for irrational n rule is not
 1 
applicable) (ii) lim cos x = lim  = 1
x 0 x 0  cos x 
x2 5 2. x 4 61x6
(x) sec–1x = 1+ + + +...... tan x lim x
2! 4! 6! (iii) lim , = =1,
x 0 x x 0 tan x
  x 3 9x 5 
(xi) cos–1x = –x    ...  tan 1 x x
2  3! 5! 
lim = lim = 1; lim tan x = 0
x 0 x x 0 tan 1 x x 0

 x 11 2  x
(xii) (1 + x)1/x = e 1   x ......  a
 2 24  (iv) lim 1   = lim (1+ax)1/x = ea
x  x x 0

ax
 1
lim (1 + x)a / x = lim 1   = ea
Example-9 x 0 x x  
cosx + e x – 2 – x ax 1
Evaluate lim (v) lim = loge a (a > 0)
x 0 x3 x 0 x
Solution.:
ex  1
(vi) lim =1
 x2 x4   x 2 x3  x 0 x
1   ....   1  x   ....   2  x
 2! 4!   2! 3!  xn  an xn  1
lim n–1 lim
(vii) lim = n a , x 1 x  1 = n
x 0 x3 x a x a
x3 x4  n (1  x ) log a (1  x)
 2.  .... (viii) lim =1, lim = logae
= lim 3! 4! x x
x 0 x 0
x 0 x3
(1  x)n  1
1 x (ix) lim = n,
= lim   ...(higher power of x) x 0 x
x0 3! 12

63
LIMIT MATHS-X II-IIT-JEE
sin x lim cos x si n(2k  3)x 2k  3 2k  3
(x) lim = =0 lim f (x)  lim . 
x x x x x 0 x 0 (2k  3)x 2 2
sin 1 / x 2k + 3 5
(xi) lim =1 Hence 3k + 4 =  k
x 1/ x 2 4
 0 if 0  a 1
 1 if a 1
 Example-12
(xii) lim ax = 
x
  if a 1
does not exist Evaluate lim(1 + ax)b / x
if a0 x0

(xiii) lim [f(x)](x) = (provided lim  is infinite & lim Solution :


x a x a x a
Clearly given limit is in the form of Lim f(x) g(x)
[Lim  log f ] x0
f = 1) = e
where Limf(x) =1
a 0 x m  a1x m1  ..... a 0 x a
(xiv) lim = ,
x b0 x n  b1x n 1  ...... b0 b
Lim (1  ax 1)
Hence Lim (1+ ax) b/ x  e x 0 x
for m = n x 0

= 0, when m < n Lim (1+ ax) b/ x  eab


x0
=  when m > n and a0b0 > 0
= – when m > n and a0b0 < 0 Example-13
1/n

Evaluate lim 2 n + 3 n + 4 n
x 

Example-11
1/ n

 sin(2k + 3)x x



Solution.: Given lim 2 n  3n  4 n  ( form)
 x<0 1/n
2x   2 n  3 n 
 = lim 4        1
If f(x) =  k + 1 x = 0 and n    4   4 
 tan 3k + 4 x  
    n n 
x>0 2 3
 x = 4  as lim    0, lim    0 
 n   4  n   4  
 
lim f(x) exist, then the value of k.
x0 Example-14
Key concept Since lim f(x) exist. That means a x + bx
x0 Evaluate lim , where a > b > 1
x a x – b x
x
lim f(x) = lim– f(x) b
x0+ x0 1  
 a   1 as lim (b / a) x  0
Solution: Solution: lim
x 
b
x x 
 
tan(3k  4)x 1  
lim f (x)  lim a
x 0 x 0 x
tan(3k  4)x Example-15
 lim (3k  4)
x 0 (3k  4)x
1 + x1/ 3
Find lim .
 tan  3k  4  x  x  –1 1 + x1/5
 3k  4  as Lim+ 
 x 0 x
  x1/3  (1) 5
Solution.: Limit = lim 
x 1 x1/5  ( 1) 3

64
MATHS-X II-IIT-JEE LIMIT

(x1/5 )5  (1) a a log a  a a a 1 log a  1


Note : lim  5(1)4   1
x 1 x1/5  (1) a a  a a log a log a  1

(x1/3 )3  (1) It is satisfied only when a = 1.


lim = 3 (–1)2
x 1 x1/3  (1)

Example-18
Evaluate lim tanx.logesinx .
xπ/2
Example-16 Solution:
lnx - 1 log e sin x  0
Evaluate lim . Above limit = lim from : 
x e x – e x /2 cot x  0
ln x / e ln t 1
Solution.: Above limit = lim = lim .cos x
x e x  t 1  t  1 e
= lim sin x [using L' Hospital's Rule]
e   1 x  /2 cosec 2 x
 e 
Putting x/e = t = lim ( sin x.cos x) = 0
x /2
ln 1  y  1
Put t = 1 + y = lim 
y 0 ey e Example-19
 1 1
6.7 L' Hospital's rule: Find Lim  – .
x 0  sinx x
0 
If function takes any of the following form, , ,
0  Solution : If x  0  , then sin x  0  and
then L' hospital's rule is applied 1 1
 
If f (A) = 0 and  (A) = 0 then, sin x x
f (x) f (x) Similarly, if x  0  , sin x  0 
lim = lim
x a  (x) x a  (x) 1 1
and     (  )    
If lim f(x) =  and lim (x) =  sin x x
x a x a Neither form reveals what happens in the limit. To find
f (x) f (x)
then lim = lim out, we first combine the fractions.
x a  (x) x a  (x)
1 1 x  sin x
Note.:   
sin x x x sin x

L' hospital's rule can be repeated required number (Common denominator is x sin x)
and then apply L' Hospital's Rule to the result.
of times in same question.
 1 1 x  sin x 0
Example-17 Lim     Lim
x  0  sin x x  x  0 x sin x 0
a x – xa 1  cos x 0
If lim = – 1 then find the value of a. = Lim s till
x a x x – aa x0 sin x  x cos x 0
Solution: sin x 0
= Lim  0
Since the given limit is in the form of 0/0, we will use
x 0 2 cos x  x sin x 2

L’ Hospital’s Rule
Example-20
a x  xa a x log a  ax a 1 1
lim x = lim  1
x a x  a a x a x x  x x log x Show that Lim(1 + x) x = e .
e +
x 0

65
LIMIT MATHS-X II-IIT-JEE

Solution :The limit leads to the indeterminate form 1 7. SOME LIMITS WHICH DO NOT EXIST
1
1
.We let f(x) = (1  x) x and find n f (x) . (i) lim  
x 0  x 
1
1
Since n f (x) =  n (1  x ) x =  n(1  x) (ii) lim x1/x
x x 0
ˆ
L' Hopital's Rule now applies to give |x|
(iii) lim
 n(1  x) 0 x 0 x
Lim  n f (x)  Lim
x 0 x 0 x 0 |xa|
1 (iv) lim
x a xa
= Lim 1  x  1 (v) lim sin 1/x
x 0 1 x 0
1
Therefore Lim(1  x) x  Lim f (x) (vi) lim cos 1/x

x 0 x 0
x 0
n f (x)
 Lim e 1
e e (vii) lim e1/x
x 0 x 0

(viii) lim sin x


x

(ix) lim cos x


x

66
MATHS-X II-IIT-JEE LIMIT

EXERCISE # 1
Based On Existence of limit x4  x2  1
8. lim =
x  x 5  x 2  1
|x|
1. The value of lim is-
x 0 x (A) 0 (B) 1
(A) 1 (B) 2 (C) 2 (D) Does not exist
(C) 3 (D) Does not exist
3x 5  x 2  13
2 9. lim =
x  1 , x  1 x  x 4  7x 2  17
2. If f(x) =  then the value of
3x  1 , x  1 (A) 0 (B) 2 (C) infinite (D) None
lim f(x) is-
x 1
Based On Factorization
(A) 1 (B) 2
(C) 3 (D) Does not exist x 3  2x  1
10. lim =
x 1 x 5  2x  1
3. Lim (1 – x + [x – 1] + [1 – x]) where [x] denotes
x 1 (A) 2/3 (B) 1/3 (C) 4/3 (D) 5/3
greatest integer but not greater than x
(A) 1 (B) –1 2sin 2 x  sin x  1
(C) 0 (D) Does not exist 11. lim =
x
 2sin 2 x  3sin x  1
6
1
4. If f (x)  3  then- (A) 0 (B) 3 (C) –3 (D) 1
1  71/(1 x)
(A) Lim f(x) = 3 x 3  7x 2  15x  9
x 1 12. lim =
x  3 x 4  5x 3  27x  27
(B) Lim f(x) = 4
x 1 2 2 1
(A) (B) (C) (D) 1
3 9 9
(C) Lim f(x) = 4
x 1
Based On Rationalization
(D) Lim f(x) does not exist
x 1
13. lim
x 
 x2 1  x2 1 = 
Based On Substitution (A) 0 (B) 1 (C) 2 (D) None
1
5. lim[cos (cos x)] , where [] denotes greatest
x 1 (1  x 2 )  (1  x)
14. lim =
integer function x 0
(1  x 3 )  (1  x)
(A) 0 (B) 1
(A) 0 (B) 1 (C) 2 (D) 4
(C) Does not exist (D) None of these

cosθ  sinθ Based On Expansion of function


6. lim =

π π 2x  sin 1 x
4 θ  15. lim =
4 x  0 2x  tan 1 x

(A) 2 (B) 1 1
(A) 3 (B) (C) 0 (D) 1
(C) 2 (D) Does not exist 3

x3  x 2  1 1 1
7. lim = 16. lim  log (1 + x) =
x  x 3  x 2  1 x0x x2
(A) 0 (B) 1 1
(C) 2 (D) Does not exist (A) 1 (B) (C) 0 (D) 2
2

67
LIMIT MATHS-X II-IIT-JEE

Based On Application of standard limits Based On Sandwich Theorem

17. lim
log e {1  tan(x  a)}
= x2  x  1
27. lim  (where [x] is greatest integer
x a tan(x  a) x  e[x]
(A) 0 (B) 1 (C) 2 (D) 3 function  x)
(A) 0 (B) 1
cos( x / 2) (C) 2 (D) Does not exist
18. lim =
x 1 1 x
(A) 0 (B)  (C) /2 (D) 2 28. lim [x]  [2x]  [3x]  .....  [nx] =
n 1  2  3  ......  n
x (where [·] denotes the greatest integer function)
 x 
19. lim  =
x 
x x
x   1  (A) 0 (B) (C) (D) x
2 6
1
(A) e (B) (C) 0 (D) 1
e Based On Miscellaneous question

20. lim (1  x)1/ (13x) = 29. lim [Max (sin x, cos x)]
x 0 
x
4
1
(A) e13 1
(B) e13 (C) e (D) 1 (A) (B) 0 (C) – 1 (D) 1/ 2
2
1 x
 1  x  1 x 2 30. Lim (cosec x)1/log x =
21. lim  = x 0

x 1  2  x 
(A) e (B) e–1 (C) e2 (D) 1
(A) (B)1/3 (B) (2/3) 1/2
31. Which of the following statement is/are
(C) (2/3)1/4 (D) Does not exist
correct -
 tan x 
1/ x 3
(A) Lim  [sgn sin x] = 1
22. lim   = x
x 0  x 
(B) Lim [sgn sin x]  –1
(A) 0 (B) 1 x  
(C) e (D) Does not exist (C) Lim [sgn sin x] = 1
x x
tan
23. Lim (2  x) 2 = (D) Lim [sgn sin x] does not exist
x 1 x
(A) e–2/ (B) e1/ (C) e2/ (D) e–1/ (Where [.] represent greatest integer function)

L Hospital’s Rule 32. The sum to infinity of the series :


Based On
3 5 7
+ 3 + 3 + ...... is-
24. Lim sin x  log(1
2
 x)
= 13
1 2 3
1  23  33
x 0 x
(A) 0 (B) 1/2 (A) 3 (B) 4 (C) 5 (D) 6
(C) –1/2 (D) does not exist 33. Let
    2 2 
(1  cos 2x)sin 5x fk() =  cos 2  i sin 2   cos 2  i sin 2 
25. Lim =  k k  k k 
x 0 x 2 sin 3x
....
(A) 10/3 (B) 3/10 (C) 6/5 (D) 5/6
  
26. Let f(A) = g(A) = k and their nth derivatives exist  cos k  i sin k  then Lim
n 
fn() =
 
and are not equal for some n.  
Further if (A) cos + i sin
2 2
f (a)g(x)  f (a)  g(a)f (x)  g(a)
lim 4 (B) cos + i sin
x a g(x)  f (x) (C) i cos + sin
then k is equal to-  
(A) 0 (B) 4 (C) 2 (D) 1 (D) i cos + sin
2 2

68
MATHS-X II-IIT-JEE LIMIT
42. Inscribed in a circle of radius R is a square, a
  cos –1 x
34. lim  = circle is inscribed in the square, a new square in
x 1 x 1 the circle, and so on for n times. Find the limit of
1 the sum of areas of all the circles and the limit of
(A)  (B) (C) 2 (D) 
2 the sum of areas of all the squares as n 
(A) 2R2, R2 (B) R2, 4R2
 1 n2 
35. If lim  an   = b, a finite number then (C) 2R2, 4R2 (D) 4R2, 2R2
n  1  n 

the 43. lim , x sin(x  [x ]) where [·] denotes the greatest
ordered pair (a, b) is-
x 1 x 1
(A) (1, 1) (B) (–1, 1) integer function, is equal to-
(C) (1, –1) (D) None of these (A) 1 (B) –1
(C)  (D) does not exist
x  2a  x  2a
36. lim  ,a>0= 1  sin 2x
x  2a
x 2  4a 2 44. The value of Lim =
x/4   4x
(A) 2a (B) 2 a (C) 1 / 2 a (D) a 1 1 1
(A) – (B) (C) (D) None
4 4 2
esin x  1  sin x
37. Lim = 45. Evaluate :
x 0 x2 2 3 2n 1

(A) 1 (B)
1
(C) e1/2 (D) e
Lim [x]  [x ]  [x ]  ......  [x ]  n  1
2
x 0 1  [x] | x | 2x
(A) 1 (B) 0
n k cos n! (C) 2 (D) None of these
38. Lim ; 0<k<1
n  n 1
(A) 0 (B) 1! 46.
x 

Lim cos x  1  cos x = 
(C) 2! (D) None of these
(A) 0 (B) 1
(C) 2 (C) None of these.
39. If Sn = a1 + a2 +........ an and Lim an = a, then
n 
Sn 1  Sn (cos ) x  (sin ) x  1
Lim is equal to- 47. lim , x  (0, /2)
n  n x 2 x2
k (A) sin2 n (sin )
k 1
(B) cos2  n (cos )
(A) 0 (B) a (C) 2a (D) 2a
(C) cos2  n (cos ) – sin2 n (sin )
 1
40. The sum  is equal to- (D) cos2  n (cos ) + sin2 n (sin )
n 1 n (n  1) (n  2)
1 1 1 1 1 1
(A) 1 (B) (C) (D) [x]  [2x]  [3x]  ...  [nx]
2 4 8 48. lim 2 3 n =
2 2 2 2
n 1  2  3  ...  n
41. The continued product of (where [·] denotes the greatest integer function)
 1  1  1  1  1 1
(A) 0 (B) (C) (D) 1
 1  4   1  9   1  16  ...........  1  2  is Pn ; 2 6
     n 
then Lim Pn is- (where n N) 49. lim x2  x  1 – x2 1 =
n 
x
1 n 1 1
(A) – (B) (C) (D) None (A)
2
(B) 1 (C)
1
(D) None
2 n 2
3 2

69
LIMIT MATHS-X II-IIT-JEE
50. The value of 58. If lim (1  a sin x)cos ecx  3 , then a is
x 1/ x x 0
 e xn(2 –1)
– (2x – 1) x sin x 
lim   = (A) ln 2 (B) ln 3 (C) ln 4 (D) e3
x 0 
 e xnx 
 x3
sin x  x 
ln 2 (C) e ln 2
1 6 is equal to
(A) e (B) (D) None 59. lim
e x 0 x5
1 1 1
51. Let f(x) = Lim {sinx + 2sin2x + 3sin3x +... (A) (B) (C) (D) 0
n  120 160 2
….+ nsinn x} If sinx  n+ /2, n  : a n  bn
1 60. lim , where a > b > 1, is equal to
2 n  a n  b n
Evaluate : Lim  1  sin x  f (x)  sin x 1
x  / 2   (A) – 1 (B) 1 (C) 0 (D) a / b
(A) 1 (B) 0
5
(C) e (D) None of these 1  x  1
61. lim is equal to
x 0 3x  5x 2
52. Lim cos (sin x)4  cos x = 3 5
x 0 x (A) (B) (C) 0 (D) 1
1 1 5 3
(A) 1 (B) 6 (C) – (D)
6 6 cot x  cos x
62. lim is equal to
53. 2
Let a = min{x + 2x + 3, x  R} and x  / 2 (  2x)3
1  cos  n  1
(A) 1 (B) (C) (D) 0
b = lim
0 2
. Then  a r . bn r = 2 16
r 0
(A) 2n+1 + 1 (B) 2n+1 – 1  4 1 2
4 n 1  1 4 n 1  1  x sin  x   x 
(C) (D) Lim   
3.2 n 3.2 n
63.  is
x 
 1 | x |3 
 
 
54. Lim– a 2  x 2 cot   a  x  is- (A) 2 (B) 1
x a 2 ax
  (C) –1 (D) does not exist.
a 2a a 4a
(A) (B) (C) – (D)
    f (x)  3
64. If f (9) = 9, f (9)  4 , then Lim .
x 9 x 3
 1 n  equals
55. lim nn  e.  1  1   = (A) 2 (B) 4 (C) 6 (D) 0
n  n 
   
3 2 x   5
(A) 1 (B) (C) (D) None 65. If lim = 500, then positive values of  is
2 3 x 5 x  5
(A) 3 (B) 4 (C) 5 (D) 6
56. The value of
sin x, x  n, n  0,  1,  2........
1  1  66. If f(x) =  and
Lim 1  x     2, otherwise
x 0  2   tan x  4  2 
 x 2  1, x  0, 2
is-
(A) loga16 (B) Does not exist g(x) =  4, x  0 , then Limg{f (x)} is
x 0
(C) 3 ln 2
 5, x2
(D) 4 log 2 
(A) 2 (B) 0 (C) 3 (D) 1
n x  [x]
57. lim = ([.] G. I. F.) 67. Lt
| x  3|
=
x [x] x3 x3
(A) 0 (B) –1 (A) 1 (B) –1
(C)  (D) None of these (C) does not exist (D)  1

70
MATHS-X II-IIT-JEE LIMIT
      x b
68. lim n.cos   .sin   is equal to 77. The value of lim
x0 a   is, .  (G.I.F.)
n   4n   4n  x
    (A) 0 (B) 
(A) (B) (C) (D)
4 6 9 3 b
(C) (D) does not exist
a
1  cos3x
69. Lt  G (x )  G (1)
x 0

x 3x  1  78. If G(x) = – 25  x 2 then lim has
x 1 x 1
(A) 9/2 (B) 9/(2 log3) the value
9 log 3 1 1 1
(C) (D) 1 (A) (B) (C) – 24 (D)
2 24 5 2 6
9 2
70. If f(x) is continuous and f    , then
 2 9  sin[x]
 , [x]  0
 1  cos3x  79. If f(x) =  [x] , where [x] denotes the
lim f   is equal to  0, [x]  0
x 0  x2  
(A) 9/2 (B) 2/9 (C) 0 (D) - 9/2 greatest integer less than or equal to x then
Limf (x) equals
 4x  3, x  1 x 0
71. f (x)   2 , then Lt f (x) = (A) 1 (B) 0
 x , x1 x 1
(C) -1 (D) does not exist
(A) 1 (B) –1 (C) 0 (D) 0
x
2.3n  3.5 n x 
tan
72. Lt =  2
n  3.3n  45n 80. The value of Lim  tan  is
x 1  4 
(A) 2/3 (B) –3/4 (C) 1 (D) 0
(A) e–2 (B) e–1 (C) e (D) 1
2 2
sin   sin 
73. lim is equal to 81. The value of
  2  2 a/x
 1x  2 x  3x  ...  n x 
sin 3 sin 2 L im   is
(A) (B) x 0 n
 4   2   
a 2a
sin 8 sin 6
(C) (D) (A) (2n !) n (B) (n !) n
 7   4  a

(2  x) (4  x)40 5 (C) (n!) n (D) 0


74. lim
x  (2  x)45
82. If  is a repeated root of ax2 + bx + c = 0 then
(A) -1 (B) 1 (C) 16 (D) 32
sin (ax 2  bx  c)
is
x  sin x (x  )2
75. lim =
x  x  cos x (A) 0 (B) a (C) b (D) c
(A) 0 (B) 1
(C) -1 (D) does not exist
x2 x  2
76. lim is equal to
x 2
x2  4
1
(A) (B) 1 (C) 2 (D) 0
2

71
LIMIT MATHS-X II-IIT-JEE

EXERCISE # 2
One or More Than One Correct Answer a cot x  a cos x
Question 5. For a > 0, let  = lim and
Type Questions x  cot x  cos x

2
1. Which of the following limits tends to unity?
(A) lim
sin(tan t) m  lim
x–
 x  ax  x 2  ax
2
 then-
t 0 sin t (A)  is always greater than ‘m’ for all values of
sin(cos x)
(B) lim a>0
x/2 cos x
(B)  is always greater than ‘m’ only when a  1
1 x  1 x
(C) lim (C)  is always greater than ‘m’ for all values of
x 0 x
‘a’ satisfying a > e–a
x2 (D) e + m = 0
(D) lim
x0 x
x
 ax  1  6. If f(x) = sinx + cosx, [x] is the greatest integer
2. Consider the function f (x)    where
 bx  2  function, then
a2 + b2  0 and a > 0 & b > 0 then lim f (x) (A) lim– [f(x)] = 0
x  x 
(A) exists for all values of a and b (B) lim [f(x)] = 1 (n I)
(B) is zero for 0 < a < b x  (2n / 2) —
(C) is non existent for a > b (C) lim [f(x)] = 0, (n I)
1 1 x 2n
   
(D) is e a or e b if a = b (D) Range of f(x) is {–2, –1, 0, 1}
3. lim f(x) does not exist when-
x c x ai
(A) f(x) = [ [x] ] – [2x –1], c = 3 7. If Ai = , i = 1, 2, 3,....n and
(B) f(x) = [x] – x, c = 1
| x ai |
(C) f(x) = {x}2 – {–x}2, c = 0 if a1 < a2 < a3 < ...< an and 1 < m < n then -
tan(sgn x) (A) lim (A1 A2... An) = (–1)n – m + 1
(D) f (x)  ,c=0 x a m
sgn x
Where [.] denotes greatest integer function & (B) lim (A1 A2... An) = (–1)n–m
x a m
{x} fractional part function.
(C) lim– (A1 A2... An) = (–1)n – m + 1
x a m
4. Identify the true statement(s).
(D) lim– (A1 A2... An) = (–1)n – m
n 1 x a m
(A) lim   r   1 , where [.] denotes the
n 
 r 1 2 
8. If f(x) = |x – 1|– [x], where [x] is the greatest
greatest integer function integer less than or equal to x, then-
(B) If f(x) = (x –1) {x}, then limit of f(x) does (A) f(1 + 0) = –1, f(1 – 0) = 0
not exist at all integers except {1} (B) f(1 + 0) = 0 = f(1 – 0)
 tan x  (C) lim f(x) exists
(C) lim    1 , where [.] denotes the x 1
x 0  x 
(D) lim f(x) does not exist
greatest integer function. x 1

 tan x  3
(D)  lim  1 , where [.] denote the 9. For lim  cos x 1/ x
 x  0 x  x 0

greatest integer function (A) LHL =  (B) RHL = 0


(C) Limit = 0 (D) limit does not exist

72
MATHS-X II-IIT-JEE LIMIT

 2x Question Passage Based Type Questions


1  , 0  x 1,
10. Let f (x)   a . If limf (x) Passage # 1 (17 to 19)
x 1
 ax, 1 x  2 Let there are two functions defined here.

exists then a is sin x x  n
(A) 1 (B) –1 (C) 2 (D) –2 f(x) = 
 2 x  n
11. If f(x) = |x – 1| – [x], where [x] = the greatest
integer less than or equal to x, then  x 2  1 x  0, x  2

(A) f(1+) = –1, f(1+) = 0 g(x) =  4 x 0
(B) f(1+) = 0 = f(1–)  6 x2

(C) limf (x) exists
x 1

(D) limf (x) does not exist 17. Find lim g(f(x))-
x 1 x 0

 1  n2  (A) 0 (B) 1 (C) –1 (D) None


12. If lim  an    b, a finite number, then
n 
 1  n 
(A) a = 1 (B) a = 0 (C) b = 1 (D) b = –1 18. Evaluate lim f (f(x))-
x 1

x p  x p 1  1 (A) sin 1 (B) sin sin1


13. lim , where p > 0, q > 0 is
x  x q  x q  2  2 (C) 1 (D) None of these
(A) 0 if p < q
(B) 1 if p = q n
(C) infinite, if p > q 19. Evaluate  xlim
i
{g(x) – 1} , where {x} is the

(D) 1 if p > q i 1

fractional part of real x-


a sin x  be x  cos x  2
14. If lim  6 , then (A) 0 (B) n, n N
x 0 x2
(C) 1 (D) None of these.
(A) a = – 7 (B) a + b = 0
(C) b + c = 2 (D) b – c = 12. Passage # 2 (20 to 22)
15. If lim f  x   lim f  x  ([.] denotes the f(x) is polynomial function of degree six. Let consider
x a xa
f(x) : a0 + a1x + a2x2 +.....+ a6x6. Such that f(A) = 2,
greatest integer function) and f(x) is non-
1/x
constant continuous function, then  f (x) 
f(–1) = 0 and satisfying lim 1  3  = e2.
(A) lim f  x  is an integer x 0  x 
x a
Another function
(B) lim f  x  is non-integer
x a
x m A(x)  B(x)  1
(C) f(x) has local maximum at x = a g(x) = lim and also satisfy the
m 2x m  3x  3
(D) f(x) has local minimum at x = a
1/x
xy  f (x) 
16. Let f(x, y) = , x + y  1, then condition lim g(x) = lim 1  3 
xy
x 1 x 0  x 
(A) lim lim f  x, y   1 20. Find the value of a2 and a3
x 0 y 0
(A) 0, 1 (B) 1, 0 (C) 0, 0 (D) None
(B) lim lim f  x, y   1
x 0 y0
21. Find the lim f(x)
(C) lim lim f  x, y   1 x 1
y0 x  0
(A) 2 (B) 3 (C) 4 (D) None
(D) lim lim f  x, y   1
y 0 x  0

73
LIMIT MATHS-X II-IIT-JEE
22. Find the value of B(A) 26. lim (PA) is equal to-
(A) 6e2 (B) 6e2 + 1 (C) 6e2 – 1 (D) None 0

Passage # 3 (23 to 24) (A) 1/3 (B) 1 (C) 2 (D) 3


x – [x] ; x  I PC 
Let ƒ(x) =  ; where I is the set of 27. lim 2   is equal to-
 1 ; xI 0  BC 
integers & [x] represents greatest integer  x. (A) 1/3 (B) 1 (C) 3 (D) 6
(f (x))2n – 1 Passage # 5 (28 to 30)
If g(x) = lim , then :
n (f (x)) 2n  1
If lim f(x) = 1 and Lim (x)   , then,
x a xa
23. Period of f(2x) is- lim  f ( x ) 1. ( x)
(A) not defined (B) 1 lim f (x) ( x)  lim [1 + (f(x) – 1)]  (x) = e xa
x a x a
(C) 1/2 (D) 2
Note .: 
24. ƒ(x) = |g(x)| is satisfied by- x
(A) no real x  1
lim 1    lim (1  x)1/x  e ,
(B) all integer values of x x   x x 0
(C) x = 0 only x ax
(D) x = 1 only  a  1
lim 1    lim (1  x)a/ x  lim 1    ea
x   x x  0 x 0  x
Passage # 4 (25 to 27)
A tangent line is drawn to a circle of radius unity at
1 cos(x 1)
point A and a segment AB is laid off whose length is
 x 3  2x 2  x  1  (x 1)2
equal to that of the arc AC. A straight line BC is 28. The value of lim   is
2
drawn to intersect the extension of diameter AO at x 1 
 x  2x  3 
point P as shown in figure. 6 5 3
(A) (B) (C) (D) 1
C B 5 6 2

x3
 2
 3x  1  x
1
P O A 29. The value of lim  2  is
x  4x  1 
 
(A) 0 (B) 1 (C) –1 (D) e
25. The length PA in terms of '' is given by-
x
4 (1  cos  )  2 sin  tan
(A) (B)  x 2a
 sin  (   tan ) 30. lim  2   is equal to
x a  a
2(   sin )  (1  cos  )
(C) (D) (A) e2/  (B) e / 2 (C) e  (D) 1
(   tan )   sin 

74
MATHS-X II-IIT-JEE LIMIT

EXERCISE # 3
Question Column Match Type Questions 4. Column I Column II
(Limits) (Values)
Match the entry in Column 1 with the entry 1/ nx
in Column 2. (A) lim  cosec x  (p) 1
x 0
1. Column 1 Column 2
(B) lim  sec x cot x (q) e
(A) lim [sin–1(sin x)] (P) –2 x

 2
x
2 1/ x 2
–1  tan x  1
(B) lim [tan x] (Q) 0 (C) lim   (r)
x– x 0  x  e
1 1
1  sin 2x (D) lim  cos x  x 2 (s)
(C) lim (R) 1 x0 e
x
   4x
4 3
(t) e
 sin | x | 
(D) lim  (S) does not exist
x0  x  5. Column I Column II
(Limits) (Values)
2. Find (a, b, c) if
(1  x 4 )  (1  x 2 ) 1
Column 1 (A) lim (p)
a sin x  bx  cx  x 2 3 x x2 3
(A) lim = Finite 2x  5  3 1
x 0 2x 2 n(1  x)  2x 3  x 4 (B) lim . (q)
x 2 x2 2
sin x  ae x  be x  cn(1  x)
(B) lim = Finite x2  1  1
x 0 x3 (C) lim . (r) 0
axe x  b log(1  x)  c x e  x
x 0
x2  9  3
(C) lim 2
x 0 x 2 sin x (D) lim
x 
 x 1  x  x (s) 2
Column 2
(t) 3
(P) (3, 12, 9)
(Q) (–4, 3, any real no.) Question Numerical Type Questions
(R) (6, 6, 0)
6. If [x] denotes the integral part of x, find f(x
 1 1 
(S)   , ,0  12 x    22 x   ...  n 2 x 
 2 2 
lim    
3
  , then f(6) is
3. Column 1
n  n
n equal to
2   1  1   1 
(A) nlim n –n (n  1)  n   n  2  ...  n  n 1  

  2  2   2  x 1  a cos x   bsin x
2
x 7. If lim  1 , then |a + b| is
 x  4x  3  x 0 x3
(B) lim  2 
x    x  2x  5  equal to
 
1/ x 2
 sin x 
(C) lim 
x 0  x 

8. Let L = Lt
sin x  ln  (1  sin 2 x )  sin x 
3
x x 0 sin x
 1 1
(D) lim  sin  cos  then find the value of 6L + 5.
x  x x
Column 2
(P) e–1/6
(Q) e
(R) e6
(S) e2

75
LIMIT MATHS-X II-IIT-JEE

EXERCISE # 4
Question Previous Year (JEE Mains) 6. Let f : R  R be a differentiable function
satisfying f '(C) + f '(B) = 0. Then
1. For each t  R, let [t] be the greatest integer 1/ x
less than or equal to t. Then [JEE MAIN 2018]  1  f (3  x)  f (3) 
lim   is equal to
1  2 x 0 1  f (2  x)  f (2)
15    
lim x        .....    
x 0 x  x   x  [April 2019]
(A) e2 (B) e (C) e–1 (D) 1
(A) is equal to 120
(B) does not exist (in R) 7. If f : R  R is a differentiable function and
(C) is equal to 0 f (x)
2t dt
(D) is equal to 15 f(B) =6, then lim  is [April 2019]
x 2
6
(x  2)
4 (A) 0 (B) 2f ' (B)
1 1 y  2
2. lim [January 2019] (C) 12f '(B) (D) 24 f '(B)
y0 y4
x4  1 x3  k3
1 8. If lim  lim 2 , then k is –
(A) exists and equals x 1 x  1 x k x  k 2
4 2 [April-2019]
(B) does not exist (A) 3/8 (B) 3/2 (C) 4/3 (D) 8/3
1
(C) exists and equals 1 1 1
2 2  
9. A  2 b c  is equal to [April-2019]
1
(D) exists and equals 4 b2 c2 
2 2 ( 2  1) 
4 3 4
3. For each x R, let [x] be the greatest integer (A) (2) 4/3 (B) (2) 4 /3 
3 4 3
less than or equal to x.
3 3 4
x ([x] | x |) sin [x] (C) (2) 4 /3  (D) (2) 3/ 4
Then lim is equal to 4 4 3
x 0 |x|
[January 2019] x 2  ax  b
10. If lim  5, then a + b is equal to :
(A) – sin 1 (B) 0 x 1 x 1
(C) 1 (D) sin 1 [April-2019]
(A) –7 (B) – 4 (C) 5 (D) 1
4. For each t R, let [t] be the greatest integer x  2sin x
less than or equal to t. Then, 11. lim is
x 0
x 2  2sin x  1  sin 2 x  x  1
  [April-2019]
(1 | x |  sin |1  x |) sin  [1  x] 
 2  (A) 3 (B) 2 (C) 6 (D) 1
lim
x 11 |1  x | [1  x]
12. Let f (x) = 5 – | x – 2 | and g (x) = | x + 1 |, x 
[January 2019] R. If f (x) attains maximum value at  and g (x)
(A) equals –1 (B) equals 1 attains minimum value at , then
(C) does not exist (D) equals 0
(x  1) (x 2  5x  6)
lim is equal to :
x  x 2  6x  8
[April-2019]
sin 2 x (A) 1/2 (B) –3/2 (C) 3/2 (D) –1/2
5. lim equals :
x 0 2  1  cos x
1/x 2
[April 2019]  3x 2  2 
13. lim  2  is equal to – [Jan. 2020]
(A) 2 2 (B) 4 2 x 0  7x  2 
 
(C) 2 (D) 4 (A) e–2 (B) e2 (C) e2/7 (D) e3/7

76
MATHS-X II-IIT-JEE LIMIT

3x  33 x  12 22. If  is the positive root of the equation,


14. lim is equal to _____ p (x) = x2 – x – 2 = 0, then
x2 3 x /2  31 x
[Jan. 2020] 1  cos (p (x))
lim
x x  x 4
 t sin (10t) dt is equal to [Sep 2020]
15. 0 is equal to – [Jan. 2020]
lim (A) 3 / 2 (B) 3/2
x 0 x
(A) 1 (B) 10 (C) 5 (D) 0 (C) 1/ 2 (D) 1 / 2
 (x 1)2   1  x2 x2 x2 x 2  
 t cos (t ) dt 
2 23. If lim  8 1  cos  cos  cos cos   =
   x  0  x 
 2 4 2 4  
16. lim  0  [Sep. 2020]
2–k, then the value of k is _______ .
x 1  (x  1) sin (x  1) 
  [Sep. 2020]
 
 
x  x 2  x 3  ....  x n  n
(A) does not exist (B) is equal to 1/2 24. If lim  820,(n  N)
(C) is equal to 0 (D) is equal to –1/2 x 1 x 1
then the value of n is equal to_______.
1 x 2  x 4 )/x [Sep.2020]
x (e (  1)
17. lim [Sep. 2020]
 9 x 
x 0 1  x2  x4 1 25. lim    is equal
x  2  n 1 n (n  1)x 2  2 (2n  1) x  4 
(A) does not exist. (B) is equal to e  
to : [Aug. 2021]
(C) is equal to 0. (D) is equal to 1.
(A) 7/36 (B) 1/5
18. Let f : (0, )  (0, ) be a differentiable (C) 5/24 (D) 9/44
function such that f (A) = e and
26. If ,  are the distinct roots of x2 + bx + c = 0,
t 2 f 2 (x)  x 2 f 2 (t) then [JEE MAIN 2021]
lim 0
tx tx 2  bx  c)
If f (x) = 1, then x is equal to : [Sep. 2020] e2 (x  1  2 (x 2  bx  c)
lim is equal to :
(A) 2e (B) 1 / 2e (C) e (D) 1/e x  (x  )2
1/x (A) b2 – 4c (B) b2 + 4c
   (C) 2 (b2 + 4c) (D) 2(b2 – 4c)
19. lim  tan   x   is equal to –
x 0  4 
27. If lim ( x 2  x  1  ax)  b, then the ordered
[Sep. 2020] x 
(A) 2 (B) e (C) 1 (D) e2 pair (a, b) is – [JEE MAIN 2021]
(A) (1, 1/2) (B) (–1, –1/2)
(a  2x)1/3  (3x)1/3 (C) (–1, 1/2) (D) (1, –1/2)
20. lim (a  0) is equal to :
x a (3a  x)1/3  (4x)1/3
sin 2 (  cos 4 x)
[Sep. 2020] 28. lim is equal to :
1/3 4/3
x 0 x4
22 2 [JEE MIAN 2021]
(A)     (B)  
39 3 (A) 4 (B) 2 (C) 42 (D) 22
4/3 1/3
2  2  2  tan 3 x  tan x
(C)   (D)    29. If   lim and
9  9  3  x  /4  
cos  x  
 4
21. Let [t] denote the greatest integer  t. If for   lim (cos x)cot x
1  x | x | x 0
some  R – {0, 1}, lim  L, are the roots of the equation, ax2 + bx – 4 = 0,
x0   x  [x] then the ordered pair (a, b) is[JEE MAIN 2021]
then L is equal to : [Sep. 2020] (A) (–1, –3) (B) (–1, 3)
(A) 1 (B) 2 (C) 1/2 (D) 0 (C) (1, 3) (D) (1, –3)

77
LIMIT MATHS-X II-IIT-JEE
30. Let f (x) = x6 + 2x4 + x3 + 2x + 3, x  R. Then (A) 

(B) 

(C) 

(D) 

the natural number n for which 4 3 6 2
x n f (1)  f (x)  x2  x  1 
lim  44 is _______. 38. If lim   ax  b  = 4, then
x 1 
x 1 x 
 x 1 
[JEE MAIN 2021] [IIT 2012]
(A) a = 1, b = 4 (B) a = 1, b = – 4
Question Previous Year (JEE Advanced)
(C) a = 2, b = –3 (D) a = 2, b = 3
sin(  cos2 x) 39. Let  (A) and (A) be the roots of the equation
31. Lim = [IIT 2001]
x 0 x2
(A) – (B)  (C)

(D) 1  3

1  a 1 x2    
1 a 1 x  6

1 a 1  0
2
where a > – 1.
a tan x  a sin x
32. Evaluate : Lim a>0 Then lim (A) and lim (A) are [IIT 2012]
x  0 tan x  sin x a 0 a 0
[REE 2001]
5 1
(A) – and 1 (B) – and – 1
33. The value of Integer n ; for which 2 2
(cos x  1)(cos x  e x ) (C) –
7
and 2 (D) –
9
and 3
Lim is a finite non zero
x 0 xn 2 2
number- [IIT 2002] 40. The largest value of the non-negative integer a
(A) 1 (B) 2 (C) 3 (D) 4
for which
1 x
(sin nx)  (a – n)nx – tan x 
34. If lim  0 then the  ax  sin  x  1  a 1 x 1
x 0 x2 lim    is
 x  sin  x  1  1  4
x 1
value of a is- [IIT 2002]
1 n 1 [IIT 2014]
(A) (B) (C) n+ (D) n
n 1 n 1 n 41. Let m and n be not two positive inters greater

1 sin x  e cos( n )  e  e


 x 1  than 1, If lim       then the
35. lim   sin x      , for x > 0 [IIT  0   m  2
x 0   x    

2006] m
value of is [IIT 2015]
n
(A) 0 (B) –1 (C) 2 (D) 1
x 2 sin(x)
x2 42. Let ,   R be such that lim = 1.
a  a2  x2  x  0 x  sin x

36. Let L  lim 4 ,a>0 Then 6( + ) equals [IIT 2016]


x 0 x4
If L is finite, then [IIT 2009] 1  x(1 |1  x |)  1) 
(A) a = 2 (B) a = 1 43. Let f(x) = cos   for
|1  x | 1 x 
1 1
(C) L  (D) L  x ≠ 1. Then [IIT 2017]
64 32
1

(A) lim– f (x) does not exist


37. If lim[1  x ln(1  b 2 )]1/ x = 2b sin2 , b > 0 and x 1
x 0
(B) lim f (x) does not exist
(– , ], then the value of  is - [IIT 2011] x 1

78
MATHS-X II-IIT-JEE LIMIT

(C) lim f (x) = 0   


x 1 46. Let f :   ,   R be a continuous function
 2 2
(D) lim– f (x) = 0 such that
x 1
 /3
44. For any positive integer n, define fn : (0, ) f (0) = 1 and f (t) dt  0

n  1  0
as fn (x) =  tan1   for Then which of the following statements is (are)
j1  1   x  j x  j  1  TRUE?
all x  (0, ) (A) The equation f (x) – 3 cos 3x = 0 has at
Here, the inverse trigonometric function tan–1x least one solution in (0, /3).
   (B) The equation f (x) – 3 sin 3x = –6/has at
assumes values in   ,  .then, which of the
 2 2 least one solution in (0, /3).
following statement(s) is (are) TURE? x
[IIT 2018] x  f (t) dt
n 0
(C) lim  1
(A)  tan 2 (f j (0)) = 55 x 0 2
j1
1  ex
x
10
sin x  f (t) dt
(B)  (1 + f ‘j (0)) sec2(fj(0)) = 10
0
j1 (D) lim  1
(C) For any fixed positive integer n, x 0 x2
lim tan (fn(x)) = 1
x n
(D) For any fixed positive integer n,
lim sec2 (fn(x)) = 1
x

45. Let f : R  R be a function. we say that f has


f (h)  f  0 
PROPERTY 1 if lim exists and is
h 0 |h|
finite, and
f (h)  f  0 
PROPERTY 2 if lim exists and is
h 0 h2
finite. Then which of the following option is/are
correct? [IIT 2019]
(A) f(x) = x2/3 has property 1
(B) f(x) = sin x has property 2
(C) f(x) = |x| has property 1
(D) f(x) = |x| has property 2

79
LIMIT MATHS-X II-IIT-JEE

ANSWER KEY

EXERCISE-1

Qus. 1 2 3 4 5 6 7 8 9 10 11 12 13 14 15 16 17 18 19 20
Ans. D B B D C A B A C B C B A B B B B C B B
Qus. 21 22 23 24 25 26 27 28 29 30 31 32 33 34 35 36 37 38 39 40
Ans. C D C C A B A D D B A,D B A B A C B A A C
Qus. 41 42 43 44 45 46 47 48 49 50 51 52 53 54 55 56 57 58 59 60
Ans. C C D A B A D A C B C D C D B D B B A B
Qus. 61 62 63 64 65 66 67 68 69 70 71 72 73 74 75 76 77 78 79 80
Ans. B C C B B D C A B B A B B A B A C D D B
Qus. 81 82
Ans. C B

EXERCISE-2

Qus. 1 2 3 4 5 6 7 8 9 10 11 12 13 14 15 16 17 18 19 20
Ans. A,B,C B,C,D B,C B,C,D A,B B,C A,D A,B,D B,C A,D A,C A,B,C A,B,C,D A,D B,C B B A C
Qus. 21 22 23 24 25 26 27 28 29 30
Ans. A C C A D D D B A A
EXERCISE-3
1. A – R, B – P, C – S, D – Q 2. A – R, B – S, C – P
3. A –S, B – R, C – P, D – Q 4. A R, B P, C T, D S
5. A R, B P, C R, D Q
6. [2] 7. [4] 8. [6]
EXERCISE-4

Qus. 1 2 3 4 5 6 7 8 9 10 11 12 13 14 15 16 17 18 19 20
Ans. A A A D B D C D C A B A A 36 4 C D D D A
Qus. 21 22 23 24 25 26 27 28 29 30 31 33 34 35 36 37 38 39 40 41
Ans. B A 8 40 D D D C C 7 C C C D A,C D B B 0 2
Qus. 42 43 44 45
Ans. 7 B.D D A,C

32. [log a + 0 = log a] 46. [A,B,C]

80
MATHS-X II-IIT-JEE CONTINUITY

CHAPTER

CONTINUITY
1. Continuity And Discontinuity of a function Examples :- 1
at a point Discuss the continuity of the function [cos x] at x =
 A function f(x) is said to be continuous at x = a, if 
, where [] denotes the greatest integer function.
lim f(a) = lim f(x) = f(a) 2
xa xa

i.e. LHL = RHL = value of the function at ‘a’ i.e. Solution:


lim f(x) = f(a). L.H.L = R.H.L = lim  cos x   1
x a 
x
2
If f(x) is not continuous at x = a, we say that f(x) is
discontinuous at x = a.    
f =  cos 2  = 0 Clearly, L.H.L  R.H.L
Note :  2  
(i) All Polynomials, Trigonometrically functions, 
So, the function is discontinuous at x = .
exponential and Logarithmic functions are 2
continuous in their domain . Examples :- 2
(ii) We never talk about continuity/discontinuity at sin2x + Asinx + Bcosx
point at which we can’t approach from either side If f(x) = is continuous at x = 0.
of the point. These points are called isolated points x3
Find the values of A and B. Also find f(0).
e.g. f(x) = a  x  x  a at x = a. Solution: As f(x) is continuous at x = 0,
(iii) There are some functions which are continuous  lim f (x)  f (0) and both f(0) and lim f(x) are finite.
only at one point. x 0 x 0

 x if x  Q  x if x  Q sin 2x  A sin x  Bcos x


e.g. f(x) =  and g(x) =  f(0) = lim
  x if x  Q  0 if x  Q
x 0 x3
are both continuous only at x = 0 As denominator  0, when x  0.
Reasons of Discontinuity Numerator should also  0, when x  0
which is possible only if
 sin 2(0) + A sin (0) + B cos(0) = 0
B = 0
sin 2x  A sin x
 f(0) = lim
x 0 x3
 sin x  2cos x  A 
 f(0) = lim   
(i) Lim f (x) does not exist. x 0  x  x2 
x a
 2cos x  A 
(ii) f(x) is not defined at x = b. = lim  
x 0  x2 
(iii) Lim f (x)  f (c) .
x c Again we can see that denominator  0 as x  0
In all the above cases, geometrically the graph of Numerator should also approach 0 as x  0
the function will exhibit a break at the point of 2+A=0 A=–2
discussion. The graph as shown is discontinuous at f(0) = – 1
x = a, b and c.
So, we get A = – 2, B = 0 and f(0) = – 1

81
CONTINUITY MATHS-X II-IIT-JEE
Examples :- 3 Examples :- 4
2cosx – 1   π Check the continuity of the function f(x)=[x2] – [x]2
f(x) = x   0,  except at x = .  x  R at the end points of the interval [– 1, 0],
cotx – 1  2  4
(where [] denotes the greatest integer function).
 π π
Define f   so that f(x) may be continuous at x = Sol.: Continuity at x = – 1
4
  4
Solution: f (–1) = [(–1)2] –[–1]2 = [1] – (–1)2 = 1 – 1 = 0

f(x) will be continuous at x 



4
 2

R.H.L = lim   x 2    x  = 0 – 1 = – 1
x 1

 so, f(– 1) R.H.L


if lim f (x)  f  
x  / 4 4 Continuity at x = 0
  2 cos x  1 f(0) = [(0)2] – [0]2 = 0 – 0 = 0
 f   = lim
4
  x  /4 cot x  1 LHL = lim
x 0
 x    x    0  1  1
2 2

( 2 cos x  1) sin x
= lim So, f(0)  L.H.L
x  /4 cos x  sin x
Hence the function is not continuous at the end points of
( 2 cos x  1)( 2 cos x  1) (cos x  sin x) sin x the interval [– 1, 0]
= lim
x  /4 ( 2 cos x 1) (cos x  sin x) (cos x  sin x)
Examples :- 5
= lim
 2cos 2

x 1  cos x  sin x  sin x A function f is defined as follows:
x  /4 (cos2 x  sin 2 x)  2 cos x  1  
sin x(cos x  sin x) 
= lim 1 , when –  < x < 0
x  /4 2 cos x  1  π
1  1 1  f(x) =  1 + sin x , when 0  x <
  2
 
2 2 2 1  π
2
π
=  
1 2 2 + x –  , when x<
2. 1   2 2
2
Discuss the continuity of f.
2. Continuity on an Interval Solution: Continuity at x = 0
 Continuity on an Open Interval L.H.L at x = 0 lim f (x)  lim (1)  1
x 0 x 0
A function f(x) is said to be continuous in an open R.H.L at x = 0 lim f (x)  lim (1  sin x)  1
x 0 x 0
interval (a, b) if it is continuous at each and every
f(0) = 1 + sin 0 = 1
point of (a, b) i.e. y = [x] is continuous in (1, 2)
= L.H.L = R.H.L = f(0) so f(x) is continuous at x = 0.
 Continuity on a Closed Interval 
continuity at x =
A function f(x) is said to be continuous in a closed 2
interval [a, b] if L.H.L at

(A) It is continuous in (a, b) x = = lim f(x) = lim (1+sinx) = 1+1 = 2
2 x  x

(B) Value of the function at “b” is equal to left 2 2

hand limit at “b” i.e., f(b) = lim f (x) R.H.L at


x b 2
  
x= = lim f(x) = 2 +     2
(C) Value of the function at “a” is equal to right 2 x
 2 2
2
hand limit at “a” i.e., f(a) = lim f (x)
x a

82
MATHS-X II-IIT-JEE CONTINUITY

   
2  Lim f(x) = Lim f(x) = f(a)
f  = 2 +    = 2 x 1 x 1
 2 2 2
 Lim a [x + 1] + b [x – 1]
x 1
 
 L.H.L = R.H.L = f   = Lim a [x + 1] + b [x – 1] a – b = 2a + 0b
 2 x 1

  a + b = 0
So, f(x) is continuous at x =  
 2  Continuity of Composite Function
Hence, f(x) is continuous over the whole real number. If f is continuous at x = c and g is continuous at
x = f(c) then the composite g(f(x)) is continuous at
 Algebra of Continuous Functions x sin x
x = c. e.g. f(x) = 2 and g(x) = |x| are
Let f(x) and g(x) are continuous functions at x 2
x = a. Then, continuous at x = 0,
(i) c.f(x) is continuous at x = a where c is any x sin x
Hence the composite (gof ) (x) =
constant x2  2
(ii) f(x) ± g(x) is continuous at x = a will also be continuous at x = 0.
(iii) f(x). g(x) is continuous at x = a
(iv) f(x).g(x) is continuous at x = a, provided g(a)  0 Examples :- 7
1
Find the point(s) of discontinuity of y = ,
Note :  2
u +u–2
(A) If f(x) is continuous and g(x) is discontinuous at 1
x = a then the product function (x)  f (x).g(x) is where u = .
x -1
not necessarily be discontinuous at x = a. Solution :
 1 1
sin x0 The function u = f (x) = is discontinuous at the
e.g. f(x) = x and g(x)   x x 1
 0 x0 point x = 1. ... (i)
(B) If f(x) and g(x) both are discontinuous at x = a then The function
the product function (x)  f (x).g(x) is not 1 1
y = g(x) =  is discontinuous
necessarily be discontinuous at x = a. u  u  2  u  2  u  1
2

1 x0  1 x  0 at u = – 2 and u = 1.
e.g. f(x) =  and g(x) = 
 1 x  0 1 x0 When u = – 2,
1
 2
(C) Continuity of an inverse Function : If the function x 1
1
y = f(x) is defined, continuous and strictly x= u=1
2
monotonic on the interval X, then there exist a
1
 1  x = 2
single valued inverse function x = (y) defined, x 1
continuous and also strictly monotonic in the range Hence, the composite function y = g (f(x)) is
1
of the function y = f(x). discontinuous at three points x = , 1, 2
2

Examples :- 6 3. Continuous function


The function f(x) = a [x + 1] + b [x - 1], where [x] is  A function is said to be continuous function if it is
the greatest integer function then find the condition continuous at every point in its domain. Following
for which f(x) is continuous at x = 1. are examples of some continuous function.
Sol.: f(x) is continuous at x = 1 (i) f(x) = x (Identity function)

83
CONTINUITY MATHS-X II-IIT-JEE
(ii) f(x) = c (Constant function)  4.2 Discontinuity of second kind:
(iii) f(x) = x2 If either of LHL and RHL or both does not exist
(iv) f(x) = a 0xn + a1xn–1+ ....+ an (Polynomial) then f(x) is said to have discontinuity of II kind.
(v) f(x) = |x|, x + |x|, x – |x|, x|x| In case lim f(x) does not exist then it is not possible
x c
(vi) f(x) = sin x, f(x) = cos x to make the function continuous by redefining it.
(vii) f(x) = ex, f(x) = ax, a > 0 Such discontinuities are known as non-removable
(viii) f(x) = log x, f(x) = logax, a > 0 discontinuity or discontinuity of the 2nd kind.
Irremovable type of discontinuity can be further
(ix) f(x) = sin h x, cos h x, tan h x
classified as-
4. Kinds of discontinuity (i) Infinite discontinuity e.g. f(x) = 1/(x – 4) or
 4.1 Discontinuity of 1st kind: If LHL and RHL g(x) = 1/(x – 4)2 at x = 4, h(x) = cot x has
both exist and are finite. infinite discontinuity at x = n, n  I, (x) = tan x,
Further classification of discontinuity of 1st kind also has infinite discontinuity at
is of two types:  2n  1 
x=   , n  I
1. Removable discontinuity  2 
2. Non-removable discontinuity (ii) Oscillatory discontinuity e.g. f(x) = sin 1/x at
x = 0. In all these cases the value of f(A) of the
1. Removable discontinuity: If LHL = RHL  f(a) function at x = a (point of discontinuity) may or
provided LHL & RHL both are unique and finite. may not exist but lim f(x) does not exist.
x a
OR
Note : From the adjacent graph note that
In case lim f(x) exists but is not equal to f(c) then
x c (i) f is continuous at x = –1
the function is said to have a removable (ii) f has isolated discontinuity at x = 1
discontinuity or removable discontinuity of the first (iii) f has missing point discontinuity at x = 2
kind. In this case we can redefine the function such (iv) f has non removable (finite type)
that lim f(x) = f(c) and make it continuous at x = c. discontinuity at the origin.
x c
y
Types of removable discontinuity of first kind can
be further classified- 2
(i) Missing Point Discontinuity.
where lim f(x) exists finitely but f(a) is not defined. 1
x a
–1
(1  x)(9  x 2 ) 1 2 x
e.g. f(x) = has a missing point
(1  x) Nature of discontinuity
discontinuity at x = 1. Examples :- 8
(ii) Isolated point discontinuity Redefine the function f(x) =[sin x] where x (0, π) in
Where lim f(x) exists and f(a) is defined and such a way that it could become continuous for x
x a
(0,  ).
lim f(x)  f(a)
x a  
2
Solution: Here lim [sin x]  0 but f    1 .
(x  16) x
 2
e.g. f(x) = ; x  4 and f(d) = 9 has a break 2
(x  4)

at x = 4. Hence, f(x) has a removable discontinuity at x = .
2
To remove this we redefine f(x) as follows
2. Non-removable discontinuity: If LHL  RHL
provided LHL and RHL are both unique and finite f(x) = [sin x], x (0,/2)  (/2,  ).
and the difference between LHL and RHL is called 
Now, f(x) = 0, x = is continuous for x(0,  ).
jump of discontinuity. 2

84
MATHS-X II-IIT-JEE CONTINUITY
Example - 9 7. Some important points
Prove that f (x) = {x} has non removable discontinuity at (i) When we say that the function f(x) is
continuous at a point x = a, if mean that at point
any xI.
(a, f(a)) graph is undertaken.
Solution: Since lim f (x) does not exist for any aI. (ii) Kinds of discontinuity
x a

Hence, f(x) = {x} has non-removable discontinuity at (A) lim– f(x) = lim f(x), then f is said to have
xa xa
any x  I non "removal discontinuity" of first kind.
5. Intermediate value theorem (B) lim– f(x)  lim f(x), then f is said to have
xa xa
 Suppose f(x) is continuous on an interval I, and a non removal discontinuity of first kind.
and b are any two points of I. Then if y0 is a number (C) At least one of lim– f(x) or lim f(x) does not
xa xa
between f(a) and f(b), their exists a number c exist then f is said to have discontinuity of 2nd
between a and b such that f(c) = y0. kind at x = a
(D) Continuity of composite function: If the function
6. Discontinuous functions u = f(x) is continuous at the point x = a and the
(i) f(x) = 1/x at x = 0 function y = g(u) is continuous the point u = f(A)
(ii) f(x) = e1/x at x = 0 then composite function = (gof)(x) = g(f(x)) is
(iii) f(x) = sin 1/x, f(x) = cos 1/x at x = 0 continuity at point x = a.
(iv) f(x) = [x] at every integer
(v) f(x) = x – [x] at every integer
(vi) f(x) = tan x, f(x) = sec x

when x = (2n + 1) , n  Z.
2
(vii) f(x) = cot x, f(x) = cosec x when x = n, n Z.
(viii) f(x) = cot x, f(x) = cosec x at x = 0.

85
CONTINUITY MATHS-X II-IIT-JEE

EXERCISE # 1
Based On Continuity 7. The set of points of continuity of the function f(x)
1
 1 =  cos2 x is
1. Let f (x)   x   [x] when – 2  x  2. Where 2
 2
[.] represents greatest integer function? Then   3 
(A) x :  2n  x   2n, n  I
(A) f(x) is continuous at x = 2  4 4 
(B) f(x) is continuous at x = 1  5 7 
(B) x :  2n  x   2n, n  I
(C) f (x) is continuous at x = –1  4 4 
(D) f(x) is discontinuous at x = 0   3 
(C) x :  2n  x   2n
2. The value of x where the function f(x) =  4 4 
tan x log(x  2)  5 7 
is discontinuous are given by:     x :  2n  x   2n
x 2  4x  3  4 4 
(A) (–, 2)  {3} (D) None of these
 8. The set of points for which
(B) (–, 2] {3, n + , n  N}
2 1
f(x) = | x | |x –1| + is discontinuous is-
(C) (– , 2) [x  1]
(D) None of these
(A) [–1, 1] (B) [–1, 0)  Z – {0}
 1 if x is rational (C) {–1, 0, 1} (D) All integral points
3. If f(x) =  is continuous on
1 if x is irrational Based On Discontinuity
(A) R (B) 
9. A function f(x) is defined as below
(C) –1, 1 (D) None of these
cos(sin x)  cos x
f (x)  , x  0 and f(0) = a f(x)
4. The set of all points where f(x) = sec 2x + cosec x2
2x is discontinuous is- is continuous at x = 0 if a equals.
(A) {n ; n = 0 , ± 1, ± 2 ......} (A) 0 (B) 4 (C) 5 (D) 6
n
(B) { ; n = 0, ±1, ±2 .......}
2 
 1
(2n  1)   tan (tan x) x  4
(C) { ; n = 0, ±1, ±2 .......} 10. If f (x)   , then jump of
4 
 [x]  1 x
(D) {n/4 ; n = 0, ±1, ±2 .......}  4
discontinuity is
5. f(x) = [sin x] + |x| is discontinuous (Here [ ]
represents greatest integer function)    
(A)  1 (B)  1 (C) 1  (D) 1 
4 4 4 4
(A) every where (B) at x = 3/2
(C) at x = – /2 (D) at infinite points 11. If the function

6. f(x) = [cosec x], where [x] represents greatest  x 2  (A  2)x  A


 ; x2
integer function - f (x)   x2
(A) f(x) is discontinuous at x = /2  2 x2

(B) f(x) is continuous at x = /2 is continuous at x = 2 then A =
(C) xLim
 / 2 f(x) does not exist (A) 0 (B) 1
(D) f(x) is continuous at x = 3/2 (C) 2 (D) None of these

86
MATHS-X II-IIT-JEE CONTINUITY

 36x  9x  4x  1 18. y = f(x) is a continuous function such that its


 , x0 graph passes through (a, 0) then
 2  1  cos x
12. If f(x) = 
log e (1  3f (x))
 lim is
 K , x0 x a 2f (x)
is continuous at x = 0, then K equals (A) 1 (B) 0 (C) 3/2 (D) 2/3
(A) 16 log2 log 3 (B) 16 2 log 6
3
(C) 16 2 log2 log 3 (D) none of these 19. The function defined by f(x) = (  1)[ x ]

(where [] denotes greatest integer function


4 1 1
sin    sin 2    1 satisfies)
13. If f(x) = x x is to be made 1/3
1
4  21 (A) discontinuous for x = n where n is any integer
cos    cos    1
x x (B) f(3/2) = 1
continuous at x = 0, then f(0) should be equal to - (C) f '(x) = 0 for –1 < x < 1
(A) 0 (B) 1
(C) 1/3 (D)1/2 (D) none of these
–1
14. f(x) = [tan x] where [·] denotes the greatest  x2
integer function, is discontinuous at -  , 0  x 1
    a
(A) – , 0 and (B) – , 0 and 20. The function f(x) =  a , 1  x  2 is
4 4 3 3
 2
(C) –tan 1, tan 1, 0 (D) None of these  (2b  4b) , 2  x  
 x2
Based On Miscellaneous
continuous for 0 < x <  then the most suitable
2
15. Let f(x) = Sgn (x) and g(x) = x (x –5x + 6). The values of a and b are -
function f(g (x)) is discontinuous at
(A) Infinitely many points (A) a = 1, b = –1 (B) a = –1, b = 1 + 2
(B) Exactly one points (C) a = –1, b = 1 (D) None of these
(C) Exactly three points
(D) No point
16. The set of all points of discontinuity of the 
 1  cos10x , x0
tan x log x  x2
function f(x) = contains 
1  cos 4x 21. If f(x) =  a , x  0 , then the
 n  
(A)  : n  Z  x
2  , x0
 625  x  25

 n 
(B)  : n  Q value of a so that f(x) may be continuous at
2 
x = 0 is -
 n 
(C) [– , 0]   : n  N  (A) 25 (B) 50
2 
(D) None of these (C) –25 (D) None of these
2
17. In [1, 3] the function [x +1] ; [x] denoting the
22. If f(x) = xP cos (1/x) ; x  0 and f(0) = 0. The
greatest integer function, is continuous -
(A) For all x except nine points condition for P (P  I) is___, which make
(B) For all x except four points function f(x) continuous at x = 0
(C) For all x except seven points (A) P > 0 (B) P < 0
(D) For all x except eight points (C) P = 0 (D) None of these

87
CONTINUITY MATHS-X II-IIT-JEE

 sin[x] 29. If [x] denotes the integral part of x and


 [x]  1 for x0
  
  sin [x  1]  sin [x  1] 
 f(x) = [x]   then-
  1  [x] 
 cos  / 2[x]
23. If f(x) =  for x  0  
 [x]
(A) f(x) is continuous in R

 (B) f(x) is continuous in all integral points
 k for x0 (C) f(x) is discontinuous at all integers
 (D) None of these
where [x] denotes the greatest integer less than 30. If f(x) is a continuous function  x  R and the
or equal to x, then in order that f(x) be  f (x) 
range of f(x) is (2, 26 ) and g(x) =   is
continuous at x = 0, the value of k is -  c 
(A) equal to 0 (B) equal to 1 continuous  x  R, then the least positive
(C) equal to – 1 (D) indeterminate integral value of c is, ([·] denoted greatest integer
function)-
24. Let f(x) = [2x3 – 5] where [ ] denotes the greatest (A) 2 (B) 3 (C) 5 (D) 6
integer function. Then number of points in [1, 2] 31. If f(x) be a continuous function for all real values
where the function is discontinuous, is -
of x and satisfies x2 + (f(x) –2)x + 2 3 – 3 – 3
(A) 14 (B) 13
(C) 10 (D) None of these f(x) = 0  x  R. Then f( 3 ) is equal to-
1
25. The number of points where f(x) = [sin x (A) 2– (B) 2(1 – 3 )
3
+ cos x] (where [·] denotes the greatest integer
function) x  (0, 2) is not continuous is- (C) 2 + 3 (D) Zero
(A) 3 (B) 4 (C) 5 (D) 6
2 x 4
32. If f(x) = , x 0 is continuous
sin(2x)
26. Let f(x) = sin 1 x + cos 1 x defined in [0,1] function at x = 0, then the value of f(0) is
then 1 1
(A)  (B) (C) 1 (D) 0
(A) f(x– 3) will be continuous in [3,4] 8 2
(B) f(x– 3) will be continuous in [0,1]
(C) f(x – 3) will be continuous in [0,1)  (x 3  x 2  16x  20)
 if x  2
(D) f(x– 3) will be continuous in (0, 1) 33. Let f(x)  (x  2)2
 k, if x  2
27. Let f(x) = [x3 – 3], where [·] denotes the greatest 
If f(x) is continuous for all x, then k is
integer function. Then the number of points in the
(A) 2 (B) 3 (C) 7 (D) 10
interval (1, 2) where the function is discontinuous,
is- 34. A discontinuous function y = f(x) satisfying x2 +
(A) 4 (B) 2 y2 = 4 is given by
(C) 6 (D) None of these  4  x 2 2  x  0

(A) f(x) = 
2
2 
 4x 0x2
28. Function f(x) = (sin 2x) tan 2x
is not defined at x =
  4  x 2 2  x  0
    
. If f(x) is continuous at x = then f   is (B) f(x) = 
2
4 4  4  4  x 0x2
equal to -
(A) 1 (B) 2 (C) f(x) = 4  x2  2  x  2
(C) 1/ e (D) None of these (D) f(x) =  4  x 2  2  x  2

88
MATHS-X II-IIT-JEE CONTINUITY
35. The function  5cos ecx    
  x  39. If f(x) =  cos x  3sin x  , x  ,   0
 2 2
 sin   , x  1  , x0
f(x) =   2  
 2 x  3 [x] , x  1 is continuous at x =0, then  will be

(A) e15 (B) e2 (C) 15 (D) 1.
(A) is continuous at x = 1
(B) is differentiable at x = 1
(C) is continuous but not differentiable at x = 1 40. Let f(x) = lim sin2nx, then number of point(s)
n 
(D) lim f (x) does not exist where f(x) is discontinuous is
x 1
(A) 0 (B) 1
(C) 2 (D) infinitely many
 x 2 cos e1/ x when x  0
36. Let f(x) =  . Then f(x) is
 1 when x  0 x for rational values of x in [0, 1]
41. If f(x) = 
(A) Discontinuous at x = 0 1  2x for irrational values of x in [0, 1]
(B) Continuous but not differentiable at x = 0
then number of points of continuity of f(x) is
(C) Differentiable at x = 0
(A) 0 (B) 1
(D) lim f(x) exist (C) 2 (D) infinite
x 0
n(1  ax)  n(1  bx)
42. The function f(x) = is not
37. The value of f(0), so that the function x
a 2  ax  x 2  a 2  ax  x 2 defined at x = 0. The value which should be
f (x)  becomes assigned to f at x = 0 so that it is continuous at x =
ax  ax
0, is
continuous for all x is given by
(A) a – b (B) a + b
(A) a (B) - a (C) a3/2 (D) -a3/2 (C) ln a – ln b (C) ln a  ln b

38. If the function f(x) = (x + 1)cotx is continuous at x = 43. Let f : [1, 10]  Q be a continuous function and
0, then f(A) = 10, then f(10) is equal to
(A) f(0) = 0 1
(B) f(0) = e (A) (B) 10
10
1 (C) 1 (D) can’t be obtained
(C) f (0) 
e
(D) not continuous at x = 0

89
CONTINUITY MATHS-X II-IIT-JEE

EXERCISE # 2
One or More Than One Correct Answer 6. Which of the following function(s) has/have
Question removable discontinuity at x = 1
Type Questions
1 x2  1
1 (A) f(x) = (B) f(x) =
1. Given the function f(x) = , the points of n | x | x3  1
1 x
1
discontinuity of the composite function 1 x x  1  2x
y = f(f(f(x))) are at x = (C) f(x) = 2 2  (D) f(x) =
x2  x
(A) 0 (B) 1 (C) 2 (D) –1
7. Which of the following function(s) defined below
  1  has/ have single point continuity.
2. Let f(x) =  x    for x > 0, where [·] denotes
  x  1 if x  Q
the greatest integer function, then f(x) is (A) f(x) = 
0 if x  Q
1
(A) f   = 0  x if xQ
2 (B) g(x) = 
1  x if x Q
3
(B) f   = 0 x if xQ
4 (C) h(x) = 
(C) Discontinuous at finite number of points 0 if xQ
(D) Discontinuous at infinite number of points. x if x Q
(D) k(x) = 
   x if xQ
 2x 3  3  
3  cot 1    for x  0
Given f(x)=    2
3.  x  
 8. If f(x) = x and g(x) = x – 1, then -
2 1/ x
 {x } cos (e ) for x  0 (A) fog is continuous on [0, )
where { } & [ ] denotes the fractional part and the (B) gof is continuous on [0,)
integral part functions respectively, then which (C) fog is continuous on [1, )
of the following statement does not hold good- (D) None of these
(A) f(0–) = 0
(B) f(0+) = 3 2n
9. If f(x) = lim (sin x) then f is -
(C) f(0) = 0  continuity of f at x = 0 n 
(D) irremovable discontinuity of f at x = 0 
(A) continuous at x =
4. Which of the following function(s) not defined 2
at x = 0 has/ have non- removable discontinuity 
(B) discontinuous at x =
at the origin- 2
1 1 3
(A) f(x) = 1/x
(B) f(x) = tan–1 (C) discontinuous at x =
1 2 x 2
1/x
e 1 (D) discontinuous at infinite number of points
(C) f(x) = 1/x (D) None of these
e 1 10. If the function
5. Which of the following function(s) not defined  x  a 2 2 sin x , 0  x   / 4

at x = 0 has/ have removable discontinuity at f (x)   x cot x  b , /4x /2
the origin- b sin 2x  a cos 2x ,  / 2  x  

1  | sin x | 
(A) f(x) = cos x
(B)f(x) = cos   is continuous in the interval [0, ] , then the
1 2  x 
values of (a, b) are
 1
(C) f(x) = x sin (D) f(x) = (A) ( 1,  1) (B) (0, 0)
x n | x |
(C) ( 1, 1) (D) (1, 1)

90
MATHS-X II-IIT-JEE CONTINUITY
11. A function f(x) is defined as  | x  1| ; x  0
A sin x  sin 2x 16. If f(x) =  and
f(x) = , (x  0). If the function is  x ;x0
x3
 | x | 1 ; x  1
continuous at x = 0, then g(x)=  then the number of jumps
(A) A = – 2 (B) f(0) = – 1   | x  2 |; x  1
(C) A = 1 (D) f(0) = 1 in f(x) + g(x) at point of discontinuities are -
(A) 1 (B) 2
12. The function f(x) = 1 + | sin x | is
(C) 3 (D) None of these
(A) continuous nowhere
(B) continuous everywhere Passage # 2 (Q.17 to Q.18)
(C) differentiable nowhere Let f: R  R be a function defined as,
(D) not differentiable at x = 0 1 | x |, | x |  1
f(x) =  &
 | x  3 |, x 1  0, | x | 1
 2
13. The function f(x) =  x 3x 13 is g(x) = f(x – 1) + f(x + 1),  x  R. Then
   , x 1
4 2 4 17. The value of g(x) is
(A) continuous at x = 1  0 , x  3

(B) differentiable at x = 1 2  x , 3  x  1
(C) continuous at x = 3   x , 1  x  0
(A) g(x) = 
(D) differentiable at x = 3
 x , 0  x 1
2  x , 1 x  3
Question Passage Based Type Questions 
 0 , x 3
Passage # 1 (Q.14 to Q.16)
A function f(x) is said to have jump discontinuity  0 , x  2
at a point x = a if both of the limits lim— f(x) and 
x a 2  x , 2  x  1
lim f(x) exists but not equal i.e. lim f(x)    x , 1  x  0
x a x a—
(B) g(x) = 
lim f(x) and f(a) may be equal to either of the  x , 0  x 1
x a 2  x , 1 x  2
above limits.The non-negative difference 
 0 , x2
between L.H.L and R.H.L is called jump of the
function y = f(x) at x = a. A function having a  0 , x0
number of jumps in a given interval is called 
2  x , 0  x 1
sectional or piecewise continuous function.
  x , 1 x  2
2
5  x x  1 (C) g(x) = 
14. If f(x) =  , then jump in the  x , 2x 3
x  4 x  1 2  x , 3 x 4

function f(x) is -  0 , 4x
(A) 3 (B) 6 (D) None of these
(C) 9 (D) None of these
18. The function g(x) is continuous for x 
15. The jump of the function at the point of (A) R – {0, 1, 2, 3, 4}
(B) R – {–2, –1, 0, 1, 2}
1  a1/ x (C) R
discontinuity of the function f(x) =
1  a1/ x (D) None of these
(a > 0) is -
(A) 4 (B) 2
(C) 3 (D) None of these

91
CONTINUITY MATHS-X II-IIT-JEE

EXERCISE # 3
Question Column Match Type Questions The number of value of x at
1.  which the function
Column I Column II 2x 5  8x 2  11
(D) f (x)  is (S) 1
x 4  4x3  8x 2  8x  4
 sin{x}; x  1
If f (x)   where Discontinuous
cos x  a; x  1
{.} denotes the fractional part 3.
(A) function, such that f(x) is (P) 1 Column I Column II
continuous at x = 1. P(x) = [2 cos x], is discontinuous
a (A) x [–,], then (P) at exactly 7
if | k| = then k is P(x) points
(4  )
2 sin If Q(x) = [2 sin x], is discontinuous
4
(B) x  [–, ] then Q(x) (Q) at exactly 4
If the function f (x)  (1  cos(sin
2
x))
points
(B) x (Q) 0
If R(x)= [2 tan x /2], has non-
is continuous at x = 0, then f(0) is removable
    ,then
 x, x  Q ,then the values (C) x  ,
 2 2 (R)
f (x)     discontinuities
1  x, x  Q R(x)
(C) (R) –1
of  x is continuous at
x at which f(x) is continuous If S(x)  3cosec  , infinitely many
(D)  3 (S)
If f(x) = x + {–x} + [x], where [x] values
  ,then S(x)
x , 2
and {x} represents integral and 2



(D) fractional part of x, then the (S) 1/2
values of x at which f(x) is is Question Numerical Type Questions
continuous
 cos x cot x 
 (cot x) .(cos x) , x  2
2. 4. If f(x) = 
 k, 
Column I Column II x
 2
If f(x) = 1/ (1– x), then the

(A) points at which the function (P) 1/2 is continuous at x = , then k is equal to
2
fofof(x) is discontinuous

1 5. Let f(x) be a continuous function defined for


f(u)  2
,where u  1 . 1  x  3. If f(x) takes rational values for all x
u u–2 x 1
(B) (Q) 0 and f(B) = 10, then f(1.5) = ………
The value of x at which ‘f’ is
discontinuous

f(x) = u2,
where u   x  1, x  0
 x  1, x  0
(C) (R) 2
The number of value of x at
which ‘f’ is discontinuous

92
MATHS-X II-IIT-JEE CONTINUITY

EXERCISE # 4
Question Previous Year (JEE Mains) 5. The values of p and q for which the function
1. f is defined in [–5, 5] as  sin(p  1)x  sin x
 ,x  0
 x
 x, if x is rational and 
f(x) =  . Then f(x) =  q ,x0
 –x,if x is irrational  2
 xx – x
[AIEEE-2002]  ,x0
x 3/ 2
(A) f(x) is continuous at every x, except x = 0 is continuous for all x in R, are [AIEEE-2011]
(B) f(x) is discontinuous at every x, except x = 0 3 1 1 3
(C) f(x) is continuous everywhere (A) p = – , q = (B) p = , q =
2 2 2 2
(D) f(x) is discontinuous everywhere 1 3 5 1
(C) p = , q = – (D) p = , q =
2 2 2 2
1 – tan x   
2. Let f(x) =
4x – 
,x ,x
4 0, 2  . 6. If : R R is a function defined by
 
 2x –1 
    f(x) = [x] cos   , where [x] denotes the
f(x) is continuous in 0,  , then f   is  2 
 2  2
[AIEEE-2004] greatest integer function, then f is
1 1 [AIEEE-2012]
(A) – (B) (C) 1 (D) –1 (A) discontinuous only at non-zero integral
2 2
values of x.
3. The function f : R – {0} R given by
(B) continuous only at x = 0
1 2 (C) continuous for every real x
f (x) =  can be made continuous at
x e2x  1 (D) discontinuous only at x = 0
x = 0 by defining f(0) as [AIEEE-2007]
7. Let f : R  R be a function defined as :
(A) 0 (B) 1 (C) 2 (D) –1
5, if x  1

4. Let f : R be a continuous function defined by a  bx, if 1  x  3
f (x)   Then, f is :
1 b  5x, if 3  x  5
f(x) = x .Statement-1 : f(c) = 1/3, 30,
e  2e – x if x  5

for some c  R. [January 2019]


(A) continuous if a = 5 and b = 5
1
Statement-2 : 0 < f(x)  , for all x  R. (B) continuous if a = –5 and b = 10
2 2 (C) continuous if a = 0 and b = 5
[AIEEE-2010] (D) not continuous for any values of a and b

(A) Statement 1 is true, Statement 2 is true; 8. Let f : [–1,3]  R be defined as


Statement 2 is a correct explanation for | x | [x],  1  x  1
Statement 1 f (x) =  x  | x | , 1  x  2
 x  [x] , 2  x  3
(B) Statement 1 is true, Statement 2 is true; 
where [t] denotes the greatest integer less than
Statement 2 is not a correct explanation for
or equal to t. Then, f is discontinuous at:
Statement 1 [April 2019]
(C) Statement 1 is true, Statement 2 is false. (A) four or more points (B) only one point
(D) Statement 1 is false, Statement 2 is true. (C) only two points (D) only three points

93
CONTINUITY MATHS-X II-IIT-JEE

  (A) (5/2, 1/2) (B) (–3/2, –1/2)


9. If the function f defined on  ,  by
6 3 (C) (–1/2, 3/2) (D) (–3/2, 1/2)

 2 cos x  1  13. Let [t] denote the greatest integer  t and


 , x
f (x)   cot x  1 4
4
  lim x    A . Then the function, f (x) = [x2]
k, , x x 0  x 
 4
sin (x) is discontinuous, when x is equal to :
is continuous, then k is equal to [April 2019]
(A) 1/2 (B) 1 (C) 1 / 2 (D) 2 [Jan. 2020]
(A) A5 (B) A 1
 a | (   x)  1, x  5
10. If the function f (x)   is (C) A (D) A  21
 b | x   | 3, x  5
continuous at x = 5, then the value of a – b is :
 sin (a  2) x  sin x
[April 2019]  ;x 0
 x
2 2 2 2 
14. If f (x)   b ;x0
(A) (B) (C) (D)
5 5 5 5  2 1/3 1/3
 (x  3x )  x ;x0
 x 4/3
x is continuous at x = 0, then a + 2b is equal to :
11. If f (x) = [x] –   , x  R , where [x] denotes
4 [Jan. 2020]
the greatest integer function, then : (A) –1 (B) 1 (C) –2 (D) 0
[April 2019]
 1 1
(A) Both lim f (x) and lim f (x) exist but are 15. If f (x) is defined in x    , 
x 4 x 4  3 3
not equal.  1   1  3x 
  log e  , x  0
f (x)   x   1  2x 
(B) lim f (x) exists but lim f (x) does not 
x 4 x 4  k , x0
exist. Find k such that f (x) is continuous.
[Jan. 2020]
(C) lim f (x) exists but lim f (x) does not
x 4 x 4 x
exist. 16. Let f (x) = x ·   , for –10 < x < 10, where [t]
2
(D) f is continuous at x = 4 denotes the greatest integer function. Then the
number of points of discontinuity of f is equal
to ________. [Sep. 2020]
 sin (p  1)  sin x
 , x0 17. If a function f (x) defined by
 x
 ae x  be x ,  1  x  1
12. If f (x)   q , x0  2
 f (x)  cx , 1 x  3
 x  x2  x ax 2  2cx, 3  x  4
 , x 0 
 x 3/2
be continuous for some a, b, c  R and
is continuous at x = 0, then the ordered pair
f ' (0) + f ' (b) = e, then the value of a is :
(p,q) is equal to : [April-2019]
[Sep. 2020]

94
MATHS-X II-IIT-JEE CONTINUITY
e e 21. Let [t] denote the greatest integer  t. The number
(A) (B)
2 2
e  3e  13 e  3e  13 of points where the function
1 e f (x) = [x] | x2 – 1| + sin – [x + 1], x  (–2, 2)
(C) (D)
2 2
e  3e  13 e  3e  13 is not continuous is _____. [JEE MAIN 2021]
18. Let [t] denote the greatest integer less than or Question Previous Year (JEE Advanced)
equal to t. Let f (x) = x – [x], g (x) = 1 – x + [x], 1
22. If f(x) = x  1 then on the interval [0, ],
and h (x) = min{f (x), g (x)}, x  [–2, 2]. 2

[JEE MAIN 2021] [IIT-1989]


(A) Not continuous at exactly four points in [– (A) tan [f(x)] and 1/f(x) are both continuous
2, 2]. (B) tan [f(x)] & 1/f(x) are both discontinuous

(B) Continuous in [–2, 2] but not differentiable (C) tan [f(x)] and f–1 (x) are both continuous
at more than four points in (–2, 2). (D) tan [f(x)] is continuous but 1/f(x) is not

(C) Not continuous at exactly three points in [– ( [.] is greatest integer function.)
2, 2].
23. The function f(x) = [x] cos {(2x – 1)/2},
(D) Continues in [–2, 2] but not differentiable [=]denotes the greatest integer function, is
at exactly three points in (–2, 2). discontinuous at [IIT 1195]
(A) all x
(B) all integer points
19. Let a, b,  R, b  0. Define a function (C) no x

(D) x which is not an integer

a sin 2 (x  1), for x  0
f (x)     
 tan 2x  sin 2x , for x  0 24. Let f(x) = [x] sin   , where [.] denotes
 bx 3  [x  1] 

If f is continuous at x = 0, then 10 – ab is equal the greatest integer function. The domain of f


to _______. [JEE MAIN 2021] is....... and the points of discontinuity off in the
domain are......... [IIT-1996]
20. If the function f (x) =
24. Let f(x) be a continuous function defined for
  x
1  1 a  1  x  3. If f(x) takes rational values for all x
 loge   , x0
x  1  x  and f(b) = 10, then f(1.5) = ……… [IIT-1997]
  b

 k , x0
 2
26. The function f (x) = [x]2 –[x2] (where [y] is the
2
 cos x  sin x  1 , x  0 greatest integer less than or equal to y), is

 x2 1 1
discontinuous at - [IIT–1999]

 (A) All integers
1 1 4 (B) All integers except 0 and 1
is continuous at x = 0, then   is equal to :
a b k
(C) All integers except 0
[JEE MAIN 2021]
(D) All integers except 1
(A) 5 (B) 4 (C) –4 (D) –5

95
CONTINUITY MATHS-X II-IIT-JEE
27. Determine the constants a, b and c for which the | x |, if x  0,
 f1 (x) =  x
 e , if x  0,
1/ x
(1  ax) ,x  0
 sin x, if x  0,
function f(x) =  b ,x  0 f2(x) = x2; f3(x) = 
 1/3  x, if x  0,
 (x  c) – 1 , x  0
 (x  1)1/ 2 – 1 f 2  f1  x   , if x  0,
and f4(x) = 
is continuous at x = 0 . [REE 99] f 2  f1  x    1, if x  0,
28. Discuss the continuity of the function
[IIT 2014]
x x List – I List – II
f(x) = , |x|  1, f(x) = , |x| < 1.
1 | x | 1 | x | P. f4 is 1. Onto but not one – one
Neigther continuous nor
[REE 2000] Q. f3 is 2.
one – one
Differentiable but not
R. f2 0f1 is 3.
29. Discuss the continuity of the function one – one
S. f2 is 4. Continuous and one - one
 e1/(x–1) – 2 P Q R S
 , x 1 (A) 3 1 4 2
f (x) =  e1/(x–1)  2 at x = 1.
 1 (B) 1 3 4 2
 , x 1 (C) 3 1 2 4
[REE 2001] (D) 1 3 2 4

30. For every integer n, let an and bn be real  1   1 


33. let f :   , 2  R and g :   2 , 2  R be
 2   
numbers. Let function f: IR  IR be given by
functions defined by f(x) = [x2 – 3] and g(x) =
 a  sin  x, for x  [2n,2n  1]
f(x) =  n , |x| f(x) + |4x–7| f(x), where |y| denotes the
b n  cos  x, for x  (2n – 1, 2n)
greatest integer less than oe equal to y for y 
for all integers n. If f is continuous, then which
R. Then [IIT 2016]
of the following hold(s) for all n? [IIT-2012]
(A) f is a discontinuous exactly at three points in
(A) an–1 – bn–1 = 0 (B) an – bn = 1
(C) an – bn+1 = 1 (D) an–1 – bn = –1  1 
  2 , 2
 
(B) f is a discontinuous exactly at four points in
31. Let f : [0,2]  R be function which is continuous
on [0,2] and is differentiable on (0, 2) with f(0) = 1.  1 
  2 , 2
 
x2
Let F(x) =  f  t  dt for x  [0, 2]. f ’(x) for (C) g is a discontinuous exactly at three points
0
 1 
in   , 2
all x Î (0, 2), then F(b) equals [IIT 2014]  2 
2 4
(A) e – 1 (B) e – 1 (C) e – 1 (D) e4 (D) g is not discontinuous exactly at five points

 1 
in   , 2
32. Let f1 : R  R, f2 : [0, ]  R, f3 : R  R and  2 
f4: R  (0, ) be defined by

96
MATHS-X II-IIT-JEE CONTINUITY
34. Let f: R  (0, 1) be a continuous function. 35. Let [x] be the greatest integer less than or
Then, which of zero at some point in the equals to x, Then, at which of the following
interval (0, 1)? [IIT 2017] point(s) the function f(x) = x cos ((x + [x])) is
(A) x9 – f(x) discontinuous? [IIT 2017]
n (A) x = 2 (B) x = 1
2
(B) f (x)   f (t) sin tdt (C) x = –1 (D) x = 0
0

n
x
2
(C) x   f (t) cos tdt
0

x
(D) e x   f (t) sin t dt
0

97
CONTINUITY MATHS-X II-IIT-JEE

ANSWER KEY

EXERCISE-1
Qus. 1 2 3 4 5 6 7 8 9 10 11 12 13 14 15 16 17 18 19 20
Ans. D B B D D B C B A C A C B C C D D C A C
Qus. 21 22 23 24 25 26 27 28 29 30 31 32 33 34 35 36 37 38 39 40
Ans. B A A B C A C C C D B A C A C A B B A D
Qus. 41 42 43
Ans. B B B

EXERCISE-2

Qus. 1 2 3 4 5 6 7 8 9 10 11 12 13 14 15 16 17 18
Ans. A,B B,D B,D A,B,C B,C,D B,D B,C,D B,C B,C,D B,D A,B B,D A,B,C C B B B C

EXERCISE-3
[MATCH THE COLUMN]
1. A  P,R; B  S; C  S; D  P, Q, R 2. A  Q, S; B  P, R, S; C  Q; D  Q
3.  P,R, S; B  P, R, S; C  Q, R, S; D  R, S 4. k = 1
5. 10

EXERCISE-4

Qus. 1 2 3 4 5 6 7 8 9 10 11 12 13 14 15 16 17 18 19 20
Ans. D A B A A C D D A A D D 2 D 5 8 4 B 14 D
Qus. 21 22 23 24 25 26 27 28 29 30 31 32 33 34 35
Ans. 2 B C D B,D B D B,C A,C A,B,C

98

You might also like